Conor Gillespie, Ameer Khan - Neurology and Neurosurgery - 200 SBAs For Medical Students-World Scientific Pub Co Inc (2022)
Conor Gillespie, Ameer Khan - Neurology and Neurosurgery - 200 SBAs For Medical Students-World Scientific Pub Co Inc (2022)
NEUROSURGERY
200 SBAs for Medical Students
Conor Gillespie
Ameer Khan
University of Liverpool, UK
NEW JERSEY • LONDON • SINGAPORE • BEIJING • SHANGHAI • HONG KONG • TAIPEI • CHENNAI • TOKYO
For photocopying of material in this volume, please pay a copying fee through the Copyright Clearance Center,
Inc., 222 Rosewood Drive, Danvers, MA 01923, USA. In this case permission to photocopy is not required from
the publisher.
Printed in Singapore
Dedications
To Alex, Ben, and Sam, thank you for your unwavering support and friendship. Abed,
I couldn’t have wished for a better mentor. Natasha, thank you for helping me get through a
difficult time. I’ll always be grateful.
— Conor Gillespie
To my dearest Mum and Dad and all my family and friends who have pushed and supported
me to get to this position in life.
— Ameer Khan
Foreword
This book presents a series of 200 questions covering neurology and neurosurgery topics for
medical students taking their final exams. As clinical neuroscience can be a daunting topic for
medical students, this book aims to cover all the important topics with a clear explanation
of the correct answer to each question. The book was conceived and prepared by a dedicated
medical student (Conor Gillespie) with input from other medical students, junior doctors,
and consultants — all with the aim of making the book as relevant and as accessible as possible
to those with ‘neurophobia’. To this end the goal has been achieved, and this book should sit
in every medical school library and medical student book shelf. We must remember that the
people who gain the most from good clinical neuroscience knowledge are our patients.
Preface
The goal of this book is simple. Use this, and never have to worry about clinical neuro again.
The Single Best Answer (SBAs) in this book are a mixture of core knowledge and challenging
questions, all designed for medical students approaching the final years of their course. One
of the most reported reasons why students find neuro difficult is its depth, so we have sought
to ameliorate this problem with focused questions and unique ‘one-sentence summaries’ of
each neuro condition containing almost all the vital information you need to know for the
purposes of exams and being a good clinician, and nothing more. A shape system is also in
place detailing which topics are essential for exams (circle), less essential (square), and for those
hoping to distinguish themselves clinically (triangle).
My tip for the self-confessed neurophobe, who hopes that no questions on neurology
or neurosurgery come up in exams, would be: just read this book; it has been tailored to this
purpose.
The book has been divided into 5 question papers. The papers get progressively more
difficult and incorporate some of the answers to the previous papers, ensuring dynamic
ascertainment and aggregation of knowledge throughout.
Our advice is to go through the first paper and then the answers, as these come in useful
for answering questions in the papers that follow, aiding memory recall. At the end of the book
is a glossary of one-sentence summaries about each condition, which form the core essentials
to be referred to.
There are a few things not covered in this book, namely ENT and neuroanatomy. A lot
of books do neuroanatomy better than I ever could, and I’ve never really been sharp at ENT
to be honest!
This book took forever and a day to finish, so I really hope you like it!
— Conor Gillespie
Senior authors:
Rhys Davies MA BMBCh, PhD, FRCP
Consultant Neurologist and Honorary Senior Lecturer, The Walton Centre NHS Foundation
Trust, Liverpool, UK
Viraj Bharambe BM BSc (Hons) MRCP
Neurology specialist registrar, The Walton Centre NHS Foundation Trust, Liverpool, UK
Benedict D. Michael MBChB, MRCP, PhD
Senior Clinician Scientist Fellow and Honorary Consultant Neurologist, University of
Liverpool, Walton Centre NHS Foundation Trust, Liverpool, UK
Abdurrahman Islim MPhil, MBChB
Academic Foundation Doctor, Royal Liverpool and Broadgreen NHS Foundation Trust,
Liverpool, UK
Ali Bakhsh MRCS, BMBS (dist), MSc, BSc (Hons), FHEA
Academic Clinical Fellow in Neurosurgery, the Walton Centre NHS Foundation Trust,
Liverpool, UK
Christopher Paul Millward MRCS, MSc, MBBS, BSc
Neurosurgery Trainee, The Walton Centre NHS Foundation Trust, Liverpool, UK
Michael D. Jenkinson MBChB (Hons), PhD, FRCSEd (Neuro Surg)
Professor of Neurosurgery and Sir John Fisher / RCSEng Chair of Surgical Trials, University of
Liverpool, The Walton Centre NHS Foundation Trust, Liverpool, UK
Acknowledgments
The authors would like to thank all of the brilliant staff at the Walton Centre NHS Foundation
Trust — without their mentorship, inspiration and support, this book would not have been
possible.
The front cover of this book was designed by rawpixel.com / Freepik.
Disclaimer
This book is for educational purposes only and should not be used to directly guide clinical
practice. Many of the things reported in this book may change as guidelines evolve. If ever in
doubt, please use your local trust/hospital guidelines.
Contents
Dedicationsv
Forewordvii
Prefaceix
About the Authorsxi
List of Editors and Illustratorsxiii
Chapter Editors and Reviewersxv
Acknowledgmentsxvii
Disclaimerxix
Practice Paper 1 1
Practice Paper 2 17
Practice Paper 3 33
Practice Paper 4 47
Practice Paper 5 63
Answers77
Practice Paper 1 — Answers 79
Practice Paper 2 — Answers 101
Practice Paper 3 — Answers 123
Practice Paper 4 — Answers 139
Practice Paper 5 — Answers 157
One Sentence Summaries Index (in Alphabetical Order) 167
Index175
Practice Paper 1
A. Tension headache
B. Glioblastoma
C. Idiopathic intracranial hypertension
D. Cavernous sinus thrombosis
E. Chronic subdural haematoma
6. A 28-year-old male who is normally fit and well presents in a dishevelled state.
His wife tells you he reported headaches for the past week, with the worst ones
occurring first thing in the morning. In the past 4 days, he has developed a fever
of 39.5°C and left leg weakness, and he is confused. His past history is
unremarkable apart from having occasional migraines, and he recovered from an
episode of sinusitis 2 weeks ago. He has not travelled abroad recently and does
not use intravenous drugs. A CT scan shows a round, ring-enhancing lesion in
the frontal lobe. What is the most likely diagnosis?
A. Encephalitis
B. Meningitis
C. Central nervous system lymphoma
D. Cerebral abscess
E. Tuberculosis
7. A 65-year-old male presents to the general practitioner with bilateral leg pains.
He has a 15-year history of poorly controlled type 2 diabetes and benign prostatic
hyperplasia. He describes the pain as ‘dull’ and ‘like an electric shock’, and it is
stopping him from sleeping. Which of the following agents should be avoided
when treating this man’s pain?
A. Pregabalin
B. Duloxetine
C. Amitriptyline
D. Gabapentin
E. Paracetamol
8. A 23-year-old male presents to the A&E after an assault. On examination, he opens his
eyes when a trapezius squeeze is applied, and he is groaning but not formulating any
words. When you apply a pain stimulus to his shoulder, he pulls his limb away from
the stimulus. What is the likely score on the patient’s Glasgow Coma Scale?
A. 6
B. 7
C. 8
D. 9
E. 10
10. A 36-year-old female attends the A&E department with a 2-hour history of sudden
onset headache, which she describes as like ‘being hit over the head with a baseball
bat’. The on-call neurologist suspects a subarachnoid haemorrhage. Her CT head
scan at 2 hours after onset is reported as normal by a consultant radiologist. What
is the most appropriate next investigation to undertake?
A. CT angiography at 24 hours
B. Repeat CT head scan at 12 hours
C. Repeat CT head scan at 2 hours
D. Lumbar puncture immediately
E. Lumbar puncture at 12 hours
11. A 65-year-old retired army sergeant attends the GP with his wife. She reports that
for the past two years, he has appeared lethargic, lost significant muscle mass
compared to his army days, and has had problems swallowing food. On
examination, the patient appears emaciated with widespread muscle wasting, and
you note the presence of fasciculations on his tongue. His reflexes are weak, he
has a positive Babinski sign in both legs, and his sensory function is intact. What
is the most likely diagnosis?
12. A patient is referred to a neurology outpatient clinic with memory loss. After
taking his history, the doctor suspects normal pressure hydrocephalus. What is
the triad associated with this presentation of symptoms?
13. A 57-year-old male is diagnosed with a brain tumour after presenting with a
2-month history of headaches. The tumour is removed and diagnosed as a brain
metastasis. Which cancer is the most common primary source of brain metastases?
A. Breast
B. Melanoma
C. Kidney
D. Lung
E. Liver
15. A 23-year-old male patient attends an epilepsy review, diagnosed after having
two tonic-clonic seizures one year ago. While his seizures have stopped since
commencing medication, he complains of unintentional weight gain of 8kg since
starting them. Which anti-convulsant medication is most associated with weight
gain?
A. Sodium valproate
B. Levetiracetam (Keppra®)
C. Lamotrigine
D. Carbamazepine
E. Phenytoin
A. Migraine
B. Tension headache
C. Cluster headache
D. Space-occupying lesion
E. Medication overuse headache
17. A 31-year-old female dental nurse attends the GP. For the past two weeks she has
been extremely troubled by intense left-sided facial pain. The pain originates from
the angle of her jaw and travels to the corner of her mouth, and she describes the
pain as ‘like little electric shocks’. She has noticed that area of her face is now
extremely sensitive, and the pain can be brought on by touching the area. She is
normally fit and well, has no allergies, and does not take any medication. The
week before, she went for a check-up with her dentist, who gave her the all-clear.
What is the most likely diagnosis?
18. For the previous question (17), assuming there are no contraindications, what is
the first line treatment to start the patient on?
A. Lithium
B. Propranolol
C. Primidone
D. Carbamazepine
E. Sodium valproate
19. A 73-year-old male presents to the A&E department. 4 hours earlier, his wife
noticed that, while watching TV, his speech suddenly became slurred and he could
not move his left arm. He has a past medical history of hypertension and type 2
diabetes, for which he currently takes ramipril and gliclazide respectively. An
acute stroke is suspected. What is the most appropriate initial investigation for
this patient (whilst in the emergency department)?
20. The concerned parents of a 9-month-old baby bring him to a GP. Over the past
2 months, they describe the child as having frequent episodes of flexing his arms
towards his chest before straightening them out, whilst drawing his knees up to
his chest. On examination, he cannot sit upright, has poor head control, and does
not babble or say any words. They had attended a GP appointment 3 months ago
and the locum GP diagnosed him with colic, but this has not resolved. What is
the most likely diagnosis?
21. You are an F1 doctor doing a ward round on a stroke ward. The stroke consultant
asks the next patient, who is recovering from a left partial anterior circulation
stroke, how he is doing. The patient replies, ‘Nice very breakfast my own horse
but then spaceships purple today’. What best describes the patient’s speech issue?
A. Broca’s aphasia
B. Wernicke’s aphasia
C. Conduction aphasia
D. Dysarthria
E. Dysphagia
22. Which of the following is NOT a risk factor for Carpal tunnel syndrome?
A. Female gender
B. Pregnancy
C. Multiple sclerosis
D. Rheumatoid arthritis
E. Hypothyroidism
23. You see a 48-year-old homeless male with a history of alcohol misuse and chronic
pancreatitis, who was brought into the ward after being found confused by a
friend. His friend reports that he had not eaten or drank anything for days and
is confused and struggling to walk. You suspect that he may have Wernicke’s
encephalopathy. What is the triad for Wernicke’s encephalopathy?
A. Autosomal dominant
B. Autosomal recessive
C. X-linked recessive
D. X-linked dominant
E. Mitochondrial
25. You are in clinic with a neurology registrar, during which he gets bored and decides
to grill you on your neuroanatomy. He asks which dermatome supplies the ventral
aspect of the little finger. What is the correct answer?
A. C6
B. C7
C. C8
D. T1
E. T2
26. In the same clinic with the neurology registrar, you examine a patient with
suspected degenerative cervical myelopathy, referred for her first specialist
appointment by her GP. The neurology registrar holds the patient’s hand and
flicks the nail of the middle finger, and you notice the patient’s thumb and index
finger flex spontaneously. What is the sign being elicited?
A. Babinski’s sign
B. Hoover’s sign
C. Hoffman’s sign
D. Brudzinski’s sign
E. Lhermitte’s sign
27. You are a final year medical student with your own clinic in a GP surgery. Your
next patient is a 70-year-old male. For the past 6 months, he appears to have
slowed down, taking much longer to get dressed in the morning and return from
the shops. On further questioning, he trivially reports losing his sense of smell 2
years ago, and his handwriting appears much smaller than it used to. On
examination, you notice a 3–5 Hz tremor that disappears when he moves his arm,
and he has a fixed, limited facial expression. What is the most likely diagnosis?
A. Muscle wasting
B. Fasciculations
C. Diminished or absent reflexes
D. Positive Babinski’s sign
E. Hypotonia
29. A 75-year-old female is brought by ambulance to the A&E after falling down a flight
of stairs while celebrating a friend’s diamond wedding anniversary. She has a past history
of alcohol misuse and atrial fibrillation, for which she takes apixaban 5mg once daily.
Her only other regular medication is atorvastatin 20mg once nightly. She appears
confused, and the paramedics hand over that she has a GCS of 10. A CT head scan
shows a hyperdense, crescent-shaped opacity that crosses suture lines with no skull
fractures. What is the most likely diagnosis?
30. A 2-week-old baby is referred to the neurosurgical team after being found to have
a rapidly enlarging head circumference, which is now measured to be in the 98th
percentile. She was born prematurely at 32 weeks due to foetal distress and suffered
an intraventricular haemorrhage at 2 days old. The paediatric neurosurgery
registrar suspects hydrocephalus. Which of the following examination signs will
NOT be present?
31. Ben, a 22-year-old medical student, attends his GP in a very concerned state. For
the past 2 months, he has complained of feeling dizzy, often after standing up.
He is very worried that he might have an arrythmia, and he has read all about
them as part of his studies. His lying and standing blood pressure measurements
are within normal range, and his 7-day ambulatory ECG reveals 8 ectopic beats
along with sinus rhythm. He does not take these results well. In his notes over the
last 4 years, he has undergone checks at a genitourinary medicine clinic with an
eventual diagnosis of non-gonococcal urethritis, was investigated for dysphagia
with no pathological cause found, and had a benign skin mole removed after
expressing concern. What is the most likely diagnosis?
A. Somatisation disorder
B. Arrythmia
C. Munchausen’s syndrome
D. Hypochondriac disorder
E. Acquired immunodeficiency
32. Mohammed, a 72-year-old retired pilot presents to his GP with a sudden onset
of back pain for the last 2 days. He describes having a pain ‘like an elastic band’
in his upper abdomen and back for 8 weeks, but did not seek treatment because
he ‘did not want to be a bother’. He is normally fully mobile, but for the last 2
days has found getting about the house difficult. He has no urinary incontinence.
He has a past history of prostate cancer, and he underwent radiotherapy 12 months
ago for a recurrence. On examination, he has MRC grade 2/5 power in both lower
limbs. What is the most appropriate immediate management for this patient?
33. A patient is brought into the A&E after suffering a stab wound to the back during
a fight in the city centre. On examination, the patient has considerable neurological
deficits and a right-sided stab wound. The consultant suspects a diagnosis of
Brown-Séquard syndrome. What examination findings would you expect?
34. You are on rotation at a medical ward when one of the senior house officers
suggests that you examine the eyes of a 65-year-old patient. On examination, the
patient’s right eyelid is significantly lower than that of the left, and his right pupil
is fixed and unreactive to light. When the patient is asked to look straight ahead,
the left eye looks forward, but the right eye appears located in the right lower
quadrant of the eye. Which cranial nerve is affected?
35. Kyle, a 12-year-old boy, is brought to his GP by his concerned parents. They have
noticed that over the last few months, his performance in school has steadily
declined. He used to be at the top of his class for mathematics, but now he struggles
to do basic sums. His mum reports that he forgot the names of his friends when
they came over to play football last week, and he has become a lot more quiet. On
examination, the patient has a resting tremor, and you note the presence of a
golden-brown ring around his eyes. He is normally fit and well, and there is no
family history. What is the most likely diagnosis?
36. Reginald, a 70-year-old retired navy captain, comes to see his GP. He apologises,
saying his wife made him come in. In the last 6 months, his shoe size has gone up
three sizes, and he has noticed his wedding ring no longer fits on his finger, but
he denies recent weight gain. His past medical history includes hypertension
resistant to 3 anti-hypertensive medications and bilateral carpal tunnel syndrome.
On examination, you note prognathism and a very large tongue. What is the most
likely visual deficit you will see on examination?
A. Unilateral blindness
B. Bitemporal superior hemianopia
C. Bitemporal inferior hemianopia
D. Left-sided homonymous hemianopia
E. Bilateral homonymous hemianopia with macular sparing
37. A 55-year-old man has been at the intensive care unit for the past 6 weeks after
being involved in a high-speed motor vehicle accident. He was not wearing a
seatbelt and he had a GCS of 3 at the scene. His recovery has been slow and he is
unable to be taken off the ventilator. A CT head scan shows small punctate
contusions but is otherwise normal. What is the most likely diagnosis?
A. Extradural haematoma
B. Coning due to raised intracranial pressure
C. Hydrocephalus
D. Diffuse axonal Injury
E. Subdural haematoma
38. You are in the A&E as a foundation doctor and the consultant shows you the
image below. What is the most likely diagnosis?
A. Hydrocephalus
B. Acute extradural haematoma
C. Normal scan
D. Subarachnoid haemorrhage
E. Acute subdural haematoma
39. On the same shift, another F1 approaches you and asks you to help them interpret
another scan. This patient was admitted to their acute medical ward after being
found unresponsive at home, and he has a significant focal neurological deficit.
Based on their CT scan taken 2 hours ago, what is the most likely diagnosis?
40. A 38-year-old scaffolder attends the A&E department after an accident at work
where 12 bricks fell on his neck, causing it to bend downwards. He noticed
immediate weakness and altered sensation afterwards. On examination, he has
MRC grade 2/5 power in both legs, but grade 4/5 power in both arms and grade
5/5 power in the rest of the upper limbs. He cannot feel a pinprick on his legs,
but can recognise when a tuning fork is placed on them. What is the most likely
diagnosis?
Practice Paper 2
1. Mary is a 78-year-old female who was referred by her GP to the movement disorders
clinic with a suspected diagnosis of Parkinson’s. She has noticed that, over the last
6 months, she has developed a resting tremor that improves with movement, and
she has slowed significantly. On taking a more detailed history, she reports that
before her symptoms started, she began suffering from frequent falls and now
uses a wheelchair to prevent them from happening. On examination, she has
limited eye movements and cannot move them upwards, as well as a fixed facial
expression with the appearance of looking surprised. What is the most likely
diagnosis?
2. Annie is a 36-year-old Caucasian female who has presented to her GP. Last week,
she noticed a gradual onset of left eye pain with associated blurred vision, which
has now thankfully settled down. She has a past medical history of hypothyroidism
and depression, and is taking thyroxine and sertraline for these respectively. She
reports no headaches or previous episodes and no other symptoms, although she
does report having a sudden onset of numbness with mild weakness of her right
leg for 2 days in the previous year. On examination, her vision is normal. What
is the most likely diagnosis?
A. Amaurosis fugax
B. Primary angle glaucoma
C. Multiple sclerosis
D. Conversion disorder
E. Space-occupying lesion
A. Riluzole
B. Dexamethasone
C. Prednisolone
D. Tetrabenazine
E. Natalizumab
4. Gerry, a 34-year-old male patient at your GP clinic, has recently been diagnosed
with Huntington’s. The inherited nature of the condition has been explained to
him. After being quite reserved throughout the consultation, he tells you that his
wife is currently 28 weeks pregnant with their first child, a baby boy. He is worried
about passing the condition on to his son and asks you what his son’s chances are
of inheriting the disease. His wife is not affected by the condition. What do you
tell him?
5. A 25-year-old PhD student attends the epilepsy clinic after having a suspected
first seizure. This was noticed by her partner, who describes finding her ‘shaking
her limbs and then going stiff’ for ten minutes in the morning. The episode resolved
spontaneously, and she had no urinary or faecal incontinence, bit the centre of
her tongue, and was noticeably tearful immediately afterwards. He describes her
eyes as being clenched shut during the episode. She has no past medical history,
but reports being ‘stressed out’ as her PhD thesis is due next week. What is the
most likely diagnosis?
A. Call the anaesthetic team for immediate support before commencing any
management
B. Give IV phenytoin 15–25 mg/kg after setting up a large bore cannula
C. Give rectal diazepam 10 mg
D. Give IV lorazepam 4 mg
E. Wait another 20 minutes before intervening
7. You are in a GP and your next patient is a 28-year-old female who is 12 weeks
pregnant. For the past 6 weeks, she has been troubled by intense headaches. They
happen once every few days and are dull and achy in nature. During an episode,
she lies still and tries to go into a dark room, which sometimes helps. There are
no associated visual disturbances and her blood pressure on examination is normal.
The headaches impact her work as a primary school teacher and she is desperate
to resolve them. Apart from well-controlled asthma with a salbutamol inhaler,
she is well. What medication would you offer her to prevent her headaches?
A. Propranolol
B. Topiramate
C. Amitriptyline
D. Metoclopramide
E. Codeine phosphate
8. You are the GP of Jean, a 78-year-old retired teacher from Wales. One day, she
comes to you with a 2-week history of troublesome headache. The headache
presented over a few days and is 8/10 in intensity. She also describes feeling tired
and weak during the same period. She has noticed that the pain is brought on by
combing her hair, and she has had some difficulty chewing foods which she thinks
is unrelated. Bloods have already been taken and sent off to test for markers that
would support the diagnosis. What is the most appropriate management step?
9. A 35-year-old female patient presents to the neurology clinic. For the past
6 months, she has been feeling extremely tired and weak, most often at the end
of her long day working as an accountant. She also complains of double vision,
which occurs exclusively in the evenings. She had seen an optometrist who ruled
out ophthalmological causes. On examination, she has MRC grade 5 power
throughout initially; however, after flexing her arm 3 times, her power reduces to
MRC grade 3. What is the most likely diagnosis?
A. Myaesthenia gravis
B. Lambert-Eaton myaesthenic syndrome
C. Multiple sclerosis
D. Amyotrophic lateral sclerosis
E. Polymyalgia rheumatica
10. You are in the A&E as a medical student and you see the consultant rush to a new
patient in triage, thought to be having an acute stroke. While the patient is getting
their CT scan, the registrar asks you, how long, after symptom onset, are patients
eligible to receive thrombolysis and thrombectomy, respectively. What would the
answer be?
11. You are shadowing doctors as part of the acute medical unit ward round and come
across the same patient you saw in the A&E the previous day (Question 10). The
patient was not eligible for thrombolysis and suffered a large stroke. On
examination, the patient’s speech is slurred, and they appear frustrated when
trying to answer the consultant’s questions. They have a dense hemiplegia affecting
the left-hand side of their whole body, and the consultant notes left-sided
homonymous hemianopia on examination. According to the Bamford
classification, what type of stroke do they have?
12. A 25-year-old male railway engineer attends a GP surgery with his concerned
parents. 8 weeks ago, he was working on the tracks when part of the track flew
up and hit him on the head. Before the accident, he was very mild mannered, but
since then he has become more aggressive, been in several bar fights, and lost ten
thousand pounds of savings on slot machines. The patient does not appear
concerned, swearing excessively throughout the consultation and demanding that
he be allowed to leave as ‘there is nothing wrong with me’. Which part of the brain
is most likely affected?
13. An 82-year-old male sheepishly walks into a memory clinic with his wife. She is
concerned about his memory, which has gradually declined over the past 2 years.
He was an avid solver of newspaper crossword puzzles for many years, but now
he cannot come up with a single answer. She has also noticed that, occasionally
at night, he fails to recognise who she is even though they have been married for
50 years. He has worked as a bus driver all his life and is fit and well. On
examination, he makes an active effort to address all questions asked, and his
Montreal Cognitive Assessment score is 17/30. What is the most likely diagnosis?
A. Peripheral neuropathy
B. Charcot-Marie-Tooth disease
C. Space-occupying lesion
D. Guillain-Barré syndrome
E. Brown-Sequard syndrome
15. You are in your final year OSCE assessment and have been asked to carry out a
neurological motor examination. The patient’s right side has normal power, but
both the left arm and leg are weak. The patient cannot raise their arm or leg when
asked to, but there is a flicker of contraction and the patient can flex the arm and
leg when both are placed flat on the examination couch. The examiner asks you:
What MRC grade power does this patient have on the left side?
A. Grade 5
B. Grade 4
C. Grade 3
D. Grade 2
E. Grade 1
A. Dystonia
B. Akathisia
C. Tardive dyskinesia
D. Hemiballismus
E. Myoclonus
18. An 18-year-old male from a travelling family presents to his local GP. For years,
his parents have noticed irregular lumps throughout his whole body, but did not
seek medical attention as he was asymptomatic and is otherwise well. On
examination, he has several bumpy lesions occupying a dermatomal pattern in
his upper limbs, strange freckles over his axilla, and a series of light, brown macules
on his lower back. What is the most likely diagnosis?
A. Tuberous sclerosis
B. Sturge-Weber syndrome
C. Neurofibromatosis type 1
D. Neurofibromatosis type 2
E. Adenoma sebaceum
19. Whilst in clinic examining a patient with intractable vomiting, the consultant
decides to test your anatomy and asks: in what region of the brain is the vomiting
centre located?
A. Midbrain
B. Pons
C. Medulla
D. Area postrema
E. Cerebrum
21. Adel, a 54-year-old male, attends the neurosurgical clinic after an incidental mass
was found during an MRI brain scan as part of a private health check. He is
asymptomatic. The consultant explains to you that it is likely a tumour, located
on the parietal convexity, with no brain invasion. The tumour is spherical and
calcified. What is the most likely cause of his lesion?
A. Meningitis
B. Encephalitis
C. Post-coital headache
D. Subarachnoid haemorrhage
E. Benign thunderclap headache
23. Oliver, a 61-year-old semi-retired neurologist, comes to see you. He has been
struggling over the past few months to use both his hands. He finds it difficult to
do his buttons up in the morning, and finds it even harder to type up patient
notes on the ward computers. He also says that the strength in his arms is ‘a lot
less than it used to be’. On examination, there is reduced power in the upper arms
and muscles of the hand, and there is a positive Hoffman’s sign. What is the most
likely diagnosis?
24. Jennie, an 82-year-old female, visits the GP for her first appointment in ten years
(she ‘doesn’t like to be a bother’). Over the last 24 hours, she has been troubled
by terrible back pain, which came on without any trigger. Since then, she reports
leg weakness and could not get out of her chair this morning, so she asks the GP
for her first appointment in ten years. She reluctantly admits that she has had ‘a
few accidents’ with her bowels and urine over the same period. On examination,
she has reduced power in the legs and cannot feel any pinprick sensation in the
peri-anal area. What is the most likely diagnosis?
25. A 53-year-old male who has a known diagnosis of glioblastoma and is being
managed with the best supportive care presents to the A&E department, acutely
unwell with a reduced consciousness level. He was found by his ex-wife, and you
have little history. On examination, his GCS is 8, breathing is regular, right pupil
is fixed, dilated and unresponsive to light, and entire left-hand side is affected by
a dense hemiparesis. What complication has the patient developed?
A. Tonsillar herniation
B. Subfalcine herniation
C. Hydrocephalus
D. Uncal (transtentorial) herniation
E. New onset stroke
26. A 27-year-old female who is recovering from a bad episode of sinusitis presents
to the A&E with painful eye movements and double vision. She is normally fit
and well. On examination, there is significant eyeball protrusion and oedema
surrounding both eyes. She has loss of forehead sensation bilaterally, and
ophthalmoscopy reveals papilloedema. What is the most likely diagnosis?
A. Pituitary apoplexy
B. Cavernous sinus thrombosis
C. Intracerebral malignancy
D. Idiopathic intracranial hypertension
E. Posterior communicating artery aneurysm
27. A 45-year-old male security guard collapsed at work and was brought to the A&E.
He had complained to his colleagues of headaches that are most severe in the
morning over the last 3 months as well as occasional double vision, but was
determined to ‘soldier on’ and not seek medical advice. An emergency CT scan
identifies a suspicious mass lesion in the frontal lobe with very prominent oedema
around the mass. What medication should be given to reduce the oedema and
help with his symptoms?
A. Nimodipine
B. Morphine sulphate
C. Dexamethasone
D. Mannitol
E. Acetazolamide
28. A 52-year-old male is brought into the emergency department by his partner, who
is concerned about his behaviour. For the past 2 months his memory has declined
significantly and he has forgotten to pay his bills, become very ‘amped up’
lately — sleeping for only 2 hours a night — and become unsteady on his feet.
He has no prior history of mental health problems, although he had a stroke ten
years ago and a sudden widespread body rash that was never investigated 20 years
ago. On examination, myoclonus is present, he has impaired joint proprioception
and vibration, and his pupils are small and do not react to light from a pen-torch.
What is the most likely diagnosis?
A. Spinal tuberculosis
B. Creutzfeldt-Jakob disease
C. Tertiary syphilis
D. Frontotemporal dementia
E. Bipolar disorder
29. A 19-year-old male is brought into the A&E after falling and hitting his head on
a kerb after a night out. He retained consciousness throughout and can remember
the incident. On examination, he has a minor scalp laceration, his GCS is 15/15
with no focal neurological deficit, he is alert, orientated in time, place and person,
and he does not complain of any neck stiffness. What is the next management
step?
30. During a ward round, the consultant demonstrates the knee jerk reflex to a group
of medical students. What spinal cord level is this reflex testing?
A. C5-C6
B. L1-L2
C. L3-L4
D. L4-L5
E. S1
31. A patient with multiple sclerosis is suffering with spasticity. Her leg is very tight
and has episodes where it ‘goes into spasm’. This is causing her considerable distress
as she has been falling over at work, and she does not want to draw attention to
her diagnosis to her colleagues. What first line medication can be used to reduce
her spasticity?
A. Pyridostigmine
B. Baclofen
C. Diazepam
D. Aspirin
E. Propranolol
A. Disc prolapse
B. Musculoskeletal back pain
C. Cauda equina syndrome
D. Spinal tumour
E. Metastatic spinal cord compression
34. A 32-year-old female with a recent diagnosis of multiple sclerosis (MS) attends a
neurology clinic. She has been doing some reading on the internet and would like
to know which MS ‘clinical pattern’ she has. The neurologist tells her that she has
the most common pattern of MS. What type does she have?
A. Primary-progressive
B. Relapsing-remitting
C. Secondary-progressive
D. Progressive-relapsing
E. Radiologically isolated
35. Samuel is a 23-year-old student who attends an outpatient first seizure clinic. He
was watching TV with his girlfriend when she noticed he was acting strangely,
and she took a video which she shows you. You see him stare blankly at the TV
before smacking his lips repeatedly and chewing in an irregular fashion. Samuel
cannot remember the event, but does recall developing a ‘metallic’ taste in his
mouth 30 minutes beforehand, as well as a strange, funny sensation in his stomach.
What location is the seizure he has had?
A. Frontal lobe
B. Parietal lobe
C. Temporal lobe
D. Occipital lobe
E. Every lobe (generalised seizure)
36. A 26-year-old male with cluster headache attends his GP. Over the past 6 weeks,
his symptoms have worsened and he now wants to explore medical options to
prevent the episodes from occurring. He has no known drug allergies and takes
no medication. What should the GP start him on?
A. Verapamil
B. Propranolol
C. Lithium
D. Lamotrigine
E. Primidone
37. What scoring system would be used in an emergency department to assess the
possibility of a patient having a stroke and to account for the possibility of stroke
mimics?
A. ABCD2
B. FAST
C. NIHSS
D. ROSIER
E. GLASGOW
38. Study the image below. What is the most likely diagnosis?
39. Study the image below of a patient who presented with headaches, fever and left-
sided weakness, and is a known intravenous drug user. What is the most likely
diagnosis?
A. Glioblastoma
B. Meningioma
C. Arteriovenous malformation
D. Cerebral abscess
E. Colloid cyst
40. Doris, an 85-year-old female with a history of heart failure, Type 2 diabetes, and
Lewy Body dementia, is recovering from a dynamic hip screw procedure for a
fractured neck of femur when she becomes confused and agitated during the ward
round the next day. She has been threatening the hospital staff and tried to punch
several nurses, and the consultant thinks she has delirium. What is the most
appropriate initial treatment?
A. Lorazepam
B. L-dopa
C. Haloperidol
D. Chlorpromazine
E. Morphine sulfate
Practice Paper 3
1. Whilst on a neurology special study module as a final year student doctor, you
review a patient with multiple sclerosis in a neurology clinic. She reports that her
symptoms get worse in hot weather, and she has occasionally experienced visual
loss after going on long bike rides with her partner. What is she describing?
A. Lhermitte’s sign
B. Hoover’s sign
C. Uhthoff phenomenon
D. Internuclear ophthalmoplegia
E. Brudzinski’s sign
2. Humayun, a 19-year-old male with epilepsy, starts having a seizure in the waiting
room of a GP surgery before a routine vaccination appointment. You arrive and
secure his airway, his breathing is constant, and he has now been fitting for
7 minutes. Your GP clinic is in a rural area, so an ambulance will take at least half
an hour to arrive. The experienced nurse has attempted intravenous access several
times with no success. A finger pinprick glucose measurement is normal. What is
the next best management step?
A. Acetazolamide
B. Ventriculoperitoneal shunt
C. Endoscopic third ventriculostomy
D. Venoplasty
E. Weight loss
5. A 68-year-old male presents to the A&E with sudden onset dizziness and vertigo,
which happened when he tried to stand up after watching TV 4 hours ago. He
has a history of hypertension controlled on amlodipine, type 2 diabetes, and atrial
fibrillation, for which he takes apixaban. An enthusiastic junior doctor attempted
Hallpike’s and Epley’s manoeuvre, but this did not resolve the symptoms. This
episode has never happened before. He reports no hearing loss, tinnitus, headache,
or weakness, and neurological examination is normal except for severe dizziness,
which prevents him from standing. What is the most likely diagnosis?
6. Fred, a 70-year-old male patient, is in the acute medical ward recovering from a
large stroke yesterday. This morning on the ward round, the nurses say he was
agitated last night and not his normal self, and now he is no longer responsive.
His GCS is 8 on examination, and an emergency CT scan reveals a substantial
area of infarct occupying 80% of the left middle cerebral artery territory with
mass effect. What is the best management plan?
7. After having teaching in the morning and missing the geriatrics ward round, an
excited F1 doctor grabs you and demands that you diagnose their patient, who
had a stroke last week. On examination, the patient’s left eye is fixed, dilated, and
unreactive to light. They have significant ptosis and normal power on the left side.
While the right eye is normal, there is a dense hemiplegia with MRC grade 0/5
power on the right-hand side of the body. What is the correct diagnosis?
8. You are an F2 doctor on your GP rotation. Your next patient is Rachel, a 72-year-
old female, who is accompanied by her husband. He is concerned that for the past
year, she has been complaining of seeing ‘gnomes’ running around the front garden
and shouting at them repeatedly. She also seems to ‘snap in and out of herself ’
once every few days, varying from being her normal self to being very forgetful
and not remembering to walk their dog, which she had always done previously.
On examination, she appears very tired, has a slow gait, and scores 24/30 on the
mini mental state examination. What is the most likely diagnosis?
A. Frontotemporal dementia
B. Alzheimer’s dementia
C. Lewy body dementia
D. Depressive pseudodementia
E. Vascular dementia
9. You are clerking a young patient in the A&E and you are concerned. For the past
week, he has had a fever as well as a seizure before coming to the A&E. His parents
had noticed him acting strangely over the last few days. On examination, he
appears confused, and you note mild neck stiffness. Based on the most likely
diagnosis, what is the next most appropriate management strategy?
10. A 5-year-old boy is brought to the paediatric outpatient clinic by his parents after
seeing him go through ‘weird funny do’s’. These episodes last 10 minutes each,
occur when they have just put him to bed, and consist of him going stiff and
shaking. He cannot remember the episodes but does say that he gets a ‘funny
feeling’, pointing to the right side of his face and tongue after an episode. What
is the most likely diagnosis?
A. Absence seizures
B. Benign Rolandic epilepsy
C. Panayiotopoulos syndrome
D. Infantile spasms (West syndrome)
E. Juvenile myoclonic epilepsy
11. As an F1 on a surgical ward, one of the nurses comes to see you, concerned about
one of her patients. Reginald, an 83-year-old male, had an elective cholecystectomy
3 days ago and was recovering well until last night. He was disorientated, shouting
at staff and relentlessly climbing out of bed. On the round this morning, he was
found asleep but in an awkward position on the bed. His post-op catheter is
draining well, has been in for three days, and the fluid in the bag is slightly cloudy.
What is the most likely diagnosis?
12. A 22-year-old female patient attends her local GP with altered sensation. On
examination, she has normal power, proprioception and vibration of the upper
limbs, but diminished sensation to pain and temperature running up the arms.
She also has muscle wasting of the small muscles of her hand. Last year, she
underwent a foramen magnum decompression for a Chiari malformation. What
is the most likely cause of her altered sensation?
13. You are a medical student sitting in a memory clinic as part of a geriatrics rotation,
but you notice the next patient is only 28 years old. The consultant explains that
he has a past history of severe alcohol misuse and malnutrition, and has now
developed the long-term complication of Korsakoff ’s syndrome. Which of the
following is a characteristic feature of the diagnosis?
14. Arish, a 58-year-old male, presents to his GP with ‘stinging’ leg pain. The pain
starts at the side of his thigh and travels down quickly to his knee, and he also
complains of ‘a funny tingling feeling’ when the pain comes on. His past medical
history includes a TIA aged 48 and poorly controlled type 2 diabetes, and his
latest HbA1c is 88 (normal range ≤48) despite being on insulin. He rarely exercises,
and his body mass index is 35. What is the most likely diagnosis?
15. Chris, a 21-year-old medical student, comes to your GP being very anxious and
concerned about ‘my tremor’. He has been learning about Parkinson’s disease as
part of his introduction to neurology block and is worried that he might have it.
His grandfather had Parkinson’s diagnosed at age 70, but there is no other family
history. Apart from irritable bowel syndrome, he is normally fit and well and takes
no medications. He says thinking and worrying about the tremor makes it worse,
but he does notice an improvement when he goes on nights out for medic’s rugby
social events and has a few drinks. On examination, he has a tremor that worsens
when he tries to touch your finger, which resolves at rest. What is the most likely
diagnosis?
A. Levodopa
B. Amitriptyline
C. Propranolol
D. Dopamine receptor agonists (e.g., ropinirole)
E. Daily low dose alcohol
17. Which of the following may cause lower motor neuron signs on examination?
A. Stroke
B. Brain tumours
C. Multiple sclerosis
D. Bulbar palsy
E. Pseudobulbar palsy
18. As an F2 doctor, you are working in a paediatrics ward and you see a 3-year-old
boy with intractable epilepsy. Extensive investigations have been ordered including
an EEG, but his parents say the cause has not been found. They also mention that
he has been globally behind with his development from a young age. On
examination, you notice a facial rash as well as a discrete scaly rash across his
lower back. What is the most likely cause of his epilepsy?
A. Neurofibromatosis
B. Ohtahara syndrome
C. Sturge-Weber syndrome
D. Intracranial tumour
E. Tuberous sclerosis
19. You have just observed a brain tumour removal in an adult patient in the
neurosurgical theatre. The next day, the consultant reads to you the pathology
report which states, ‘The tumour shows marked brain invasion and crosses the
corpus callosum on imaging, with areas of extreme vascularity and necrosis, and
is the most aggressive primary brain tumour’. What was the tumour type?
A. Meningioma
B. Haemangioblastoma
C. Anaplastic astrocytoma
D. Cavernoma
E. Glioblastoma
20. Which of the following could be a cause of visual loss noted as ‘bitemporal
hemianopia’?
A. Pituitary adenoma
B. Multiple sclerosis
C. Optic nerve glioma
D. Stroke
E. Occipital cortex tumour
21. You are on placement in the intensive care unit, when the consultant intensivist
asks you to diagnose a long-term patient who was admitted after a severe head
injury sustained in a skiing accident. She tells you that the patient still has sleep-
wake cycles, opens their eyes but cannot make any purposeful movements, and
cannot communicate by any means. What is the most likely diagnosis?
A. Coma
B. Brainstem death
C. Persistent vegetative state
D. Locked in syndrome
E. Minimally conscious state
22. You are on a weekend run when you see a large road traffic accident, and you go
over and offer to help. You find a young male patient with severe bleeding from
a scalp wound. He opens his eyes to pain, makes vague groaning noises, and on
applying a pain stimulus, his right arm extends away from the pain site. What is
the most appropriate next management step?
23. A patient is diagnosed with malignant spinal cord compression. What vertebrae
is most likely to be affected?
A. Cervical
B. Thoracic
C. Lumbar
D. Sacral
E. None of the above
24. Amelia, a 43-year-old English teacher, is admitted to the A&E after suffering a
seizure. She had suffered from a ‘horrendous headache’ for 5 minutes immediately
prior to the seizure. A CT head scan shows a large volume of intracerebral
haemorrhage, but no evidence of blood in the subarachnoid space. A magnetic
resonance angiography scan reveals a tortuous collection of blood vessels within
the haemorrhage with a central nidus, and is labelled a ‘Spetzler-Martin grade 2’.
What is the diagnosis?
A. Cavernoma
B. Subarachnoid haemorrhage
C. Arachnoid cyst rupture
D. Arteriovenous malformation
E. Lobar haemorrhage
25. You are a newly qualified doctor on a neurosurgical ward, and on the ward round
you see a patient who suffered a subarachnoid haemorrhage. The registrar
asks you to order bloods to look for a specific complication of the haemorrhage
before running off to theatre without his bleep. What blood test should you
order?
A. Haemoglobin levels
B. Urea and electrolytes
C. ESR and CRP
D. Autoimmune screen
E. Liver function tests
26. You are investigating a patient with a suspected anterior communicating artery
aneurysm. What is the best investigation to assist in making the diagnosis?
A. Carotid doppler
B. MRI brain scan
C. CT angiography
D. CT head scan
E. Cranial ultrasound scan
27. Lisa, a 21-year-old female, attends her GP with a persistent headache for the past
6 months. It is located near the back of her head in the occipital region and is
worse when she coughs or lies flat. She has a history of myelomeningocele (spina
bifida), but is otherwise well. An outpatient MRI scan shows no masses or lesions,
but does demonstrate a displaced lower cerebellum with mild compression of the
foramen magnum. What is the most likely diagnosis?
A. Chiari malformation
B. Brain tumour
C. Idiopathic intracranial hypertension
D. Benign occipital headache
E. Atlantoaxial subluxation
28. Your next patient in the GP surgery is Graham, an 81-year-old male. His wife
strongly believes he has dementia as he has been forgetting to turn the hobs off
the stove or walk the family golden retriever, and he keeps getting lost in familiar
streets. Graham denies this and says his wife is ‘out to get him’. He does not seem
concerned and would rather talk about his recent incontinence issues — he has
had to wear adult nappies for the last year — and says that ‘it can’t be a part of
getting older’. As they leave, you note that he has a slow gait and appears clumsy
on his feet. What is the most likely diagnosis?
A. Alzheimer’s disease
B. Benign prostatic hyperplasia
C. Frontotemporal dementia
D. Mild cognitive impairment
E. Normal pressure hydrocephalus
A. Postural hypotension
B. Gambling disorders and hypersexuality
C. Diarrhoea
D. Taste disturbance
E. Reduced seizure threshold
30. A 45-year-old female attends the emergency department after being sent by her
GP. Over the past few days, her multiple sclerosis symptoms have been worsening,
she has had paraesthesia affecting the left leg, and her vision is blurred. On
examination, her visual acuity is 6/6 in the right eye and 6/60 in the left eye. Apart
from paraesthesia, her motor examination is normal with normal power. What is
the most appropriate management?
A. Acetazolamide
B. Natalizumab
C. Alemtuzumab
D. Pulsed methylprednisolone
E. Glatiramer acetate
31. A 21-year-old male is under investigation for a series of suspicious ‘episodes’. They
last 3–5 minutes and involve his right arm and leg spontaneously twitching. He
can remember the episodes and does not lose consciousness when experiencing
them. What is the diagnosis?
32. A 28-year-old female attends the GP with severe headaches. On taking a detailed
history, she describes them as starting in her early teens, and she found that taking
paracetamol or ibuprofen would ‘take the edge off ’. She has no auras, visual
disturbances, weakness, or numbness. She currently takes paracetamol, ibuprofen,
and amitriptyline every day for these headaches, and has been following this
regime for the last 6 months. She is otherwise fit and well. Given the likely
diagnosis, what is the best management plan for her?
33. As a final year medical student, you and your F1 doctor are called to attend to a
patient who suffered a seizure 5 minutes ago. The seizure has terminated, and they
are now rousable and slowly recovering. The attending consultant asks the F1 to
order a blood test that may help to differentiate between a seizure and a psychogenic
seizure. Which blood test should the F1 request?
A. Lactate
B. Prolactin
C. Mast cell tryptase
D. Ferritin
E. Urea and electrolytes
34. A patient attends a specialist nurse follow up appointment after suffering a stroke
2 weeks ago. Her partner tells the nurse that she presented with left-sided weakness
that affected her leg, with her arm relatively spared, sudden urinary incontinence,
and personality change. Which artery has been affected by the stroke?
35. Whilst on a psychiatry placement, you encounter a patient who has a long-standing
history of alcohol misuse. They presented originally with Wernicke’s encephalopathy,
but Korsakoff ’s syndrome has unfortunately developed as a result. Which vitamin
is deficient to cause both of these conditions?
36. A 62-year-old male patient attends the GP for a review after having a stroke whilst
on holiday in Tenerife 2 weeks ago. The patient was managed conservatively and
his left-sided weakness has since improved. The patient mentions that the doctors
in Tenerife told him to ask his GP to prescribe long-term medication to prevent
further strokes. His inpatient hospital admission form notes that he suffered a
severe allergic reaction to Clopidogrel when he was started on it by the hospital
team. What medication would you commence him on?
A. Clopidogrel 75 mg OD
B. Aspirin 300 mg for 2 weeks, then 75 mg
C. Aspirin 75 mg and modified release dipyridamole
D. Do not commence any medication until an ABCD2 score has been calculated
E. Apixaban 5 mg
37. A patient presents with unique clinical features and signs of finger agnosia,
agraphia, and acalculia, as well as confusion between the right- and left-hand side.
What is the name for this syndrome?
A. Weber syndrome
B. Gerstmann’s syndrome
C. Wallenberg syndrome
D. Parinaud syndrome
E. Claude syndrome
38. A 22-year-old male university student attends the emergency department looking
very unwell. For the past 3 days, he has been generally unwell, has not been eating,
and has a severe headache. His flatmates have noticed that he has been in his room
with the lights off for the past few days. On examination, he is barely rousable,
his neck is held in position, and he grabs his neck in pain when extending a flexed
knee at the hip. What is the most likely diagnosis?
A. Encephalitis
B. Carbon monoxide poisoning
C. Meningitis
D. Guillain-Barré syndrome
E. Subdural haematoma
39. A patient attends the emergency department and receives the CT head scan shown
below:
40. A patient attends the emergency department with reduced consciousness level
and receives the CT head scan shown below:
A. Cerebral contusion
B. Skull fracture
C. Normal scan with contrast
D. Hydrocephalus
E. Subarachnoid haemorrhage
Practice Paper 4
1. A 23-year-old male patient presents to the GP with a new onset headache for the
past two days. It is worse when he stands up from a sitting or lying position, and
he has been feeling quite drained since the headache started. The headache
improves when bending down and lying flat, and he has no photophobia or visual
changes. He is normally fit and well, and he underwent a lumbar puncture 4 days
ago when he was admitted to hospital with a meningitis scare, which turned out
negative. What is the most likely diagnosis?
2. A 43-year-old male is currently a long-term patient in the intensive care unit after
suffering respiratory failure as a complication of Guillain-Barré syndrome. You
are asked to review his neurological function. On examination, you note that he
does not have any voluntary movement, but has sleep wake cycles and can still
communicate using eye blinking movements. What is the diagnosis?
A. Brainstem death
B. Minimally conscious state
C. Persistent vegetative state
D. Coma
E. Locked in syndrome
3. A 53-year-old female smoker attends the GP clinic with back pain. For the past
12 weeks she has had pain mostly on her left-hand side, which travels down her
left leg. She also describes developing a numb feeling on the dorsum of her foot
during this time. There was no trigger and she did not have any recent falls. She
was started on NSAIDs and paracetamol, but this has not helped her symptoms.
She works as a primary school teacher. The GP suspects a diagnosis of disc
herniation. What is the best investigation to diagnose this?
5. You are asked to examine the cranial nerves of a patient in the neurology clinic.
On examination, when asking the patient to look to the left, the right eye moves
but the left eye remains stationary. When asked to move to the right, the left eye
and right eye both move to the right. All other components of the examination
are normal and the pupil size is regular. What is the most likely diagnosis?
7. While in a fifth year OSCE, you are asked to perform a sensory examination of
the lower limb. After you finish the examination, the examiner asks you what
dermatome is represented by the medial side of the leg. What answer should you
give her?
A. L2
B. L3
C. L4
D. L5
E. S1
8. A 63-year-old male attends a TIA rapid access clinic after experiencing a transient
episode of left arm weakness. This episode started whilst watching a football game
and resolved 15 minutes later. He did not have speech disturbances or any other
symptoms. His regular medications are listed as atorvastatin for high cholesterol
and amlodipine for hypertension. He does not have diabetes. What is his ABCD2
score?
A. 1
B. 2
C. 3
D. 4
E. 5
A. Cerebral abscess
B. Glioblastoma
C. Progressive multifocal leukoencephalopathy
D. Wernicke’s encephalopathy
E. Intracerebral haemorrhage
10. A 21-year-old male patient attends the emergency department with a headache.
He has had repeated episodes of cluster headache in the past 3 months, and they
have now gotten worse to the point of being unbearable today. What is the most
appropriate management of his headache?
11. A 32-year-old male who is normally fit and well attends the out-of-hours urgent
care centre with left-sided weakness. Apart from a rough day with the kids yesterday
where he was turning his head around a lot while chasing them, he has no triggers.
On examination, all of his observations are normal including heart rate and blood
pressure. He has considerable right-sided weakness (MRC grade 2) in the upper
and lower limbs. An MRI head scan arranged the next day shows an area of
infarction in the middle cerebral artery territory. What is the most likely cause of
his stroke?
A. Homocystinuria
B. Carotid dissection
C. CADASIL
D. Tertiary syphilis
E. Multiple sclerosis
12. As a fourth year medical student, you are asked to interpret a lumbar puncture
report of Doris, a 68-year-old female, who has presented with a vague history of
fever, headache, and neurological decline.
Colour: Turbid
A. Neisseria meningitidis
B. E. coli
C. Listeria monocytogenes
D. Viral
E. Mycoplasma tuberculosis
A. Lacunar stroke
B. Ramsay Hunt syndrome
C. Motor neurone disease
D. Bell’s palsy
E. Trigeminal neuralgia
14. You are watching a video during a clinical skills session showing different gaits.
The next patient is middle aged and, on walking, appears to swing their affected
legs around and flexes their leg before placing it on the ground. What kind of gait
does this patient have?
A. Trendelenburg gait
B. Circumduction
C. Waddling gait
D. Spastic gait
E. Neuropathic gait
15. A 2-day-old neonate is referred to the paediatric neurosurgery team as the local
women’s hospital had concerns. He is feeding poorly and has been drowsy since
birth with a weak, high pitched cry. The neonate was delivered by emergency
C-section at 28 weeks due to signs of fetal compromise. On examination, you
note an enlarged head circumference, the veins around his scalp appear grossly
dilated, and his eyes remain fixed in a downward position. What is the most likely
diagnosis?
A. Anencephaly
B. Myelomeningocele
C. Encephalocele
D. Cerebral palsy
E. Hydrocephalus
17. You are on the ward as a medical student, and you take a history of a 42-year-old
female who attended the A&E for left-sided weakness. This came on suddenly
over an hour or so, and she also describes a ‘tingling’ feeling on the left-hand side
as well as a dull, persistent headache that she has had for some time. On
examination, she has dense hemiparesis with MRC grade 2/5 power on the left
upper and lower limbs. Sensation, cranial nerves, and speech are all normal.
A CT head scan obtained acutely was negative, and an MRI brain scan taken an
hour ago is normal. What is the most likely diagnosis?
A. Acute stroke
B. Subarachnoid haemorrhage
C. Conversion disorder
D. Multiple sclerosis
E. Hemiplegic migraine
18. A consultant is quizzing you on rare headache types. She describes a headache
that feels like an ‘icepick’ on the skin and has a characteristic response to the
medication indomethacin. What type of headache is she describing?
A. Paroxysmal hemicrania
B. Cluster headache
C. SUNCT
D. Atypical migraine
E. Thunderclap headache
19. You are on the ward when a nurse asks for your help. A 48-year-old male has
presented to the A&E with severe vertigo. He has a known diagnosis of benign
paroxysmal positional vertigo and says this is similar. Which manoeuvre may help
treat this patient?
A. Epley
B. Dix-Hallpike
C. Romberg
D. Smith
E. Jacobsen’s
20. A 2-year-old girl is brought into the GP surgery by her mother. She was delivered
through an emergency C-section following an antepartum haemorrhage and is
globally behind on her milestones. On examination, she has a spastic gait, with
weakness of both legs which have increased tone. Which subtype of cerebral palsy
does the patient have?
A. Spastic diplegic
B. Ataxic
C. Dyskinetic cerebral palsy
D. Quadriplegic cerebral palsy
21. You are asked to examine the pupils of a patient. The patient’s pupils do not react
to light, but when you perform the ‘swinging light’ test, her pupils dilate. Where
is the lesion or cause of her pupil abnormality?
A. Optic chiasm
B. Optic radiation
C. Optic nerve
D. Paramedian pontine reticular formation
E. Occipital cortex
22. A 22-year-old male attends his local GP with his mother. She says he has not been
himself for the last few months, being more forgetful and irritable than usual. He
plays for the university’s rugby team and admits to taking a few hard hits during
the season. What is the most likely diagnosis?
A. No pathology
B. Post-concussion syndrome
C. Early onset dementia
D. Bipolar disorder
E. Somatisation disorder
23. Karen, a 53-year-old female, attends the GP complaining of ‘tingling in her hands’.
She describes numbness in her left hand with a ‘dull’ feeling. There is no associated
weakness, although it has been going on for 6 months and is now disrupting her
work as an NHS typist. She has a past history of hypothyroidism for which she
takes thyroxine, but is otherwise well. On examination, there is no hand muscle
wasting, but flexing her hand for thirty seconds produces the same ‘dull’ feeling.
What is the most likely diagnosis?
24. A patient who has been taking long-term antipsychotics for paranoid schizophrenia
is diagnosed with tardive dyskinesia. Which medication is essential to avoid in
this case?
A. Procyclidine
B. Domperidone
C. Propranolol
D. Ezetimibe
E. Ciprofloxacin
25. You are working as a junior doctor in a medical ward. You are called to see a patient
whose consciousness level has rapidly declined over the last four hours. She
presented 2 days ago with confusion and was found to have a urinary tract
infection, and her sodium was 120 mmol/l. The on-call SHO set up rehydration
with 1L of NaCl 0.9% and her sodium levels increased to 130 mmol/l over the
next 12 hours. Her other observations were normal up until this point. What is
the most likely diagnosis?
A. Sepsis
B. Worsening of UTI
C. Spontaneous subarachnoid haemorrhage
D. Progressive multifocal leukoencephalopathy
E. Central pontine myelinolysis
26. A patient presents to the emergency department with sudden onset facial weakness
after waking up this morning. You also notice a strange, bumpy vesicular rash on
her ear, and she has reported reduced hearing during this time. What is the most
likely diagnosis?
A. Bell’s palsy
B. Ramsay Hunt syndrome
C. Gorlin syndrome
D. Acute stroke
E. Neuromyelitis optica
27. You are the junior doctor in the A&E when a 32-year-old male presents with a
4-hour history of sudden onset back pain, leg weakness, and urinary retention.
The neurological examination reveals MRC grade 4/5 power in both legs, and he
cannot feel a pinprick sensation in the peri-anal area. An urgent MRI confirms
that the cauda equina is being compressed. His performance status is zero. What
is the best management of this patient?
28. A 72-year-old patient with a history of 3 cardiac stents and previous coronary
artery bypass graft operation presents with acute onset vertigo and double vision.
On examination you note loss of facial sensation to pinprick and temperature on
the left side of the face, as well as loss of pain and temperature on the right-hand
side of the body. What artery territory has been affected?
29. A 76-year-old female attends the GP ‘feeling a bit funny’ after she spent the day
fixing a lightbulb with her head up, describing generalised weakness ever since.
On examination you note that she has MRC grade 3/5 power in the arms and 5/5
in both legs, and her speech is completely normal. What is the most likely
diagnosis?
30. A patient is being investigated for suspected myaesthenia gravis. So far, they have
had no investigations besides routine bloods, which have all come back normal.
What would be the best investigation to perform next?
A. Electromyography
B. CT thorax
C. Edrophonium (tensilon) test
D. MRI head scan
E. Lumbar puncture
31. A 24-year-old male presents to the A&E with generalised weakness and respiratory
distress. He is normally fit and well with no past medical history, but he does
recall having an episode of bad diarrhoea two weeks ago. The weakness started
in his upper limbs before moving downwards. On examination, he has an unsteady
gait, MRC grade 2/5 power throughout with reduced tendon reflexes, and limited
eye movement globally. What is the most likely diagnosis?
32. A 63-year-old male with treatment refractory Parkinson’s disease attends the
neurology outpatient clinic and you take a medication history. He describes how
he was diagnosed 15 years ago and was started on levodopa followed by ropinirole
(dopamine agonist), before moving on to other complex treatment regimens as
his treatment progressed. Lately, his main problem has been periods where he is
‘frozen’, but he also tells you that this has reduced after starting a new medication.
The consultant informs you that this is a last resort medication and it is a rapidly
acting dopamine agonist. What medication has he been taking?
33. You are taking a history from a patient with epilepsy, and he describes his usual
seizure pattern as starting with a metallic taste in his mouth and a ‘funny feeling’
in his stomach, following which he cannot remember what he does. His partner
states that he walks around the room seemingly without purpose, moving his
head in different directions, and smacking his lips repeatedly. What type of seizure
does he have?
A. Focal-impaired awareness
B. Focal-aware
C. Generalised tonic-clonic
D. Generalised absence
E. Complex partial
A. Give immediately
B. Wait 24 hours
C. Wait one week
D. Wait two weeks
E. One month
35. You observe a lumbar puncture on a 19-year-old female with suspected meningitis.
The cerebrospinal fluid results are shown below:
Appearance: Turbid
Lymphocytes: Normal
A. Bacterial
B. Viral
C. mycobacterium TB
D. Cryptococcal
E. Listeria
36. A 43-year-old male patient presents with fever, confusion, and headaches. He has
a past history of tuberculosis, oesophageal candidiasis, and cytomegalovirus
infection. Examination reveals that he is confused, and a friend says he has been
like this for the last week. A CT head scan reveals a cystic lesion in the parietal
lobe with prominent intracerebral calcifications. Which of the following is the
most likely diagnosis?
A. Cerebral abscess
B. Toxoplasmosis
C. Progressive multifocal leukoencephalopathy
D. CNS lymphoma
E. Cryptococcal meningitis
A. Encephalocele
B. Meningocele
C. Myelomeningocele
D. Anencephaly
E. Spina bifida occulta
38. A patient presents with weakness on the left side of the face, but strangely has
weakness of the upper and lower limbs on the right side of their body, and their
left-hand side is neurologically intact. Where is the lesion most likely to be located?
A. Cerebrum
B. Brainstem
C. Spinal cord
D. Ventricles
E. Cavernous sinus
39. A 5-year-old boy is brought into the paediatric A&E department after a 4-week
history of progressive headache and vomiting. The attending paediatric registrar
notes papilledema on examination. The emergency CT head scan taken 30 minutes
after admission is shown below:
A. Pneumocephalus
B. Hydrocephalus
C. Malignant MCA syndrome
D. Colloid cyst
E. Frontal space-occupying lesion
40. You take a history from a 74-year-old female, who is currently an inpatient in the
long-term neurological rehabilitation ward. After the history, you assess one of
her scans on a ward computer. A CT head scan obtained 6 months ago is shown
below:
Practice Paper 5
1. A 31-year-old female patient attends the accident and emergency department with
her friends, who believe she is having a stroke as her left-hand side has gone
completely limp. She has a past history of focal seizures, with her last seizure being
16 hours ago. On examination, she has a dense left-sided monoplegia affecting
the lower limb, with MRC grade 0 power. She also appears groggy, but is orientated
in time and person and her speech is normal. What is the most likely diagnosis?
A. Acute stroke
B. Hemiplegic migraine
C. Todd’s paresis
D. Sepsis
E. Post-syncopal paralysis
2. A 65-year-old male, who is being treated conservatively for stage four small cell
lung cancer, presents with a 2-week history of ‘weakness’. On examination, he
has generalised weakness, but this improves remarkably after repetitive testing.
What is the best management option for this patient?
A. 3, 4 dipyridylamine
B. Surgery and chemotherapy for small cell cancer
C. Prednisolone
D. MRI thorax to look for thymoma
E. Pyridostigmine
3. A 52-year-old male presents with an episode of ‘passing out’. Before the episode,
he complained of dizziness and has experienced intermittent palpitations over
the previous 6 weeks, as well as ‘tightness’ of the chest that is worse when walking
up the hill to his house. He had no urinary or bowel incontinence afterwards, and
recovered consciousness quickly after the episode. What is the likely cause of the
patient’s syncope?
A. Vasovagal syncope
B. Carotid sinus syncope
C. Seizure
D. Postural hypotension
E. Cardiac arrythmia
4. A patient in clinic has the classical triad of ptosis, miosis, and facial anhidrosis,
and you diagnose Horner’s syndrome to the delight of your registrar. To your
horror, she asks you a follow up question: Which of the following is not a cause
of Horner’s syndrome?
A. Multiple sclerosis
B. Myaesthenia gravis
C. Lung cancer
D. Carotid dissection
E. Cavernous sinus thrombosis
5. A 62-year-old male has recently been diagnosed with mild dementia and is
commenced on Donepezil after counselling. What characteristic side effect may
he suffer from?
A. Dizziness
B. Headache
C. Diarrhoea
D. Constipation
E. Urinary incontinence
A. Multiple sclerosis
B. Transverse myelitis
C. Neuromyelitis optica
D. Idiopathic intracranial hypertension (IIH)
E. Complex migraine
A. Dysarthria
B. Dysphasia
C. Dysmetria
D. Dysgeusia
E. Dysphonia
A. Polymyositis
B. Myositis
C. Guillain-Barré syndrome
D. Motor neurone disease
E. Progressive supranuclear palsy
A. Doxorubicin
B. Cisplatin
C. Vincristine
D. Docetaxel
E. Bevacizumab
10. You are asked to examine a hospital day case patient with multiple sclerosis who
has ‘unusual’ eye signs before his routine clinic appointment. On examination,
his pupils are normal and react to light, and his fundoscopy and visual fields are
normal. However, when he is asked to look to the left, his right eye does not move,
and you notice nystagmus of his left eye. Where is the most likely location of the
pathology that is causing this condition?
A. Spinal cord
B. Optic nerve
C. Optic tract
D. Optic radiation
E. Medial longitudinal fasciculus
A. Procyclidine
B. Donepezil
C. Galantamine
D. Apomorphine
E. Haloperidol
12. While on a ward round in a small district hospital, you see a 20-year-old female
university student who, for the past two weeks, has been admitted with
‘unexplained clumsiness and left leg weakness’. She is normally fit and well, has
been studying at a prestigious local university, and has her geography final year
exams coming up. The neurology consultant comes to see her. On examination,
she has MRC grade 5 power in both the upper and lower limbs, and her tone,
reflexes, and sensory examination are all normal. When asked to walk, she takes
odd, circumducted gaits with no pattern to them, with some steps appearing
normal. The consultant sits her on a chair, brings her right knee up, and notes
that he can feel her left leg extend/exert force when he does this. What is the sign
that the consultant has elucidated?
A. Babinski’s sign
B. Hoover’s sign
C. Hoffman’s sign
D. Brudzinski’s sign
E. Lhermitte’s sign
13. A patient presents with an 8-month history of hearing loss. They have no other
symptoms and no medical history. An audiogram shows globally reduced hearing
in the left ear only, with an equal reduction in air and bone conduction. What is
the best investigation to look for a possible cause?
14. A 70-year-old female who is normally fit and well presents to the A&E complaining
of sudden onset headache with blurred vision. It is the most painful headache she
has ever had. She has hypertension and anxiety, and she is also on the neurosurgical
waiting list for the removal of an acromegaly-causing pituitary adenoma. A CT
scan shows an area of haemorrhage in the pituitary gland. What is the most likely
diagnosis?
A. Sheehan’s syndrome
B. Pituitary apoplexy
C. Subarachnoid haemorrhage
D. Giant cell arteritis
E. Empty sella syndrome
15. A 43-year-old male is clerked into the A&E with suspected subarachnoid
haemorrhage after suffering from a thunderclap headache whilst playing golf. An
initial CT head scan was negative, so a lumbar puncture was arranged for 12 hours
after symptom onset. What is most likely to be seen on the lumbar puncture if he
does have a subarachnoid haemorrhage?
16. In the intensive care unit, you see a 32-year-old who was involved in a high-speed
road traffic accident 3 days ago and has not been able to be weaned from
ventilation. The critical care consultant explains to you that a provisional diagnosis
of brainstem death is being considered. Which of the following tests can be used
to indicate brainstem death?
17. The clinical triad of pseudo-Argyll Robertson pupils, supranuclear gaze palsy,
and convergence-retraction nystagmus best describes what?
A. Tertiary syphilis
B. Parinaud syndrome
C. Holmes Adie pupil
D. Marcus Gunn pupil
E. Weber syndrome
18. A 43-year-old lady presents with her extremely concerned husband. Over the last
2 months, she has experienced a steady decline in overall functioning. Previously
a lawyer in a high stakes firm, she has been dismissed from her position due to
her forgetting even the simplest of basic laws, and she can no longer attempt the
crosswords that she used to be an expert in. Examination reveals myoclonus and
severely impaired memory on the Montreal cognitive assessment test. What is the
most likely diagnosis?
A. Frontotemporal dementia
B. Normal pressure hydrocephalus
C. Creutzfeldt-Jakob Disease
D. Encephalitis
E. Depressive pseudodementia
19. You are a medical student undertaking your fourth year GP placement when a
patient presents with a headache. In her drug history, you note that she has been
taking diazepam for anxiety for the last 20 years, and the GP is currently trying
to slowly reduce her dose with the aim of eventually stopping it altogether. What
is the action of this class of medication?
A. Glutamate antagonist
B. GABA antagonist
C. ACh receptor agonist
D. GABA agonist
E. Acetylcholinesterase inhibitor
20. A patient presents to you with visual loss after a stroke. On examination, they
have a right inferior quadrantanopia. What part of the brain is most likely affected
to cause this visual loss?
21. Whilst on a paediatrics rotation, you find a rather interesting case of a 9-year-old
female who has presented with 3 distinct episodes of left arm weakness and slurred
speech in the past year. An MRI scan reveals multiple areas of widespread infarcts
across the brain. Apart from this, she is fit and well. She moved to the UK with
her parents from Japan 5 years ago. What is the most likely diagnosis?
A. CADASIL
B. Muscular dystrophy
C. Hurler syndrome
D. Benign Rolandic epilepsy
E. Moya Moya disease
22. A patient presents with a 6-week history of numbness and tingling in their feet.
They have hypertension and a body mass index of 40. On examination, you note
sensory loss in a stocking pattern on the feet, with several small ulcers present on
the dorsum of the foot. The right foot also looks significantly deformed and
concave in appearance. What is the most likely diagnosis?
23. A 14-year-old boy from a travelling family attends the GP with his parents. They
describe him experiencing a gradual decline over the last 6 months, saying ‘he has
just not been himself ’. He used to be at the top of his class, but now he has lost
his ability to write and is displaying personality changes, becoming increasingly
quiet and reserved when he used to be very bright and energetic. He has no allergies,
was not vaccinated as a child, and his medical history is insignificant. His parents
report that he suffered from a ‘really bad bout’ of measles when he was 2 years
old. What is the most likely cause of his decline?
A. Depression
B. CNS lymphoma
C. Creutzfeldt-Jakob disease
D. Subacute Sclerosing panencephalitis
E. Hypothyroidism
24. A patient presents with a 6-week history of double vision only when looking to
the right, accompanied by progressive left-sided weakness. Where is the lesion?
A. Cerebrum
B. Midbrain
C. Pons
D. Medulla
E. Spinal cord
25. A 29-year-old female presents to the A&E after a head injury following a road
traffic accident. As an F1, you and the trauma team undertake an A-E assessment.
You are asked by the consultant to assess her Glasgow Coma Scale (GCS). On
examination, she opens her eyes to a painful stimulus, makes incomprehensible
sounds, and localises to pain. What is the GCS you should tell the A&E consultant?
A. 7
B. 8
C. 9
D. 10
E. 11
26. The parents of a 3-year-old boy bring him to see a local paediatric consultant.
They are struggling with his intractable epilepsy, having tried 4 different
anticonvulsant medications without success. He is also struggling in nursery, and
the teacher has said that he is well behind his peers. On examination, it is clear
that he has global developmental delay, and you notice a roughened patch of skin
over the lumbar area of the back. What is the most likely diagnosis?
A. Tuberous sclerosis
B. West syndrome
C. Global developmental delay
D. Rett syndrome
E. Juvenile myoclonic epilepsy
27. A patient presents to the A&E with an acute episode of migraine. They are
demanding treatment as they say the pain is unbearable. What is the best
management plan?
A. Topiramate
B. Propranolol
C. Sumatriptan
D. Domperidone PO
E. Simple analgesics
28. James, a 44-year-old patient with known HIV and subsequent AIDS, presents
with a 6-month history of general decline. He has developed progressive weakness
affecting his left arm and leg, is very clumsy when he walks, and has double vision.
His last CD4+ count done 2 weeks ago comes back as 30 cells/mm3. An MRI scan
reveals bilateral white matter and thalamic lesions. What is the most likely
diagnosis?
A. Psychological amenorrhoea
B. Menopause
C. Infarction of the pituitary gland
D. Hypothalamic lesion
E. Meningioma
30. In a patient with status epilepticus, what is the first line management?
A. Buccal midazolam 10 mg
B. Intravenous lorazepam 4 mg
C. Rectal diazepam 10 mg
D. Intramuscular glucagon 2 mg
E. Do nothing until directed by a senior
31. Assuming no contraindications, what is the first line treatment (besides simple
analgesics) for the prevention of migraine?
A. Topiramate
B. Verapamil
C. Diltiazem
D. Propranolol
E. Nimodipine
33. What MRC grade of power is assigned to a limb that has ‘active movement against
gravity’ but not against resistance?
A. 1
B. 2
C. 3
D. 4
E. 5
A. Skull fracture
B. Hydrocephalus
C. Acute subdural haematoma
D. Acute extradural haematoma
E. Scalp haematoma
35. A 55-year-old female presents to her GP with a 6-week history of headache and
worsening balance. She has a past medical history of depression and irritable
bowel syndrome, and she underwent a mastectomy 2 years ago for breast cancer.
She consumes 30 units of alcohol a week. What is the most likely diagnosis?
A. Glioblastoma
B. Medulloblastoma
C. Multiple sclerosis
D. Cerebral metastases
E. Cerebellar degeneration due to alcohol abuse
37. A patient presents to the A&E with leg weakness that occurred suddenly without
warning 4 hours ago, affecting both legs. She has also not been able to pass urine
or feel the urge to urinate during this period. She has a history of stage 4 breast
cancer. What is the best immediate management plan?
A. Verapamil
B. Lidocaine
C. Phenytoin
D. Nimodipine
E. Enalapril
39. A 73-year-old male attends the A&E department accompanied by his carer, who
states that he has been more confused as of late and has been falling repeatedly.
A CT head scan is shown below:
40. A patient attends their GP with a 4-week history of headache, which is progressively
getting worse. An outpatient MRI head scan is shown below:
A. Foreign body
B. Cerebral abscess
C. Space-occupying lesion
D. Acute ischaemia
E. Hydrocephalus
ANSWERS
1) Correct answer: D
Pretty much a case of rote learning for this one — the classical triad of raised intracranial
pressure is hypertension, bradycardia, and irregular respiration. The reasons behind this are
complicated and not worth knowing. Ptosis, miosis, and facial anhidrosis are associated
with Horner’s syndrome, and supranuclear gaze palsy, pseudo Argyll Robertson pupils, and
nystagmus are associated with Parinaud syndrome, which is due to a midbrain (brainstem)
lesion such as a pineal gland tumour.
2) Correct answer: D
This would be correct as the diagnosis is a chronic subdural haematoma. This means that blood
will not be acute, which would be in keeping with answer B, as the patient described in answer
B fell 4 hours ago, representing an acute presentation. A refers to a classical acute extradural
haematoma, while C is a subarachnoid haemorrhage which commonly occurs with exertion,
neck stiffness from presence of blood in the cerebrospinal fluid, and subsequent meningeal
irritation. Hence, D would be the answer as chronic subdural haematoma can classically
present with intermittent confusion and should be considered in any elderly patient with a
history of falls and/or who is on anticoagulation. E is describing an acute stroke.
3) Correct answer: E
CT scan head within 1 hour is the most appropriate initial management because the patient,
although seemingly healthy, has features of a basal skull fracture, including bruising behind the
mastoid process (‘Battle’s sign’) and cerebrospinal fluid rhinorrhoea (clear nasal discharge). The
other signs of a basal skull fracture are haemotympanum (blood in the ear) and panda/raccoon
eyes (periorbital bruising). Patients with any of these signs after a head injury need to undergo
a CT scan within 1 hour.
Indications to do a CT within 1 hour after a head injury are worth learning and they
include:
GCS <13
GCS <15 2 hours after the injury
Signs of a basal skull fracture (as above)
Focal neurological deficit
Vomiting more than one time
Seizures after the head injury
4) Correct answer: C
5) Correct answer: D
This lady has recently been given thrombolysis, with the biggest potential risk of doing so
causing a bleed in the brain — also known as a haemorrhagic transformation. This is the most
likely explanation as infarcted tissue is most likely to bleed, and any patient that displays an
altered GCS or reduced consciousness level after thrombolysis requires an immediate CT head
scan to rule out haemorrhagic transformation of the infarct. Malignant MCA syndrome would
show a widespread infarct in the MCA territory during a CT scan (this patient’s scan was
normal), and subarachnoid haemorrhage, hypoglycaemia, and overuse of opiate medication
would not be reasons for thrombolysis or the presentation.
6) Correct answer: D
The correct answer here is a cerebral (brain) abscess, because the patient has presented with
fever, raised intracranial pressure, focal neurological deficits and confusion. The patient also
has had a recent episode of sinusitis, which if bacterial, can rarely spread to the CNS causing a
cerebral abscess. Other risk factors include recent neurosurgical intervention such as surgeries
that can introduce infection, ENT infections (such as sinusitis, mastoiditis, or otitis media),
endocarditis (from fragments or vegetations embolising to the brain), intravenous drug use, or
being immunocompromised. Imaging such as CT or MRI scans (MRI is preferred for making
the diagnosis) can outline ring-enhancing lesions that are often well circumscribed. The ring-
enhancing nature often discriminates this from tumours, which may be less well formed.
Management involves urgent referral to a neurosurgery unit for advice and guidance, with
usual treatment being a prolonged course of broad spectrum intravenous antibiotics. The triad
of fever, headache, and focal neurological signs should make you think of a possible cerebral
abscess in an exam question.
*Source: Eureka! Neurology and Neurosurgery (2016)
7) Correct answer: C
This gentleman has the classical features of neuropathic pain, most likely due to his poorly
controlled diabetes. Amitriptyline is frequently utilised as a first line agent for neuropathic
pain. However, he also has BPH, and giving amitriptyline in this scenario could precipitate
urinary retention due to its antimuscarinic effects and should be avoided here. Pregabalin
and gabapentin are similar agents to the GABA inhibitory receptor and are commonly used
for neuropathic pain and anxiety. Duloxetine is a serotonin and noradrenaline reuptake
inhibitor and is used as an antidepressant and for neuropathic pain (DUALoxetine: blocks two
transmitters, used for 2 conditions). Paracetamol is not commonly used for neuropathic pain
specifically.
A stepwise approach to neuropathic pain (NICE guidelines) are outlined below:
8) Correct answer: C
The Glasgow Coma Scale (GCS) is a quick and frequently utilised assessment of consciousness
level, is very examinable, and should be learned by heart. It is categorised into eye (E), verbal
(V), and motor (M) responses, with all three combined to make a total score out of 15. Eye
responses have a maximum score of 4, verbal a maximum score of 5, and motor a maximum
score of 6 (EVM-456). A key point of the GCS is that you should score the patient’s BEST
response (i.e., if they score higher on one arm compared to the other, the higher score should
be used). The GCS is outlined in the table below. A tricky one to remember is that withdrawal
from the pain stimulus scores 4 points, and groaning (or incomprehensible sounds) scores 2
points.
9) Answer: B
The correct answer is B. Horner’s syndrome is caused by damage to the sympathetic innervation
of the eye (which can happen at 3 levels) and classically gives a triad of ptosis (drooping or falling
of the upper eyelid), miosis (small or constricted pupil), and ipsilateral facial anhidrosis (lack of
sweating on the same side of the face as the lesion). There may also be enophthalmos (posterior
displacement of the eyeball). First-order neurons in the pathway come from the hypothalamus
and brainstem, second-order neurons enter the cervical sympathetic nervous system chain, and
third-order neurons return up the neck to innervate the eye. Causes of Horner’s syndrome
are remembered at the level of the dysfunction where they occur and are outlined in the table
below (with Pancoast tumours [apical lung tumours] being the most commonly asked about
in exams!):
Location Causes
Brain and brainstem Massive cerebral infarction
Lateral medullary syndrome
T1 root Apical lung tumour (Pancoast tumour)
Cervical rib
Sympathetic chain/carotid artery Carotid artery dissection
After thyroid/carotid/laryngeal surgery
Miscellaneous Cluster headache (usually temporary)
Diagnosis is based on clinical signs and treatment is usually related to identifying the cause, such
as treating lung cancer if a Pancoast tumour is the cause.
Ataxia, ophthalmoplegia, and confusion describe the triad for Wernicke’s encephalopathy,
which is caused by acute thiamine (vitamin B1) deficiency, while headache, fever, and altered
consciousness level describe encephalitis.
This lady is presenting with classical symptoms of a subarachnoid haemorrhage, and this
should be urgently ruled out. The most appropriate initial investigation is a non-contrast CT
head scan, which can detect 95% of subarachnoid haemorrhages within 6 hours of symptom
onset. If this is normal, a lumbar puncture should be requested at 12 hours after the headache
onset. The classic sign on lumbar puncture of a subarachnoid haemorrhage is xanthochromia, a
yellow appearance of cerebrospinal fluid due to the presence of bilirubin, a breakdown product
of red blood cells that have got into cerebrospinal fluid due to the bleed. Another (more subtle)
sign of subarachnoid haemorrhage is raised cerebrospinal fluid pressure (see the cerebrospinal
fluid interpretation table on Page 153).
If the cause of a subarachnoid haemorrhage is suspected to be from rupture of an aneurysm,
a CT angiography is often requested to test for this once the patient is stable, which can help
delineate to the blood vessels involved in a possible aneurysm.
This is a classical diagnosis of motor neurone disease, the most common of which is called
amyotrophic lateral sclerosis. This typically presents as an elderly male patient (it is more
common in males) with progressive weakness, swallowing difficulties, and general decline.
On examination, you will typically find a combination of upper and lower motor neuron
signs, such as wasting, fasciculation coupled with brisk reflexes, and clonus (for more upper
and motor neuron signs, see Paper 1, Question 28, Page 10). To diagnose, the patient needs
clinical, electrophysical, or neuropathologic evidence of both left and upper motor neuron
degeneration, as well as evidence of progressive spread within a region or to other body
regions. The condition presents insidiously and is incurable. Riluzole, a drug designed to block
glutamatergic transmission (thought to be involved in the disease process leading to damage
of neurones), only improves survival by a few months and most patients unfortunately die
within 3–5 years, mostly due to complications around ventilation (e.g., respiratory failure) or
dysphagia and subsequent malnutrition/aspiration.
The triad for normal pressure hydrocephalus (NPH) can be remembered as wet, wacky, and
wobbly, or urinary incontinence, dementia/memory impairment, and gait ataxia/falls. This is
an important cause of potentially reversible dementia and needs to be identified quickly. See
Paper 3, Question 28, Page 41 for a description of NPH and its management.
The most common primary cause of brain metastases is bronchial (lung) carcinoma, which
accounts for almost 50% of all brain metastases, followed by breast cancer (15%), melanoma
(10%), and colon cancer (5%). The most common causes of brain metastases are shown below:
Primary site %
Lung 50
Breast 15
Melanoma 10
Colon 5
Other 20
Any patient with a known malignancy and new onset headache or progressive neurological
symptoms or decline needs an urgent CT or MRI head scan to rule out metastasis. Treatment
is with dexamethasone to acutely reduce cerebral oedema (swelling around the tumour) and
referral to neurosurgery services to consider surgical removal if appropriate. If surgery is not
appropriate due to comorbidities or patient choice, radiotherapy or stereotactic radiosurgery —
a more targeted type of radiotherapy — can be considered.
*Source: Davidson’s Principles and Practice of Medicine, 24th Edition (2018)
A lumbar puncture is a useful way to test for multiple conditions such as meningitis, intracranial
hypertension, and subarachnoid haemorrhage. A needle is inserted into the subarachnoid space
with the patient in the left lateral decubitus position (lying down on the left-hand side) and
cerebrospinal fluid is withdrawn and sampled. The most severe (although rare) risk of doing a
lumbar puncture is the risk of coning, where pressure changes during needle insertion cause parts
of the brain to ‘herniate’ down towards the medulla, putting pressure on the brain’s respiratory
centres and causing respiratory arrest. This is life-threatening and should be avoided. The most
common reasons for coning include raised intracranial pressure from mass lesions such as
tumours and other space-occupying lesions, and thus it is important to rule these out before
proceeding.
If in doubt, request a CT head scan BEFORE doing a lumbar puncture to rule out raised
intracranial pressure. The following are absolute contraindications to performing a lumbar puncture,
and for finals you should know at least 2 of them:
*Source: NICE Meningitis (Bacterial) and Meningococcal Septicaemia in Under 16s: Recognition,
Diagnosis and Management (2015)
The correct answer is sodium valproate, which is associated with weight gain and hair loss
(which then grows back curlier!) and is highly teratogenic — it is associated with neural tube
defects, so this medication should be avoided in women of childbearing age. The side effects of
commonly used anticonvulsant medications are outlined below:
Cluster headache is the correct diagnosis, because cluster headache is more common in
males (6:1 male-female ratio), the headache lasts 2 hours (cluster headaches typically last
between 15 minutes and 3 hours) whereas migraines typically last for 4–72 hours, and
the patient has restlessness during the episodes given that he cannot stay still and walks
around — a migraine patient will want to stay still and may have photophobia (lights
appearing very bright causing disturbance) and excessive tearing (autonomic symptoms are
common with cluster headache). Examination and ophthalmoscopy are normal, which rules
out a space-occupying lesion, and there is no medication history, so it is unlikely that he
has medication overuse headache. The key differences you need to know between headache
types for exams is outlined below:
Treatment
Type Duration Triggers Clinical features Treatment (acute) (chronic)
Migraine 4–72 Foods (e.g., Female Any of: First line:
hours chocolate, Photophobia Paracetamol Propranolol or
cheese, wine) Visual disturbance Aspirin topiramate
(scotoma and Metoclopramide/ Second line:
fortification spectre) Domperidone Antidepressants
Phonophobia (fear (if nausea present) (amitriptyline)
of sounds/noises) Triptans Third line/
resistant:
Greater occipital
nerve block
Botulinum toxin
type A
Cluster 15 Alcohol Male (3:1) First line: Verapamil
headache minutes– Restlessness High flow O2 + Lithium
3 hours Autonomic Subcutaneous
symptoms (e.g., red triptans (5HT
eye, lacrimation) agonists, e.g.,
‘Alarm clock’ sumatriptan)
headache — wakes
patient up and
occurs at the same
time each day
Tension 30 Dehydration Bilateral Not usually Simple analgesia
headache minutes– Stress ‘band-like’ pain required (e.g., paracetamol)
1 week Inappropriate around the head Avoid opioids and
prescription Mild to moderate frequent drugs
lenses Non-pulsatile due to medication
No nausea and overuse headaches
vomiting
(Continued )
Treatment
Type Duration Triggers Clinical features Treatment (acute) (chronic)
Medication Variable Withdrawal Headache worse Not treated Gradually
overuse from despite medications acutely withdraw pain
headache medications More medication medications
increases do not
resolve the headache
Long history of
taking medications
Raised Subacute Coughing, Worse in mornings Dexamethasone Urgent 2-week
intracranial (weeks to sneezing, Blurred/double to reduce cerebral referral if
pressure months) bending vision, may have oedema and malignancy/space-
headache down weakness intracranial occupying lesion
pressure suspected
This patient has a classic case of trigeminal neuralgia, thought to be caused by compression of
the trigeminal nerve by a branch of the superior cerebellar artery. Patients present just like this
one — with excruciating facial pain (they may describe this as the worst pain they have ever
had) and extreme sensitivity (the featured exam buzzword is that shaving, brushing teeth, and
even the wind can trigger pain). The pain is sharp, stabbing, and electric shock-like in nature.
It was historically known as ‘suicide disease’ because the severity of pain often drove those
afflicted to end their own life.
The first line treatment for trigeminal neuralgia is carbamazepine, which reduces the
firing of nerve impulses that generates the pain. If carbamazepine is contraindicated or not
effective, a specialist neurology referral is required. Other approaches include using other
drugs such as Gabapentin and Lamotrigine, and refractory cases can be managed with surgical
decompression or balloon compression.
As explained above, the first line treatment for trigeminal neuralgia is carbamazepine.
Propranolol is used for the prevention of migraine, primidone is used as a treatment for
benign essential tremor, sodium valproate is used for generalised seizures, and lithium is used
occasionally as a second line treatment (after verapamil) for cluster headache.
This patient has a suspected acute onset stroke, so an immediate non-contrast CT head
scan should be arranged. This is to rule out a brain bleed or any other contraindications, as
the patient may be eligible for thrombolysis, a clot-busting intravenous treatment that can
improve prognosis if given quickly enough (for thrombolysis criteria see Paper 2, Question 10,
Page 20). However, it is important to first rule out hypoglycaemia as this can potentially cause
symptoms that mimic a stroke. This is especially important in this case as he is currently taking
gliclazide, an oral hypoglycaemic agent, for his diabetes. A capillary blood glucose performed
within seconds can quickly rule this out and therefore would be the first investigation to carry
out in the A&E department as part of an A–E assessment. Other important stroke mimics can
be remembered by the 4 S’s: seizures, sepsis, syncope, and sugar (hypoglycaemia).
This child has Infantile spasms (also known as West syndrome), a childhood epilepsy syndrome.
Infantile spasms are most common among children aged 3–12 months and involve ‘salaam’
attacks — violent flexor spasms of the head, neck, and arms followed by hyperextension and
drawing up of the knees to the chest (this sometimes resemble colic). An electroencephalogram
may reveal the classic exam buzzword of ‘hypsarrhythmia’ — chaotic and irregular high
amplitude waves. It is associated with tuberous sclerosis. Treatment is with vigabatrin or steroids,
and prognosis is poor. The childhood epilepsy syndromes are summarised in the table below:
(Continued )
Diagnosis Key features EEG findings Treatments
Blue breath holding Under 3 years old n/a Conservative
spells Hold breath when upset or
angry = go blue and lose
consciousness
Reflex asystolic 6 months–2 years n/a Conservative
syncope (reflex Triggered by head trauma, pain/
anoxic seizure) cold
Vagal-induced cardiac asystole =
syncope +/– jerks
Febrile convulsions Fever + seizures in any child 6 n/a Antipyretics
months–5 years old (but does not
reduce seizure
risk) — many
recur
Reassurance
This patient has Wernicke’s aphasia, also known as receptive dysphasia. Both aphasia and
dysphasia refer to problems with speaking and are often used interchangeably. The Wernicke’s
area is located in the left temporal lobe and is responsible for understanding and processing what
other people say to us, whilst the Broca’s area is located in the left frontal lobe and is situated
on the left-hand side for about 90% of right-handed people and 70% of left-handed people.
Broca’s is responsible for controlling the muscles of speech that enable us to talk. Therefore, if
Wernicke’s is damaged but the Broca’s area is intact, a patient will be able to talk fluently but
have impaired comprehension. That is, they will not be able to understand dialogue or questions
asked, so speech will be jumbled and not make sense. For Broca’s, patients are able to understand
what they are being asked but cannot articulate themselves, so they are often frustrated at their
inability to do so. The most common cause of both Broca’s and Wernicke’s dysphasia is stroke.
Conduction aphasia refers to damage to the articulate fasciculus, which connects the two areas
together. Damage to this area causes impaired repetition (for some reason!), so a patient with
preserved Broca’s and Wernicke’s areas would not be able to repeat words that are asked of them.
Dysarthria is damage to the muscles involved in articulating speech without higher involvement
(e.g., tongue muscles), and dysphagia is difficulty with swallowing.
*Source: StatPearls: Conduction Aphasia (2020)
Carpal tunnel syndrome is caused by compression of the median nerve, often by the flexor
retinaculum (a band above the carpal tunnel). It is more common in females and patients with
This patient has Charcot-Marie-Tooth disease, also known as hereditary motor and sensory
neuropathy, which is a group of autosomal dominant inherited peripheral neuropathies that
causes demyelination or axon loss, followed by ‘onion skin’ regeneration of nerves. This leads to
the classical syndrome of distal muscle weakness (most usually in the lower limbs like the foot,
tibia, and peroneal groups), leading to an ‘inverted champagne bottle’ appearance, variable loss
of sensation and reflexes, and pes cavus (a deformed, concave-shaped foot) or hammer toes. It
often presents in children or in early adulthood. Diagnosis can be confirmed by family history
and neurophysiology studies showing reduced motor conduction velocity in affected areas.
There are two types of Charcot-Marie-Tooth disease, but it is very unlikely that you will need
to know their differences for medical finals.
Dermatomes are easily examinable (even for medical finals) and easy to forget, so It’s important
to stay on top of them as a clinical medical student. The little finger is supplied by Dermatome
C8. C6 supplies the thumb and the first 1.5 fingers, and the middle finger is supplied by C7.
The key dermatomes for the upper limb are shown in the table below:
The sign being elicited is called Hoffman’s sign and it is exactly as described in this question: you
hold the patient’s hand and flick the nail of the middle finger, and if it is positive, the thumb
and index finger will spontaneously contract. A positive Hoffman’s sign is an indication of an
upper motor neuron or corticospinal cause of pathology, such as cervical cord compression
secondary to degenerative cervical myelopathy (see Paper 2, Question 23, Page 24). Babinski’s
sign is indicated by curling of the big toe upward and not downward when an object is rolled
up the sole of the foot, which is indicative of an upper motor neuron lesion. Hoover’s sign is
when flexing one knee from a flexed position causes the contralateral knee to extend — this is
often used to diagnose functional neurological disorders. Brudzinski’s sign is when you flex the
neck, which causes automatic flexion of the hips and knees and is indicative of meningitis, and
Lhermitte’s sign is pain and paraesthesia — similar to electric shocks moving down the arms
and legs after neck flexion, and is positive in multiple sclerosis. These signs are summarised in
the table below:
This is a classic diagnosis of idiopathic Parkinson’s disease. The patient has almost all
of the hallmark features, the core of which can be remembered using the mnemonic
‘TRAP’: Tremor, Rigidity (like a lead pipe), Akinesia (or bradykinesia — slowness of
movements), and Postural instability. The tremor appears at rest, is better with movement
(in contrast to a benign essential tremor where the tremor appears or is made worse on
movement), and is usually 3–7 Hz (3–7 shakes per second). Other features include loss of
smell (anosmia — very commonly reported as the first symptom!), micrographia (small,
illegible handwriting), and reduced facial expression (described as mask-like facies or, more
correctly, hypomimia). Depression, urinary problems, and dementia are also common
in the condition. All suspected cases of Parkinson’s disease should be referred urgently to
a neurologist specialising in movement disorders BEFORE commencing treatment —
this is to allow confirmation of diagnosis and initiation of specialist treatments (discussed
later in the book). There are no features in this case to indicate Parkinson’s plus syndrome
(Paper 2, Question 1, Page 17).
A positive Babinski’s sign (movement of the big toe upwards when dragging an object from
the heel towards the top of the big toe) is suggestive of an upper motor neuron lesion, which
indicates damage to the brain, higher centres, or central spinal cord, whereas a lower motor
neuron lesion indicates damage to the distal nerves that directly connect to muscles. A
summary of the two signs is shown below — seeing both upper and lower motor neuron
signs on examination almost always indicates motor neurone disease.
Spastic catch: limb ‘catches’ with spasticity suddenly on movement, leading to abrupt
stop.
Clasp knife: rapid, sudden decrease in resistance when trying to flex a joint.
*Source: Macleod’s Clinical Examination, 14th Edition (2018)
The sudden nature of the presentation after a fall suggests an acute pathology. The diagnosis is
an acute subdural haematoma. This is caused by the rupture of bridging veins in the subdural
space in response to trauma. The two main risk factors are increased age and alcohol abuse
(both of which this patient has), leading to brain atrophy, stretching of the cerebral veins,
and hence increased chances of rupture. It is important to know how this will look on a CT
scan (as well as for descriptive purposes in an OSCE scenario): subdural haematomas are
crescent, concave, or banana-shaped and cross suture lines. In contrast, an acute extradural
haematoma will be convex or lens-shaped, will most likely not cross suture lines, and may often
be associated with a parietal or temporal skull fracture. Identifying this difference is crucial and
it is a very frequently asked question in exams.
An acute extradural haematoma is seen often in young patients after head trauma, such
as a cricket ball hitting the side of the head. The patient may briefly lose consciousness before
recovering completely — this is referred to as the ‘lucid interval’ — and then may quickly
deteriorate due to swelling and raised intracranial pressure from the bleed occurring outside
the dural space. The haematoma appears as a convex or lens-shaped hyperdensity (i.e., brighter
or more white than the surrounding brain tissue).
A chronic subdural haematoma tends to present with intermittent confusion, often
following a seemingly trivial head trauma, and the patient may have residual weakness on the
opposite side of the head injury as well. Pneumocephalus simply refers to the presence of air
inside the brain tissue, and more details are not likely required for medical students. It is most
commonly encountered following trauma or surgery.
Basal skull fracture has been described in Paper 1, Question 3, Page 1.
Source: Radiopaedia.org
A rapidly enlarging head circumference over the 95th percentile in any infant requires at least
an urgent (if not emergency) referral to a paediatrician or paediatric neurosurgical team. This
is because the cause may be from hydrocephalus (enlarged ventricles in the brain), which can
be due to serious causes such as brain tumours, brain bleeding (intraventricular haemorrhage
is most common in neonates), and infections like meningitis. This patient has likely developed
hydrocephalus secondary to intraventricular haemorrhage, the risk of which is increased in
premature babies and should be considered the most likely diagnosis until indicated otherwise.
Hydrocephalus leads to increased intracranial pressure, which can manifest with the following
signs in infants specifically: bulging anterior fontanelle (as this has not yet closed in infants),
enlarged head circumference or macrocephaly (microcephaly is a small head circumference and
therefore is not a cause), dilated scalp veins, and ‘sun-setting’, whereby the infant continually
looks down and cannot bring their eyes or eyebrows up. Diagnosis can be confirmed by cranial
ultrasound if the fontanelle is open — this can help with identification of the enlarged ventricles.
Treatment usually involves inserting a ventriculoperitoneal shunt — a device that drains excess
accumulated cerebrospinal fluid from the ventricles into the abdomen — reducing ventricle
size and intracranial pressure as a result.
This patient has somatisation disorder. Ectopic beats are generally a normal finding in the
ECGs of healthy people, so there is no arrythmia. The patient is not seeking out a specific
diagnosis such as cancer, which rules out hypochondrial disorder (hypochondriac = Cancer),
and the patient does not appear to be inventing symptoms for the psychological gain of the
sick role, ruling out Munchausen’s.
Somatisation disorder involves a patient complaining of multiple symptoms affecting
different body systems (this patient has cardiac, dermatological, genitourinary, and gastro
symptoms), often over many years. The key to diagnosis is that the investigation results are
NORMAL. Treatment is with reassurance, although this is often difficult to communicate to
the patient.
Hypochondrial disorder is a fear of having a medical disorder (most commonly cancer)
despite evidence to the contrary. The patient may refuse to accept medical assurance or normal
investigations and demand more extensive tests. The fixation on a single condition or system is
key here. The difference between hypochondrial and somatisation disorder can be remembered
as Somatisation = Symptoms, hypochondriac = Cancer.
Munchausen’s syndrome is a factitious disorder where patients feign physical or
psychological symptoms in order to receive medical attention and benefit from appearing sick.
Source: Psychiatry PRN: Principles, Reality, Next Steps (2020)
This patient has malignant spinal cord compression, most likely secondary to his advanced
prostate cancer. Malignant spinal cord compression occurs in 3–5% of patients with cancer,
with the most common cancer causes being multiple myeloma, followed by prostate, breast,
and lastly lung cancer. 70% occur in the thoracic vertebrae (because there are more of them),
with 20% in the lumbosacral region and 10% being cervical. It is a neurological emergency
due to the potentially reversible risk of severe disability (quadriplegia/paraplegia) and loss of
bowel or bladder function. It has a poor prognosis, with 30% survival at one year.
The typical symptoms are back pain ‘like a band’, particularly around the thoracic
region, and symptoms of cauda equina syndrome (weakness, urinary/bowel incontinence,
sexual dysfunction etc). Immediate management is crucial, and the following steps should
be undertaken: the patient should be placed FLAT (not upright) and given a high-dose
dexamethasone (16mg) to reduce swelling from the compression. Next, the patient should,
depending on their previous functional status and comorbidities, be referred to neurosurgery
for consideration of emergency surgical decompression, but in most cases calling the oncology
team for urgent advice would be the best management after dexamethasone.
*
Source: Oxford Handbook of Palliative Care, 3rd Edition (2019).
Oh Brown-Sequard syndrome — another ‘you’re never going to see it but it’s a med school
favourite’ question. Caused by a spinal cord hemi-section, it can be remembered by the memory
aid ‘One leg weak and one leg numb’ — On the Ipsilateral (same) side of the lesion/injury,
you get impaired sensory transmission of the dorsal columns which supply proprioception
and vibration, and you also get impaired motor transmission of the corticospinal tract, which
causes weakness. On the side opposite the lesion (contralateral), you get impaired sensory
transmission of the spinothalamic tract, which supplies pain and temperature. So putting this
altogether, a lesion on the right hand side would cause:
Weakness on the right-hand side, with impaired proprioception and vibration and:
Loss of pain and temperature on the left-hand side.
The most common cause of Brown-Sequard syndrome Is trauma (typically from a
stabbing/gunshot wound).
That’s probably all you need to know.
Source: Crash Course Neurology, 2019
This patient has a right third cranial nerve palsy and has all the characteristic features: The eye
is deviated in a ‘down and out’ fashion due to weakness of muscles that bring the eye up and
centrally, the pupil is fixed, dilated, and unreactive to light because the parasympathetic part
of the oculomotor nerve is responsible for pupil constriction in response to light, and there is
a droopy eyelid because the oculomotor nerve supplies the levator palpebrae superioris muscle
that elevates the eyelid.
The causes of third cranial nerve palsies are raised intracranial pressure from a space-
occupying lesion such as a tumour, a posterior communicating aneurysm (which would
classically cause a painful third cranial nerve palsy), and diabetes (results in only a partial third
cranial nerve palsy as the pupil response is spared due to the diabetic microangiopathy not
affecting the parasympathetic fibres affecting the pupil).
A fourth cranial nerve palsy often presents with double vision that is classically vertical
(so two images are seen, one on top of the other), and when asked to look down and laterally,
the eye would not be able to do so (in a third cranial nerve palsy, the eye goes down and out
because the trochlear nerve muscles are free to act on their own). The most common cause is
trauma.
An abducens nerve palsy presents with double vision, often occurring side by side, and
when asked to move the eye laterally (looking to the side), the patient will be unable to do
so as this is the function of the abducens nerve. It is commonly affected in raised intracranial
pressure caused by anything, which is often referred to as a ‘false localising sign’ because
although the sixth cranial nerve is affected, the cause of the problem could be anywhere in the
brain and cause raised intracranial pressure.
The trigeminal nerve does not supply eye movement.
This young boy has Wilson’s disease, an autosomal recessive disorder caused by deficiency
of caeruloplasmin, which binds to copper. This leads to excess intracellular copper in the
body, which most severely affects the liver, brain, and basal ganglia. The classical examination
finding is the presence of a golden-brown ring (Keyser-Fleischer ring) during eye examination,
which is pathognomonic. Patients can also present with signs of liver disease and liver failure
if untreated.
Wilson’s disease commonly presents with psychiatric complications in children, such
as behavioural disturbance, memory problems, and even dementia. This patient displays all
of these symptoms. Diagnosis is supported by a low caeruloplasmin level, elevated unbound
serum copper, and high urinary copper excretion, while treatment is with copper-binding
agents that facilitate its excretion (e.g., penicillamine) and is reversible in most cases.
There is no family history or chorea to suggest Huntington’s disease (see Paper 2, Question
4, Page 18), the patient does not have a previous history of syphilis, and there is no mention of
a previous measles infection or antivaccination history to suggest SSPE (Paper 5, Question 23,
Page 70 — a rare but fatal long-term complication of measles infection).
This patient has acromegaly, which is caused by excess growth hormone secretion. Growth
hormones make every tissue in the body grow, and you characteristically get a patient
whose appearance has slowly changed over time, with increases in shoe and finger size
being the classic changes. The patient may also have hypertension due to excess growth
hormones and carpal tunnel syndrome from excessive tissue growth in the compartment.
Patients can also get sleep apnoea from an enlarged tongue and tissues, which reduce
the flow through the airway. Examination may reveal an enlarged tongue (macroglossia)
and prognathism (protrusion of the lower jaw outwards). The most common cause of
acromegaly is pituitary adenoma, which can grow enough to compress the optic chiasm
and result in bitemporal hemianopia (tunnel vision — the outer visual fields are affected).
Most students know this part, but it can also only cause the superior visual field to be
affected. A cause of inferior bitemporal hemianopia is a craniopharyngioma (brain tumour
derived from pituitary gland embryonic tissue), which would present in someone much
younger (usually a child) and with growth disturbance.
The causes of different visual field losses that you need to know for exams are summarised
in this table:
Diffuse axonal injury occurs when shearing forces from trauma cause mechanical damage
to axons, causing widescale damage to the brain itself. It is common in high speed motor
vehicle accidents. The GCS is usually low and the CT scan may paradoxically be normal or
show small punctate brain contusions or nothing dramatic. However, the injury to the brain
is usually diffuse and therefore it is common to have severe neurological deficits from the
injury. Treatment is supportive and prognosis is variable. All of the other injuries (extradural,
subdural, hydrocephalus) would show up on a CT scan and are less likely to be caused by
severe road traffic accidents.
*Source: Davidson’s Principles and Practice of Surgery (neurosurgery chapter)
The scan shows a large area of hypodensity (more black than surrounding brain tissue). This
pattern is somewhat wedge-shaped/organised, and it covers a large territory of the brain.
Causes of hypodensity on scans can either be infarction (dead tissue) or fluid. Infarcts often
appear in such a pattern, and the massive territory affected indicates a large right-sided middle
cerebral artery infarction. This patient is most likely to have suffered a significant stroke, which
accounts for their neurological deficit on examination.
The correct diagnosis here is anterior cord syndrome. This is most common following a flexion
injury, which would be caused by this gentleman flexing his neck from the bricks falling on
him. The injury causes damage to the spinothalamic and corticospinal tracts, with subsequent
loss or difficulty in sensing pain and temperature. The corticospinal tract is characteristically
more affected in the lower areas, and this causes greater loss of power in the legs compared to
the arms.
The opposite is central cord syndrome, which is caused by a hyperextension injury, and
causes weakness of the arms but relative sparing of the LOWER limbs in contrast to anterior
cord syndrome. Syringomyelia causes a classical ‘cape distribution’ sensory loss, with loss of
pain and temperature and relative sparing of vibration and proprioception.
Spinal shock refers to a transient state of autonomic dysfunction that occurs after a severe
spinal cord injury. As a result of damage to the autonomic pathways of the cord, you get
marked hypotension and varying spinal cord dysfunction (flaccid paralysis, urinary retention,
and faecal incontinence). Most patients have a prolonged recovery and need extensive
rehabilitation afterwards.
1) Correct answer: D
Parkinson plus syndromes are often difficult to remember, so using some form of one sentence
summary is really useful here. The correct answer is progressive supranuclear palsy, which is
characterised by upward gaze palsy, having a fixed, ‘surprised’ look, and early falls that often
lead to the patient using a wheelchair despite having normal power to avoid the falls (also
known as ‘wheelchair sign’).
Multiple systems atrophy would fit with early falls, but you would need other forms
of autonomic disturbance like gastroparesis, erectile dysfunction, demonstrated postural
hypotension etc, as these are the predominant symptoms. Corticobasal degeneration can be
remembered as having movement manifestations, such as ‘alien limb’ — where the patient
describes having a limb (usually an arm) that they cannot control, is detached, or feels under
the control of someone else — and myoclonus (electric shock-like jerks of a body part). Drug-
induced Parkinson’s is suggested if any patient is on medications that may reduce dopamine
(the most common ones are antipsychotics which block the dopamine D2 receptor) and
prokinetics like metoclopramide. The tremor is also commonly bilateral. For medical finals,
knowing a simple sentence about the key features of each Parkinson plus syndrome should
suffice.
2) Correct answer: C
Progressive-relapsing
Relapsing-reming
Unpredictable aacks
which may or may not
leave permanent deficits
Secondary-progressive
Inial relapsing-reming
that suddenly declines
without periods of
remission
Primary-progressive
3) Correct answer: A
The only medication known to improve prognosis in patients with motor neurone disease
is riluzole, a glutamate receptor antagonist. This reduces the concentration of glutamate, an
excess of which is thought to contribute towards motor neurone damage in the disease. The
drug only improves survival by a 3–6 months, and this should be communicated to the patient.
Steroids like prednisolone and dexamethasone have no effect on prognosis, tetrabenazine is a
medication used for severe chorea in Huntington’s, and natalizumab is a monoclonal antibody
used in the treatment of refractory multiple sclerosis.
4) Correct answer: C
Mother (unaffected
female)
h h
5) Correct answer: B
This lady has had a psychogenic seizure, which is also called a non-epileptic attack disorder
(NEAD). There are several features in her history that indicate this to be the case. First, the
seizure lasted ten minutes. If this was an epileptic seizure, she would be in status epilepticus, a
medical emergency. She also has her eyes shut, and many patients with NEAD will shut their
eyes and resist an examiner from opening them during an episode. In most seizures, the eyes
are open and the patient classically bites the SIDE of the tongue, but this lady had bitten the
central part of it. Incontinence is variable and can occur in both NEADs and seizures, and
in epileptic seizures the recovery time is often prolonged, with the person appearing ‘groggy’
for anywhere between 5 minutes and an hour afterwards. In NEAD, patients may recover
quickly. Another key point is that seizures will often have the same pattern (such as limb
jerking or tonic-clonic features), whereas NEAD may affect different limbs during different
episodes. The key differences between epilepsy and NEAD are outlined in the table below:
Conversion disorder occurs when a psychological event or strain of some sort is converted
into a physical symptom, such as spontaneous leg weakness. It is most common in young
adult females and often mimics neurological disorders such as a stroke and seizures. A classic
feature is the so-called ‘belle indifference’, where a patient will not be stressed or concerned
by their newfound deficit. Treatment is conservative and the patient should fully recover with
psychological support.
Malingering is the feigning of an injury or problem for external gains, which are often
obvious in the history. A wise clinician once told me, NEVER diagnose malingering! (always
get someone else to do it for you due to the legal implications if you are wrong!!).
6) Correct answer: D
The patient is in status epilepticus, defined as continuous seizures lasting for more than
5 minutes, or repeated seizures between which consciousness is not fully recovered. His previous
brain surgery is the most likely trigger for this. Status epilepticus is a medical emergency as it
can cause brain damage from hypoxia and even death, and should be managed in a stepwise
fashion (shown in the flow chart below). First, stabilise the airway in an A-E approach whilst
calling for help. This includes, crucially, checking blood glucose to rule out hypoglycaemia.
After this, the first treatment to give is intravenous lorazepam 4 mg, which can be repeated
after 10 minutes if the patient is not responding. This is the first line treatment and can be
given without delay in this patient because they have a cannula in situ. You should remember
this drug and the dose as it will help you immensely both in exams and in real life as a doctor.
If this is unsuccessful, the next step is intravenous phenytoin (15–25 mg/kg), and at this
point anaesthetic guidance should be sought. If this is again unsuccessful, you could then give
phenobarbital (an anaesthetic agent) or consider intubation and ITU care.
If you are in the community such as a GP or have no intravenous access available, it is
appropriate to manage status epilepticus with either rectal diazepam 10 mg or buccal (under
the tongue) midazolam 10 mg. These can be repeated after 10 minutes if no response is
observed.
Status epilepticus management pathway.
7) Correct answer: C
This woman has classic migraine and, considering her symptoms, should be treated with
prophylaxis. The first line treatment for prevention of migraine is propranolol. However, this
is contraindicated due to her history of asthma. Second line treatment is topiramate, but this is
also contraindicated as she is currently pregnant and topiramate is teratogenic. This should be
mentioned to any woman of childbearing age who is started on it, and the importance of using
contraception should be emphasised. Codeine is not effective for migraines, so amitriptyline
should be offered. If all these treatments fail, more specialist treatments should be considered,
such as acupuncture, botulinum toxin (botox) injections, and specialist neurology referral.
8) Correct answer: D
This lady most likely has giant cell arteritis (GCA) and should be treated as such until indicated
otherwise. As bloods have already been sent for ESR (an ESR over 50 makes the diagnosis likely),
the appropriate next step is to start her on high-dose steroids like prednisolone (60–80 mg),
which is effective for GCA and can lead to symptom resolution in a few days, along with
a proton pump inhibitor to protect the stomach and possibly a bisphosphonate for bone
protection if she is to be on it long term. Giving 15–20 mg is low dose and would be used for
polymyalgia rheumatica only, and because a complication of untreated GCA is visual damage
and blindness, she should be started on treatment immediately before any referrals are made.
GCA is a large vessel vasculitis often affecting the temporal arteries (it is often called
temporal arteritis) and should be considered in any patient over the age of 50 presenting
with new onset headache. The headache is classically brought on by touching the area near
the temporal arteries, and actions like hair combing can exacerbate it. Patients may have
9) Correct answer: A
In acute ischaemic stroke, depending on the time window that patients present in, they may be
eligible for thrombolysis or thrombectomy. Thrombolysis involves intravenous administration
of a tissue plasminogen activator (tPa), most commonly alteplase, which activates plasminogen
to form plasmin, which then breaks down the clot in the brain that is causing the stroke.
Thrombectomy is an interventional radiology procedure and involves removal of the clot
manually by a catheter inserted into the patient’s groin.
Thrombolysis is indicated within 4.5 hours of symptom onset and should be given
intravenously by an experienced stroke physician and team only. The patient must not
have any contraindications to this treatment, the main ones being uncertain time of onset
(e.g., ‘wake up strokes’, whereby the patient wakes up with symptoms), recent peptic ulcer,
major brain or other recent surgery, and pregnancy. The main risk of thrombolysis is bleeding
and haemorrhagic transformation of the infarct, which can be fatal.
Thrombectomy, which is an interventional radiology procedure whereby a catheter
is inserted into the patient’s groin to manually remove the clot causing the stroke, can be
performed within 6 hours of symptom onset and has less risks than thrombolysis. Availability
is an issue in many centres in the UK, such as being available only 9–5 Monday to Friday in
some centres. In some circumstances, it can be used up to 24 hours of symptom onset, but
these indications are rare and decided by specialists.
The Oxford-Bamford classification is used to classify stroke based on three clinical features:
unilateral weakness and/or sensory disturbance affecting the arms or legs, homonymous
hemianopia, and higher cortical disturbance such as Broca’s or Wernicke’s dysphasia. This
patient has all three, and because the weakness is on the left-hand side, a stroke affecting the
right-hand side of the brain caused it and thus it is called a right total anterior circulatory
stroke (confusing, I know!). If a patient has 2 of the 3 (or just higher cortical dysfunction), it is
called a partial anterior circulatory stroke, and if they have 1 of the 3, it is classed as a lacunar
stroke. Typically examinable but easily learnable, you should definitely know the table below
for med school finals:
This patient has presented with signs of personality change, most markedly loss of inhibition
(becoming more aggressive and rude, gambling), and inability to function. This is generally
referred to as the ‘dysexecutive syndrome’ and occurs in lesions or damage to the frontal lobe of
the brain, where the prefrontal cortex is responsible for higher executive functioning. Parietal
lobe lesions would present with sensation change, temporal lobe lesions would present with
problems with memory, hearing, and occasionally speech if Wernicke’s area is affected, and
there are no visual changes reported in the case which rules out an occipital lesion. The train
track injury is the most likely cause of his symptoms, and the case closely resembles that of
Phineas Gage, a respectful man who became much more aggressive and disinhibited with
marked personality change after a railing pipe impaled his frontal lobe.
Classic Alzheimer’s disease. Gradual decline in memory, reduction in higher functions (like
crossword puzzles), and forgetfulness that is worse at night (this is often seen in dementia)
all point towards a diagnosis of Alzheimer’s. A MOCA score of less than 23 is considered
abnormal, so this patient’s score of 17 suggests significant cognitive impairment. Mild
cognitive impairment would present with a much milder memory decline and not cause much
disruption to functional life like how it is in this case; vascular dementia would be suggested
if the patient had a vascular history like ischaemic heart disease or repeated strokes, and a
classic ‘stepwise progression’ of cognitive deficit would be present. Frontotemporal dementia
would usually present in a relatively younger patient (aged 50s–60s) and marked personality
change would be a prominent feature. In Alzheimer’s, patients often make a good attempt at
responding to questions, whereas someone with depressive pseudodementia may simply reply
‘I don’t know’ to questions asked and be worried about having dementia themselves, as well as
having other symptoms of depression.
Alzheimer’s disease is the most common form of dementia, causes a gradual decline
in memory and higher cognitive functioning, and is due to the build-up of amyloid
and tau protein deposits in the brain, causing atrophy. Risk factors include increased age,
cardiovascular disease, trisomy 21 (so-called because the amyloid precursor protein is located
on chromosome 21), and low educational status. Patients may report getting lost frequently in
places that were previously familiar, ‘leaving the hobs turned on’, and not recognising people
they know. Short-term memory is affected first. There is no cure and the only treatments
available are acetylcholinesterase inhibitors like donepezil, galantamine, and rivastigmine for
mild to moderate dementia (increasing ACh is thought to reduce rate of memory decline) and
memantine (a glutamate receptor antagonist) for moderate to severe dementia.
and the most common causative organism is Campylobacter jejuni. It presents with distal
sensory loss and patchy weakness, with symptoms ascending up the body over a few days, as
well as loss of tendon reflexes (hence why it is called ascending areflexic paralysis). In patients
with cranial nerve or bulbar involvement, the big problem is intercostal and diaphragmatic
weakness causing respiratory failure, and vital capacity must be monitored to decide if the
patient needs ventilation.
Diagnosis is usually clinical, supported by nerve conduction studies that show reduced
nerve conduction speed, and lumbar puncture may show raised cerebrospinal fluid protein and
a normal cell count or mild lymphocytosis.
Patients need careful monitoring of respiratory and cardiac function, and in severe cases
can be treated with intravenous immunoglobulin and occasionally plasmapheresis. Most
patients recover, but roughly 5% die due to complications.
Source: Crash Course Neurology, 5th Edition (2018).
The medical research council’s (MRC) power scale is commonly examined even in final year
OSCEs, so it is worth knowing given the significant chance that it will appear. Any neurological
exam should consist of sensation, motor, cerebellar, and cranial nerve examinations. When
making motor assessments, the MRC grades of power are used and are shown below. This
patient has movement that is possible with gravity eliminated (by putting the arm and legs flat
on the examination couch) and would therefore classify as grade 2.
This patient has a Holmes-Adie pupil, a benign finding of a dilated pupil which does not
react to light but slowly reacts to accommodation. The finding may also be accompanied by
generalised hyporeflexia (again a benign finding) indicating Holmes-Adie syndrome. It does
not require investigation or treatment.
A relative afferent pupillary defect is pupillary dilation that occurs when light is moved
from the contralateral eye to the affected eye (it should normally constrict). It is a sign of
retinal or optic disc disease, and the most common causes are optic neuritis in young people
and retinal disease in the elderly.
An Argyll Robertson pupil is a miotic (small) pupil that does not constrict to light but
constricts to accommodation, and it is associated with tertiary syphilis. Both the absence of
syphilis in the history and the dilated pupil make this diagnosis unlikely in this case.
A third cranial nerve palsy would present with limited eye movements, with the pupil in
a ‘down and out’ pattern, and ptosis which are not present here.
The patient is describing myoclonus. Terms to describe abnormal movements are easily
remembered with one sentence summaries:
Myoclonus is sudden and brief shock-like contractions described as jerking or startling.
Dystonia is involuntary muscle contraction causing twisting or repetitive movements or
abnormal postures.
Tremors are involuntary rhythmic movements of a limb or body part which is shaking,
jerking, or twitching in nature.
Chorea are brief and irregular flowing movements, often described as ‘dance-like’.
Athetosis refers to slow, writhing, non-purposeful, and often flowing movements.
Tardive dyskinesia refers to irregular non-purposeful movements such as lip smacking.
Akathisia refers to motor restlessness.
Hemiballismus involves involuntary unilateral large-amplitude flinging movements.
This young boy has all the cutaneous features of neurofibromatosis type 1 (NF1). NF1 is
an autosomal dominant inherited condition and it is the most common type (1 in 2500
people). Patients are at increased risk of malignancies (most commonly optic nerve gliomas,
neurofibroma, and glioma) as well as haematological cancers like leukaemia and neuroblastoma.
The characteristic skin features are café au lait spots (light brown macules), axillary freckling,
and neurofibromas, all of which this patient has.
Neurofibromatosis type 2 (NF2) is less common (1 in 50,000), the skin manifestations
are not usually present, and patients have the pathognomonic feature of bilateral vestibular
schwannomas (acoustic neuromas), or tumours of the Schwann cells that line the vestibular
nerve. They are also at risk of meningioma and cataracts.
Tuberous sclerosis is explained separately in Paper 3, Question 18, Page 38.
Sturge-Weber syndrome is a congenital neurocutaneous syndrome, with hallmark features
of a facial port wine stain, abnormally fragile blood vessels on the brain surface, and ocular
problems (most commonly glaucoma). This is very rarely asked in finals, so don’t bother
learning it!
The vomiting centre is located in the medulla, a part of the brainstem, and is responsible for
controlling vomiting. It contains histamine (H1) and acetylcholine receptors, and drugs that
act on these to prevent vomiting include cyclizine.
The postrema area, located at the floor of the fourth ventricle in the brain, is where
the chemoreceptor trigger zone is located. This area is responsible for sending signals linked
to vomiting to the vomiting centre and contains serotonin (5HT3) and dopamine (D2)
receptors. Blocking these receptors with drugs like ondansetron (a 5HT3 antagonist) and
prochlorperazine (a D2 antagonist) can control vomiting.
This case describes a textbook acute Extradural haematoma. Caused by rupture of the middle
meningeal artery (MMA) often as a result of trauma over a weakened part of the skull (pterion),
it causes a characteristic ‘lucid interval’ of apparent recovery followed by a drop in consciousness
level. CT head scan will show a convex, lens shaped hyperdensity (whiteness) of acute blood
which generally does not cross suture lines, and there may be an associated skull fracture. Urgent
Neurosurgical referral is advised for guidance and possible transfer and surgery.
Acute subdural haematoma is explained in paper 1, question 29, page 10.
Post-concussion syndrome would present weeks to months after the injury and is
characterised by mild cognitive difficulties, often accompanied by personality change.
Management is conservative and outcomes are favourable.
Malignant MCA syndrome is explained in Paper 3, question 6, page 34.
This mass is most likely to be a Meningioma, an often benign tumour of the meninges that line
the brain. They are frequently discovered in asymptomatic patients (like this one), and appear
as spherical masses often located separately to the brain, do not invade it and may be calcified.
There also may be surrounding hyperostosis (extra bone) due to bone invasion.
Management is by active observation with MRI scans, and most can be surgically removed
without complication if the patient becomes symptomatic (most commonly seizures).
Glioblastoma (GB) is less likely in this patient because it often presents with the patient
being symptomatic, often with a subacute decline in neurological function- there is also
brain invasion, and the tumour may appear irregular, vascular or necrotic on scans (Paper 3,
Question 19, page 38).
Medulloblastoma is less likely given the patients age (it most commonly affects children in
the first decade of life), there is no mention to suggest previous brain surgery ruling out gliosis
(fancy term for brain scarring) and astrocytoma is more common in children, and would be
located in the brain itself.
There is only one real answer here: this is the textbook example of a subarachnoid haemorrhage.
The characteristic features are thunderclap headache (reaches maximum intensity within
5 minutes of onset) or the ‘worst ever headache’, which is often described as being hit over
the head with something (often a bat of some sort), and it is often triggered by exertion (e.g.,
exercise, sexual intercourse). There is also neck stiffness as the blood in the subarachnoid space
can cause meningeal irritation.
The most common cause of a subarachnoid haemorrhage is actually trauma, but for exam
purposes it is also good to be aware that they can be caused by the rupture of saccular (or ‘berry’)
aneurysms, and risk factors include autosomal dominant polycystic disease and hypertension.
All patients will need to be admitted to hospital with regular neurological observations and a
CT head scan (which can identify up to 95% of subarachnoid haemorrhages within 24 hrs).
If this is negative, perform a lumbar puncture at 12 hrs for bilirubin (a yellow appearance that
resembles blood breakdown products indicates a subarachnoid haemorrhage).
An aneurysm may need coiling or clipping if it is found to be the cause, and subarachnoid
haemorrhage carries a high mortality rate (50% die before reaching the hospital). Complications
of subarachnoid haemorrhage are discussed in Paper 3, Question 25, Page 40.
The most likely diagnosis here is degenerative cervical myelopathy (DCM). Motor neurone
disease is less likely as there are only upper motor neuron signs specifically on examination,
while primary lateral sclerosis is just a subtype of motor neurone disease where the corticospinal
tract is primarily affected. He only has symptoms affecting the hands which rules out lumbar
radiculopathy, and his profile does not fit that of the typical multiple sclerosis patient (young
female, eye problems etc).
DCM is a common neurodegenerative condition that often occurs due to spinal cord
compression at the level of the cervical spine, and it is linked to cervical spondylosis. Patients
present with weakness and difficulty using both arms and hands due to compression of the
cervical spinal cord, and symptoms are usually progressive over many months. On examination,
there may be loss of reflexes and flaccid weakness at the level of the lesion, but spasticity
and increased reflexes below it. One of the pathological reflexes may be present: Babinski’s
sign, clonus, or Hoffman’s sign may be positive (flicking the middle finger causes reflex
contractions of the thumb and index finger). Patients with suspected DCM should undergo an
MRI scan of the cervical spine and be referred to neurological or spine surgery services for
consideration of surgical decompression, which may relieve the pressure on the cord and
subsequent symptoms. The disease is progressive and surgery only stops progression, often
without benefit to symptoms.
Myelopathy.org is a good website for further information on the condition!
The correct answer is cauda equina syndrome. A lumbar disc prolapse would not usually
present with urinary and bowel incontinence, and the pain may be neuropathic in nature
(shooting down the leg) as well as having a dermatomal pattern. She has no systemic features
to suggest malignancy, and an osteoporotic fracture would present with less severe weakness
with more prominent back pain and possible loss of height.
The patient has all the key features of cauda equina: back pain, altered perianal sensation
(‘saddle’ anaesthesia), and sphincter dysfunction (urinary and/or bowel retention and
incontinence). She needs an urgent MRI of the lumbosacral spine with potential urgent
neurosurgical referral for emergency decompressive surgery as she is fit and well.
Cauda equina syndrome is a medical emergency and can cause irreversible damage to
sphincter function, leading to long-term bladder and bowel incontinence as well as neurological
deficits. It is most often caused by herniation of a lumbar disc into the cauda equina (nerve
roots that continue after the spinal cord ends), leading to compression and damage to the
lower sacral nerves (S2–S4) and subsequently sensory, motor, and autonomic dysfunction as
described above. It is more common in the elderly, but can affect any age group. The key
features are bowel and/or bladder dysfunction, back pain, bilateral sciatica, leg weakness, and/
or sensory dysfunction, and examination may reveal perianal ‘saddle’ anaesthesia.
Cauda equina is a significant cause of medical negligence claims and should never
be missed. Patients need an urgent MRI of the whole spine to rule this out (usually within
12 hours).
Acute onset bladder and/or bowel dysfunction with perianal ‘saddle’ anaesthesia is cauda
equina syndrome until proven otherwise, and urgent spinal MRI is essential to rule this out in
all patients. Management is with urgent surgical decompression, preferably within 24 hours of
symptom onset (NICE).
This patient has suffered uncal (transtentorial) herniation as a result of progression of his
glioblastoma, producing mass effect and raising the intracranial pressure. The signs specific
to uncal herniation are the dilated pupil on one side (often the ipsilateral side to the tumour)
due to midbrain compression, as well as contralateral hemiparesis due to compression of the
corticospinal tracts. A new onset stroke could be possible given the weakness, but with the
pupil signs is less likely, and the patient has a brain tumour already which points more to
herniation. Subfalcine herniation is usually less serious and the patient may not be gravely
ill, and the breathing pattern is regular. In tonsillar herniation, the breathing centres are
compressed, meaning that they may be irregular or the patient will be close to death. The main
types of herniation are summarised in the table below:
Brain herniation occurs because the brain is effectively closed in a box (the skull). Too
much pressure inside (from mostly tumours, bleeds, and trauma) can cause structures to move
and press on others. The pressure on vital brain structures is what causes herniation and should
be suspected in any patient with a traumatic brain injury. A known malignancy or bleed that
suddenly deteriorates is life threatening and needs urgent senior review.
Cavernous sinus thrombosis can cause compression of all of these structures and
characteristically affects them. The causes are often due to infection (e.g., facial infection,
sinusitis) spreading to the cavernous sinus, malignancy with spread to the sinus, or idiopathic
factors. Classical features are painful ophthalmoplegia (from compression of the cranial nerves
that move the eyes), chemosis (periorbital tissue oedema), and eyeball protrusion (proptosis).
You may also get loss of facial sensation due to trigeminal nerve compression and papilloedema
from ophthalmic vein dilatation. Complications include visual loss and intracranial spread of
the original infection or malignancy.
Management is dependent on the cause: those linked to infections should be managed
with antibiotics, idiopathic causes should be managed with anticoagulants, and suspected
malignancy should be referred to specialists depending on the primary site.
Pituitary apoplexy is an acute haemorrhage into the pituitary gland, usually associated
with a pituitary tumour (Paper 5, Question 14, page 67) with subsequent infarction into
the pituitary gland. It can cause acute cavernous sinus syndrome and requires emergency
neurosurgical review.
This gentleman most likely has a progressive brain tumour given the suspicious mass and
prominent oedema, and dexamethasone is commonly used to manage peritumoral oedema,
which can help to reduce compression of vital neurological structures and minimize associated
symptoms. Mannitol could be suitable as it is an osmotic diuretic used to acutely reduce raised
intracranial pressure, but dexamethasone is a better option in the presence of a known tumour.
There is no indication for morphine sulphate, and acetazolamide is used in acute angle closure
glaucoma to reduce intraocular pressure, and can also be used for the treatment of idiopathic
intracranial hypertension. Nimodipine is a calcium channel blocker that is used to reduce
vasospasm in cases of subarachnoid haemorrhage.
This gentleman unfortunately has the symptoms of tertiary syphilis, and the maculopapular
rash he describes from 20 years ago is likely a manifestation of secondary syphilis. He has
several features of neurosyphilis to boot — Argyll Robertson pupils, signs of tabes dorsalis
(degeneration of the dorsal columns creating impaired proprioception and vibration), stroke
at a young age (most likely another manifestation of tertiary syphilis), and general paresis
(dementia and motor symptoms such as myoclonus). Creutzfeldt-Jakob disease is possible in
this patient, but is less likely given the prolonged history of neurovascular events and syphilis
symptoms proceeding from it (there would be a rapid decline). Bipolar disorder would not
cause such neurological manifestations and this is part of general paresis. Frontotemporal
dementia would also present with less motor manifestations and more prominent personality
change with memory loss.
Tertiary syphilis is a rare manifestation of untreated syphilis infection (only 7% cases of
untreated syphilis) and thus is rare. They classically occur between 10 and 20 years after the
initial infection and can manifest in several ways:
Time after
Symptom primary infection Details
Meningovascular 5–10 years Meningitis stroke (in a young
syphilis person)
Argyll Robertson 10–20 years Pupils do not react to light
pupils but slowly to accommodation,
small pupils
General paresis 15–20 years Dementia, manic behaviour,
paralysis, myoclonus
Tabes dorsalis 15–10 years Lightning pains, loss of joint
proprioception and vibration
(falls, Charcot joint)
Other manifestations include spinal and optic atrophy. Treatment is with benzylpenicillin
injections, but in tertiary syphilis it is often not enough.
This young patient has no concerning features of head injury that would merit a CT scan as
he is alert, fully orientated, does not have a neurological deficit, and has no signs of a basal
skull fracture (see Paper 1, Question 5, Page 2). Therefore, apart from active neurological
observations to make sure that there is no evolving head injury like an expanding bleed or
extradural injury, no CT scan is required. He also has no neck stiffness or concerns of a cervical
spine fracture, so an X-ray of the cervical spine is not indicated.
Spinal cord levels of reflexes are commonly tested in OSCE scenarios at every level of medical
school (even up to finals on occasion), so the basic ones (e.g., dermatomes) should be
memorised:
Baclofen is an antispasticity agent that is used as adjunct therapy in patients with multiple
sclerosis, whereby spasticity causes pain or interruption of daily life. Pyridostigmine is a short-
acting acetylcholinesterase inhibitor that inhibits the enzyme that breaks down acetylcholine,
so it is used for myaesthenia gravis. Diazepam can be used as medication to reduce spasticity,
but not as first line treatment due to its considerable side effect profile (e.g., addiction,
drowsiness). Aspirin is not used for spasticity, and neither is propranolol.
This patient has suffered from a lumbar disc protrusion (also called disc prolapse/herniation).
This is where the intervertebral disc ‘slips’, compressing the nerve roots of the spinal cord and
producing symptoms. There is classically low back pain and sciatica (pain radiation from the
back to the buttock and leg). Onset is typically acute and follows a lifting, bending, or minor
injury, but may also be spontaneous. On examination, straight leg raise may be limited, and
there is loss of reflex such as ankle jerks as well as weakness of plantar flexion or great toe
extension. Sensory loss over the L4-S1 dermatomes may also be present, like in this case. It can
be diagnosed with a spine MRI, which will show the protrusion.
Management of sciatica and lumbar disc protrusion is conservative, as most cases resolve
within 12 weeks with rest and analgesia. Surgery is only indicated when a significant disc
protrusion is persistent, symptomatic, or severe and corticosteroid joint injections have not
been effective.
Source: Kumar and Clark’s Clinical Medicine, 10th Edition (2020).
This patient is suffering from an acute stress reaction. This occurs in response to exceptional
physical and/or psychological stress. While severe, such a reaction usually subsides within days.
The stress is often an overwhelming traumatic experience, such as an accident, physical or
sexual assault, or death. Acute stress reaction often involves feeling ‘dazed’ or numb, with
strange neurological symptoms such as reported loss of vision but no visible defects on
examination. Most cases resolve within a month. No treatments beyond reassurance and
support are normally necessary.
Source: Goldman-Cecil Medicine, 26th Edition (2020).
Relapsing-remitting: the most common type (85% of patients with multiple sclerosis),
involving relapses of unpredictable clinical deterioration followed by periods of remission, with
less recovery as disease goes on.
Primary-progressive: 10–15% of patients with multiple sclerosis, more common in men,
progressive neurological disability from symptom onset, no remissions.
Secondary-progressive: 40–50% of patients with relapsing-remitting multiple sclerosis
develop this after 10–15 years; no more relapses, only progressive neurological symptoms and
decline.
Progressive-relapsing: Least common pattern (5–10%) — progressive disease from onset
with exacerbations.
*Source: Eureka! Neurology and Neurosurgery (2016).
This is a partial seizure (abnormal seizure activity is restricted to one part of the brain) because
of the lack of generalised symptoms like tonic-clonic movements of arms and legs. The
temporal lobe is known to produce ‘odd’ seizure symptoms such as feelings of familiarity (déjà
vu) and non-familiarity (jamais vu), gustatory (taste), olfactory (smell), and complex motor
phenomena, as well as a ‘rising epigastric sensation’. These are often called ‘automatisms’ and
consist of things like lip smacking and chewing.
Any of these symptoms should prompt you to suspect the temporal lobe as the region
affected by the seizure. The frontal lobe is characterised by motor dysfunction, expressive
speech difficulty, and behaviour changes. Parietal lobe seizures have abnormal sensations or
sensory dysfunction and possible motor symptoms due to involvement of the frontal lobe,
while occipital lobe seizures have visual symptoms such as flashing lights.
Verapamil (calcium channel blocker) is the first line treatment for the prevention of cluster
headache. Propranolol is the first line treatment for migraine prevention. Lithium is a second
line agent for prevention of cluster headache and should only be initiated by a specialist.
Topiramate is also used for migraine, and primidone is used for the management of benign
essential tremor.
For cluster headache explanations, see Paper 1, Question 16, Page 6.
The Recognition of Stroke in The Emergency Room score is used to assess the possibility
of stroke in the emergency department and looks at symptoms and signs that more or less
suggestive of stroke. This includes asymmetric facial, arm or leg weakness, speech disturbance,
and visual field defects, with points lost for loss of consciousness or seizure activity. A score
of more than 0 indicates that a stroke is possible and should be treated as such. It is better
than the FAST score (face, arms, speech time) because FAST does not include posterior
circulation stroke symptoms (e.g., vertigo, diplopia, ataxia) or stroke mimics and is often used
by ambulance crews but not emergency doctors.
The National Institute of Health Stroke Scale is a more detailed stroke score used by
specialist stroke physicians to evaluate the extent of a stroke, including motor, speech, language,
vision, and interpretation tests, and is scored out of 42 points.
The ABCD2 score is used in transient ischaemic attack management to assess the risk of
an imminent future stroke and is scored as follows:
Feature Score
Age: >60 yrs 1
Blood pressure
>140/90 or currently undergoing treatment for hypertension 1
Clinical features
Unilateral weakness 2
Speech disturbance 1
Any other symptoms 0
Duration of symptoms
0–10 minutes 0
10–59 minutes 1
>59 minutes 2
Diabetes
Yes 1
No 0
A score of 4 or over is bad and indicates a 7-day stroke risk of 6–11%, and those with a
score of 2 or more should be referred to a stroke specialist for assessment, investigations, and
possible preventative treatment, usually with an antiplatelet such as clopidogrel (now first line,
although ticagrelor is becoming increasingly used), aspirin, or dipyridamole/ticagrelor.
This image displays an acute extradural haematoma. As stated in Paper 1, there is an underlying
temporal bone fracture and a brightness on the scan (hyperdensity) that does not cross suture
lines, and the haematoma is convex or lens-shaped in nature. This suggests the presence of acute
blood and, in the context of the image, an extradural haematoma is the most likely diagnosis,
with a good differential being an acute subdural haematoma. This patient will almost certainly
require urgent neurosurgical management, such as evacuation of the haematoma.
This image shows a cerebral abscess. This is most likely as it has a classical well-defined ‘rim’
shape, coupled with the patient’s history (fever, headaches and weakness is the triad for cerebral
abscess, IVDU also contributes here). A meningioma would not have the rim and would be
towards the edge of the brain (at least for exam purposes!). A glioblastoma would be large,
irregular, and less spherical, there will be prominent oedema, and it will just look horrible on
a scan. Finally, a colloid cyst would be smaller, fluid-filled entirely, and located in between the
ventricles instead of the brain parenchyma itself.
This patient has classic delirium, which is likely linked to her recent surgery. Normally,
the standard medical treatment for acute delirium is haloperidol, a dopamine D2 receptor
antagonist. However, this patient has Lewy body dementia, so giving haloperidol in these
patients can exacerbate Parkinsonian symptoms and cause clinical decline, even leading to
death. NEVER prescribe antipsychotics of any kind in patients with Parkinson’s disease
or Lewy body dementia! A benzodiazepine (preferably a short-acting one) is the next best
treatment here, so lorazepam fits the bill. Chlorpromazine is an antipsychotic that would
worsen symptoms, while L-dopa would not help the delirium and acute agitation.
1) Correct answer: C
This patient is describing the Uhthoff phenomenon — the transient worsening of multiple
sclerosis symptoms after an increase in body temperature — usually after exercise, fever, or
being in a sauna or hot tub. This may manifest as multiple sclerosis symptoms that worsen with
the aforementioned activities, and they are usually transient.
Lhermitte’s sign describes electric shock-like pains in the hands and legs produced by
neck flexion, which is also seen in multiple sclerosis. Hoover’s relates to functional disorders.
Internuclear ophthalmoplegia is a complex eye pattern seen in multiple sclerosis or stroke, in
which the affected eye cannot adduct, along with nystagmus of the contralateral abducting
eye. Brudzinski’s sign describes knee and hip flexion on neck flexion and is seen in meningeal
irritation or meningitis.
2) Correct answer: E
This patient is in status epilepticus as his seizure has now lasted more than 5 minutes, and
an ambulance is at least half an hour away. As it is a medical emergency and waiting for an
ambulance is not plausible, he needs urgent medical treatment. In this situation, it is clear that
he has difficult intravenous access, and trying to gain access would only delay vital treatment.
So although intravenous lorazepam is the first line treatment usually in status epilepticus in a
hospital or healthcare setting, with poor intravenous access you should use buccal midazolam
10 mg or rectal diazepam 10–20 mg. This will mean that the patient can receive rapid treatment
before the ambulance arrives. A glucose reading is normal, so he does not need glucagon, and
there are no signs of anaphylaxis, so he does not need adrenaline. For an explanation of status
epilepticus, see Paper 2, Question 6, Page 19.
3) Correct answer: B
This is a very concerning presentation, and a brain tumour or space-occupying lesion should
be excluded with an urgent MRI or CT scan (within 2 weeks) depending on availability and
her clinical status. This headache has almost all the features of raised intracranial pressure — it
is often worse in the mornings and worse on lying flat, and there may be nausea, vomiting,
and double vision. This can be due to tumour oedema or compression/stimulation of key
structures like the vomiting centre or chemoreceptor trigger zone, which induce vomiting. The
presentation is also progressively worsening which is concerning.
She has no weakness listed which rules out hemiplegic migraine, and she has no vertigo
which rules out benign paroxysmal positional vertigo. Tension headache would present with
‘band-like’ bilateral headache and be less severe, and she lacks the classic features of Horner’s
syndrome (ptosis, miosis, enophthalmos, facial anhidrosis).
4) Correct answer: E
5) Correct answer: A
Sudden onset of dizziness and vertigo in any elderly patient or one with vascular risk factors?
Think posterior circulatory stroke. This rarely presents with the usual stroke features (e.g.,
weakness, speech problems) and actually presents with sudden onset dizziness, vertigo,
syncope, and sometimes diplopia.
This patient needs to be considered for thrombolysis if it is within the time criteria, so an
emergency CT head scan and urgent stroke team referral is the next step.
Benign paroxysmal positional vertigo would be worsened by Dix-Hallpike’s manoeuvre
and would be resolved by Epley’s, which involves holding the head over a bed and rotating it
in different directions.
Vestibular migraine would present with vertigo, dizziness, or balance disturbance with
migraine-like features. The Meniere’s triad of dizziness, vertigo, tinnitus, and sensorineural
hearing loss is not present.
6) Correct answer: D
This patient has malignant middle cerebral artery (MCA) syndrome, a medical emergency that
can cause raised intracranial pressure, brain shift, and subsequent coning (herniation of the
brain with subsequent pressure on the breathing centres, leading to death). Malignant MCA
syndrome occurs in large MCA infarcts where the area of infarct occupies more than 50%
of the MCA territory on scanning. Subsequent cytotoxic oedema from dead tissue can cause
significant brain swelling, which can lead to reduced consciousness and death if not treated. The
treatment is with mannitol to reduce the intracranial pressure followed by urgent neurosurgery
(if the patient is well enough) in the form of a decompressive craniectomy (literally removing
part of the cranium, allowing the brain to expand outwards to relieve the pressure). Prognosis is
poor but some patients do make a full recovery. The stroke team will most likely be managing
the patient and should also be informed.
7) Correct answer: C
Firstly, this doctor is being mean expecting this diagnosis from a medical student. Nevertheless,
third cranial nerve palsy with contralateral hemiplegia or hemiparesis is characteristic of
Weber’s syndrome. It is caused by midbrain infarction (most commonly a stroke), and that’s
all I would expect you to know for finals. Keep it simple.
Claude syndrome is the opposite and can be remembered by the mnemonic ‘Weak Weber,
Clumsy Claude’, with a third cranial nerve palsy accompanied by ataxia.
Gerstmann’s syndrome is finger agnosia, acalculia, agraphia, and confusion between the
right- and left-hand sides, and is caused by a lesion or damage to the angular gyrus in the brain.
Wallenberg syndrome, also known as lateral medullary syndrome, is loss of pain sensation
on one side of the face ipsilateral to the lesion, with loss of pain and temperature on the
other side of the body with minimal to no weakness. You may also get an ipsilateral Horner’s
syndrome, vertigo, ataxia, and nystagmus. It is caused by a stroke infarction of the posterior
inferior cerebellar artery more specifically.
Millard-Gubler syndrome is ipsilateral sixth and seventh cranial nerve palsy with
contralateral hemiplegia.
*Disclaimer*: in-depth knowledge is not required for these topics for finals, so learning a
sentence about them is more than enough.
8) Correct answer: C
9) Correct answer: C
This patient most likely has encephalitis and should be managed urgently with intravenous
acyclovir (antiviral treatment) and senior advice. The history is not convincing for meningitis as
there is no prominent neck stiffness, headache, or photophobia, while seizures, confusion, and
fever lasting over a week is more characteristic of encephalitis. There is no evidence to suggest
that he is septic, so sepsis six is not the best answer, and intramascular (NOT intravenous)
benzylpenicillin should be given if you suspect meningitis to be in the community.
Encephalitis is inflammation of the brain itself and is most commonly caused by viral infection
(especially of the herpes simplex virus type 1 in the UK). It is rarely caused by autoimmune disease
such as NMDA receptor antibody encephalitis. The patient presents with confusion, headache,
altered mental status, and seizures, in contrast to meningitis which more commonly presents with
headache, fever, and neck stiffness. The best investigation for encephalitis is a lumbar puncture,
which may show pleocytosis in viral encephalitis and antibodies in autoimmune disease.
If the diagnosis is clinically obvious, treatment should supersede the lumbar puncture and use
intravenous acyclovir and immunotherapy if autoimmune disease is the cause.
This child is presenting with benign rolandic epilepsy, also called epilepsy with centrotemporal
spikes, which is a type of benign seizure that usually resolves in adolescence and hence does
not usually require treatment. It consists of tonic-clonic seizures during sleep, with abnormal
sensations on the face and the tongue. An EEG may show spike and wave morphology over the
centrotemporal (or rolandic) area of the brain. Treatment is usually conservative as it is often
benign and subsides as the child grows older.
For the different kinds of childhood epilepsy syndromes and seizure types, please see
Paper 1, Question 20, Page 7.
Acute delirium is the most likely diagnosis. Also known as the acute confusional state, it is
defined as acute alteration of consciousness and cognition level with reduced ability to focus,
sustain, or shift attention. Risk factors include increased age (>1/3rd of general admission
patients over the age of 70 have it), recent surgery, drugs, infection (most commonly a urinary
tract infection), or displacement from usual settings (e.g., being in hospital, changing beds and
wards). It develops over a short period and fluctuates during the day. It is a serious diagnosis
that is often missed, with a risk of dying twice higher than average in the sixth month after an
episode.
Management involves treating the source of delirium (e.g., infection) and conservative
measures such as keeping the patient’s room well-lit and furnished with clocks, reminding
them of the date, and being reassuring and supportive. In cases of acute agitation, haloperidol
or another short-term antipsychotic can be considered in the absence of any contraindications.
There is no clinical or medical history of Parkinson’s, so drug withdrawal is less likely and
new onset dementia would present more slowly and not over a few hours. There is also no
history of alcohol misuse or malnutrition, so Wernicke’s encephalopathy is less likely, and the
patient lacks the classical features of Lewy body dementia (Parkinsonism, visual hallucinations,
sleep disturbances, fluctuating cognition).
This lady has syringomyelia, the development of a cystic cerebrospinal fluid-filled cavity (syrinx)
in the spinal cord. This most commonly occurs at the C5–C8 level and includes the classic
symptoms of altered pain and temperature in a ‘cape’ distribution, as well as wasting of the
small muscles of the hands. It is often bilateral. The only association is a Chiari malformation
(Paper 3, Question 27, Page 41), whereby parts of the cerebellum protrude down into the
foramen magnum, causing symptoms. MRI of the spine is the investigation of choice, which
should demonstrate if the syrinx and Chiari malformation is present, and management Is
based on the cause, such as surgical treatment of a Chiari malformation.
The correct answer to this question is anterograde amnesia, which refers to the inability to
retain NEW information. Retrograde amnesia is the inability to recall events BEFORE the
event or condition started, and both occur in Korsakoff ’s psychosis. This is a result of chronic
thiamine deficiency, which is most commonly caused by low dietary intake (e.g., from chronic
alcoholism, severe malnutrition), severe vomiting, and gastric band surgery.
There is destruction of the mamillary bodies in the brain, which causes memory
disturbances. Another unique feature of Korsakoff’s psychosis is confabulation — false
perception and memories on direct questioning — making the patient seem as though they are
‘making things up’. Treatment is with urgent thiamine replacement, but Korsakoff’s psychosis
when present indicates long-term damage and is often irreversible.
Wernicke’s encephalopathy is acute thiamine deficiency and manifests with the triad of
ataxia, ophthalmoplegia, and neurocognitive dysfunction (confusion, stupor). The key idea
with Wernicke’s is that thiamine replacement is very urgent, as it is in theory a reversible
encephalopathy.
This patient has meralgia paraesthesia, a neuropathic pain caused by compression of the lateral
cutaneous nerve of the thigh. It is linked to 2 things, obesity and diabetes, and this patient has
both. It is described as a ‘stinging’ or ‘burning’ pain that runs across the anterolateral surface of
the thigh in line with the nerve’s distribution. Treatment is with adequate control of diabetes
and, if severe, neuropathic pain medication (e.g., amitriptyline, duloxetine).
The other most sensible diagnosis here is diabetic amyotrophy, a severely painful
condition with wasting of the quadriceps muscle, linked to poor control of diabetes. However,
the ‘stinging’ pain with paraesthesia and obesity sways the diagnosis more to meralgia
paraesthesia. Peripheral neuropathy secondary to diabetic amyotrophy would manifest in the
limb extremities such as the feet, and may present with associated sequelae such as a foot
ulcer, altered sensation, or a severely deformed foot due to damage accumulated from absent
sensation (Charcot or concave foot). Trochanteric bursitis is more common in runners and
can cause a similar pain, but given the obesity, poorly controlled diabetic amyotrophy, and
neuropathic pain description, it is most likely meralgia paraesthesia.
The most likely diagnosis is benign essential tremor because the tremor is not resting, he gets
better with alcohol, and his young age presenting with a tremor makes it more likely than
Parkinson’s (although benign essential tremor is most common in people aged 40 and older).
Familial Parkinson’s syndromes do exist, but this is rare and a more significant family history
would be expected to consider this as a potential diagnosis (a grandparent getting diagnosed
age 70 is not uncommon). A rubral tremor (also called a Holmes tremor) is due to a lesion in
the cerebellar peduncle, is a resting and intention tremor with sometimes irregular amplitude,
and is linked to multiple sclerosis.
As the diagnosis suggests, the condition is benign, not linked to any other neurological
conditions, and can subside with time.
The first line management for distressing benign essential tremor is propranolol or primidone.
Although alcohol may help with symptoms, for obvious reasons it is not recommended for
first line treatment!
Stroke, tumours, and multiple sclerosis are all causes of upper motor neuron examination
findings, so the answer is between bulbar and pseudobulbar palsy. Lower motor neuron signs
are seen in bulbar palsy, and the differences are illustrated in the table below. If you have to
remember anything, just know that bulbar palsy causes lower motor neuron signs, in particular
the tongue (flaccid, wasting/fasciculations), while pseudobulbar palsy does the opposite (stiff
and spastic, brisk reflexes). The most common causes of bulbar palsy are motor neurone disease,
poliomyelitis, and Guillain-Barré syndrome. The most common causes of pseudobulbar palsy
are stroke, multiple sclerosis, and motor neurone disease.
Motor neurone disease can cause both bulbar and pseudobulbar palsy, which makes sense
because it affects both upper and lower motor neurons!
Tuberous sclerosis is an autosomal dominant inherited disorder and can be remembered by the
clinical triad of intellectual disability, intractable epilepsy, and facial angiofibromas (adenoma
sebaceum — a benign tumour made up of blood vessels and connective tissue).
Skin features include facial angiofibromas (adenoma sebaceum), depigmented spots on
the torso (ash leaf macules), and rough ‘shagreen’ patches over the lower back.
Apart from intellectual disability and epilepsy, the other features include brain tumours
(most commonly giant cell astrocytoma), cortical and subependymal hamartomas, and cysts
or tumours affecting the kidneys, muscles, and lungs.
Neurofibromatosis type 1 can be remembered as having ‘Lisch’ nodules in the eyes (iris
hamartomas), axillary freckling, café au lait spots, and neurofibromas, as well as optic gliomas
(optic nerve tumour).
Neurofibromatosis type 2 would have the oh-so-classic bilateral vestibular schwannomas,
and they may also present with cataracts and multiple meningiomas.
Ohtahara syndrome, a very rare cause of epilepsy and early death in children, is a
progressive encephalopathy that shows ‘burst suppression’ activity on the EEG (I wouldn’t
study this any further).
Sturge-Weber syndrome will present with a port wine stain, fragile blood vessels, and
ocular disorders such as glaucoma.
Glioblastoma is a grade 4, aggressive glioma (brain tumour of glial cells) that carries a poor
prognosis. It is highly vascular, often has areas of necrosis that will be found on pathology
reports, and may even be evident on imaging. There is often brain invasion, and the tumour
can invade the corpus callosum and adjacent structures. Treatment is with the triad of maximal
surgical resection, adjuvant radiotherapy (after surgery), and temozolomide (a chemotherapy
drug used only for glioblastoma).
Meningioma might be calcified and would more likely be a benign lesion;
haemangioblastoma occurs in young patients and is thus unlikely in an adult although it will
be vascular; an anaplastic astrocytoma may display these features but is a grade 3 glioma (less
aggressive than a glioblastoma); and a cavernoma is a collection of benign blood vessels that
would be evident on imaging and is not technically a brain tumour.
Bitemporal hemianopia describes loss of the outer (temporal) visual fields for each eye, creating
‘tunnel vision’. This is due to lesions of the optic chiasm, or the part where the optic nerves
cross over with each other. Causes of bitemporal hemianopia include pituitary adenomas and
craniopharyngiomas (mainly affecting children).
An optic nerve glioma would cause unilateral blindness or loss of vision, while a stroke
would cause homonymous hemianopia (loss of the lateral half of vision on the side contralateral
to the stroke and loss of the medial half of vision on the same side as the stroke).
An occipital cortex lesion would cause homonymous hemianopia with macular sparing,
which is the same as homonymous hemianopia but with a region of central spared vision.
See Paper 1, Question 36, Page 12 for a full diagram of different patterns of visual loss.
This patient is in a persistent vegetative state, because they still have sleep-wake cycles which
excludes coma or brainstem death, they do not make any voluntary eye movements like
blinking which rules out locked in syndrome, and they do not show brief episodes of higher
consciousness level which would be consistent with a minimally conscious state. Knowing
these key distinguishing features is important, and they are summarised in the table below:
This patient has a GCS of 6 and should therefore be intubated, because when a patient’s
level of consciousness drops to a GCS of 8 or less, they are generally unable to maintain their
own airway. This can be remembered as ‘GCS 8, intubate’. All of the other options are not
as suitable because a needle or surgical cricothyrotomy is only a temporary solution, while a
nasopharyngeal airway is not a definitive airway and may be contraindicated if the patient
has a basal skull fracture, which is entirely possible given the obvious head injury after the
road traffic accident. This should be performed by an experienced anaesthetist or emergency
clinician. This will also help to reduce intracranial pressure and stabilise the patient in the event
that surgery or critical care intervention is needed.
Source: Advanced Trauma Life Support, 9th Edition.
The most commonly affected vertebrae in malignant spinal cord compression (MSCC) is the
thoracic vertebrae (70% overall), simply because there are more of them. The lumbosacral
vertebrae makes up 20% while the cervical vertebrae accounts for 10% (the least common,
but potentially the most serious). Management involves urgent MRI within 24 hours if
MSCC is suspected, with a view to using radiotherapy or surgical decompression depending
on patient age and clinical status. Dexamethasone 16 mg should be given immediately unless
contraindicated.
For a detailed explanation of MSCC, See Paper 1, Question 32, Page 11.
Urea and electrolytes is the correct answer here, specifically sodium levels. This is because one
of the complications of subarachnoid haemorrhage is hyponatraemia, which is due to the
syndrome of inappropriate antidiuretic hormone secretion in response to the event, which
leads to excess water retention with resulting low sodium levels. Therefore, sodium levels
should be monitored in the immediate period after a subarachnoid haemorrhage.
Other complications of subarachnoid haemorrhage include cerebral artery vasospasm
(prevented or treated with a calcium channel blocker called nimodipine), hydrocephalus
(enlargement of the ventricles of the brain due to bleeding into the ventricles), seizures, and
death if severe.
This patient has a Chiari malformation as evidenced by the occipital headache that worsens
when intracranial pressure is increased (e.g., by a headache or lying flat), the risk factor of
spina bifida (patients with this are at increased risk of having a Chiari), and the imaging
showing enlarged cerebellar tonsils compressing the foramen magnum, which is the classical
radiological feature.
A Chiari malformation is a group of posterior fossa brain abnormalities causing herniation
or displacement of the cerebellar tonsils through the foramen magnum (the connection
between the brain and spinal cord), leading to compression and subsequent symptoms. The
most common symptoms are suboccipital headaches and neck pain. Risk factors include
myelomeningocele (spina bifida) and syringomyelia.
Patients often need surgical decompression to relieve pressure on the spine, and this can
be achieved by a foramen magnum decompression operation.
The patient’s imaging does not reveal a brain tumour, she is not obese which makes
idiopathic intracranial hypertension less likely, she does not have a history of rheumatoid
arthritis, and she has headache — not neck issues — which rules out atlantoaxial subluxation.
Dementia, gait ataxia, and urinary incontinence? Think normal pressure hydrocephalus.
This man’s dementia may be treatable, and he needs a referral to a neurologist for further
investigations. See Paper 1, Question 12, Page 4 for a full explanation of normal pressure
hydrocephalus.
A common side effect of dopamine agonist drugs is gambling disorder, impulsivity, and
hypersexuality due to the increase in dopamine levels. Postural hypotension is a characteristic
side effect of levodopa specifically, which is also used to treat Parkinson’s. Diarrhoea is a
side effect of donepezil, an acetylcholinesterase inhibitor used to treat Alzheimer’s disease.
Finally, reduced seizure threshold is a side effect of clozapine, an atypical antipsychotic used in
refractory schizophrenia.
The correct answer is methylprednisolone. This patient is having an acute exacerbation of her
multiple sclerosis, and the treatment of acute exacerbations is steroids with pulsed intravenous
methylprednisolone being used most frequently to acutely reduce the immune response.
Acetazolamide is a carbonic anhydrase inhibitor used in acute glaucoma treatment.
Natalizumab and alemtuzumab are monoclonal antibody treatments used for refractory
multiple sclerosis. Glatiramer acetate is used to reduce the frequency of relapses in relapsing-
remitting multiple sclerosis. Natalizumab has a unique side effect of causing progressive
multifocal leukoencephalopathy, a demyelinating brain disorder linked to long-term
immunosuppression. Some common side effects of neurological drugs are listed here:
This patient has had a focal seizure with preserved awareness. The history is classic of a focal
seizure, which is localised to one part of the brain, so it includes twitching, weakness, and
stiffness, most commonly of one part of the limb or body. In addition, the patient can recall the
events and is therefore ‘aware’ throughout. The classification of seizures has recently changed,
and now complex partial seizure is frequently referred to as a focal seizure with impaired
awareness.
A table summarising the new seizure classification is shown below:
This is a tricky diagnosis, but the most likely one is a medication overuse headache. The features
are not typical of migraine (e.g., no auras or photophobia), and the patient found that starting
medications at a young age helped and has likely continued to use them despite not having
symptoms, leading to her developing a medication overuse headache. To diagnose this, a
patient must have a headache occurring on 15 or more days/month, whilst being on headache
medication for more than 15 days/month, for more than 3 months. Treatment involves slowly
withdrawing all medications to see if doing so has a benefit.
Prolactin can be used if tested within 30 minutes to separate a seizure from a psychogenic
(pseudo) seizure. It is historically raised in seizures and is less likely to be raised in psychogenic
seizures, so this is the test you would order. Lactate is more useful in diagnosing sepsis, mast
cell tryptase is often used to help in the diagnosis of an anaphylactic reaction, and ferritin is
sometimes used as a marker of inflammation (as it is an acute phase protein). Electrolytes
can be a cause of seizures and should therefore be requested in seizure patients, but not to
differentiate an epileptic seizure from a psychogenic seizure.
Anterior cerebral artery strokes present with any of the following: Personality change, urinary
incontinence, and weakness affecting the lower limbs more than the upper limbs.
Middle cerebral artery infarcts cause face and arm weakness that affects the arms more
than the legs, as well as prominent speech disturbance due to the Broca’s and Wernicke’s areas
being affected.
Posterior inferior cerebellar artery infarcts present with lateral medullary (Wallenberg’s)
syndrome, consisting of loss of pain and temperature on the same side of the face as the stroke
and contralateral loss of pain and temperature on the body. The patient may also have Horner’s
syndrome, ataxia, and diplopia.
Posterior cerebral artery infarcts present with predominantly visual disturbances, often
with ‘macular sparing’ (the central part of vision is often preserved) due to the macula having
a dual blood supply.
Lacunar artery strokes present with one of: motor deficit, sensory deficit, and ataxic
hemiparesis.
The vitamin he is deficient in is thiamine (vitamin B1). This vitamin is heavily involved in
brain metabolism and acute or chronic deficiency as seen in excessive alcohol consumption
due to reduced nutritional intake, and excess vomiting and starvation can lead to the
destruction of mamillary bodies involved in memory processing. Acutely, this leads to
Wernicke’s encephalopathy, a clinical triad of confusion, ataxia, and altered conscious level.
This is reversible. If this deficiency persists, it can cause Korsakoff’s syndrome, which leads to
permanent memory loss.
Vitamin B3 (niacin deficiency) causes pellagra, which can be remembered by the ‘Casal
necklace’, an erythematous pigmented rash in the distribution of a collar, and the 3 D’s:
Dementia, Diarrhoea, and Dermatitis.
Vitamin B12 deficiency causes subacute combined degeneration of the cord, where you
get destruction of the white matter tracts leading to sensory deficits, weakness, ataxia, and gait
disturbance, as well as mixed upper motor neurone signs (e.g., extensor plantar response) and
lower motor neurone signs (e.g., absent knee reflexes).
See Paper 4, Question 16, Page 52 for a more detailed explanation of subacute combined
degeneration of the cord.
This patient has had a stroke, and thus needs lifelong medication to prevent further strokes
(secondary prevention). The standard of care for stroke prevention after a stroke or transient
ischaemic attack is now clopidogrel. However, this patient has an allergy to clopidogrel, in
which case modified release dipyridamole and aspirin should be commenced. High-dose
aspirin (300 mg) is typically used for the first two weeks after a stroke before switching to
75 mg, but it is 2 weeks after his stroke and so this should be avoided. Apixaban is a factor Xa
antagonist and direct oral anticoagulant that is used in stroke prevention, but this patient does
not have atrial fibrillation and thus there is no indication to start it.
Gerstmann’s syndrome is a clinical tetrad of finger agnosia, acalculia, agraphia, and confusion
between the left and right sides due to a lesion or damage in the fusiform gyrus in the brain.
I wouldn’t learn anything else about this as it is HIGHLY unlikely to come up in finals
examinations; just one for those who are very interested in neuroanatomy!
Weber syndrome simply describes a third cranial nerve palsy with contralateral hemiplegia,
most commonly caused by a stroke.
Wallenberg (lateral medullary syndrome) is ipsilateral loss of pain and temperature on the
face with contralateral loss of pain and temperature in the upper and lower limbs, often due to
a stroke affecting the posterior inferior cerebellar artery.
Parinaud syndrome is a triad of upgaze palsy, unreactive pupils, and nystagmus, and is
due to midbrain compression from either a tumour affecting the pineal gland (the one that
produces melatonin) or a midbrain stroke.
Claude syndrome is a third cranial nerve palsy accompanied by ataxia.
Out of all these, lateral medullary syndrome is really the only one worth learning for
exams if you ask me!
This scan shows a chronic subdural haematoma with all of the hallmarks — concave-shaped
HYPOdensity (darker than the surrounding brain tissue) that crosses suture lines. Depending
on patient symptoms, neurosurgical review with a view to drain the haematoma should be
considered.
This scan shows a subarachnoid haemorrhage. This is because there is blood within the
subarachnoid space (there is white near the edge of the brain that should normally be black),
which commonly produces a ‘starfish’ pattern on CT scanning.
1) Correct answer: E
The correct diagnosis here is a low-pressure headache. This is caused by reduced circulating
cerebrospinal fluid and is most commonly iatrogenic as a result of a procedure that removes
cerebrospinal fluid, such as a lumbar puncture, which this patient has had. This reduces
cerebrospinal fluid levels, which causes a headache that is characteristically worse when the
patient stands up and is relieved by lying down. This can be thought of as the opposite to a
raised intracranial pressure headache, where the headache would be worse when lying down,
coughing, or straining, and is due to a lesion that causes raised intracranial pressure such as a
brain tumour. Low pressure headache usually improves with time, but if it is disabling, there
is some evidence that caffeine treatment can help (yes you heard it right; caffeine increases
cerebrospinal fluid production apparently!).
Cluster Headache and post-coital headache diagnoses are not likely in this patient.
2) Correct answer: E
This patient has locked in syndrome, which is due to a lesion, infarction, or other damage
to the pons and the midbrain region. As such, respiratory centres that initiate the breathing
process are preserved, the patient will be conscious and have sleep wake cycles, and some eye
movements may be preserved. The key thing to remember is that they have normal sleep wake
cycles and may have preserved eye movements that allow for communication, just that all
other voluntary muscle movement is lost. Prognosis is variable — some recover slowly — and
treatment is mainly supportive.
Brainstem death, coma, persistent vegetative state, and minimally conscious state are
summarised in Paper 3, Question 21, Page 39.
3) Correct answer: B
Disc herniation (prolapse) is caused by part or all of the nucleus pulposus (inner part of an
intervertebral disc) protruding through the annulus fibrosus (outer part of the disc), which
can compress the spinal cord and produce symptoms. It is a common presentation in general
practice and can cause back pain and discomfort. Unlike mechanical back pain, the pain is
usually burning or stinging, and there may be associated numbness or tingling that travels
down the leg on the affected side. It is most common in the age groups of 30 to 50, and
it is diagnosed using an MRI of the spinal area, usually the lumbosacral spine as the most
commonly affected region is the L4/L5 and L5/S1 vertebrae. The other investigations would
not help diagnose it except for a CT scan, which is used only if an MRI is contraindicated (see
investigations guide) or in acute trauma (as it is much quicker than MRI).
Management of disc prolapse depends on the patient and severity of symptoms, but
options include neuropathic pain medications and neurosurgical laminectomy (removal of
excess prolapsed disc). 85% of cases will resolve within 8–12 weeks without any surgical
treatment.
4) Correct answer: C
The correct answer here is a CT head scan within 8 hours because the patient does not satisfy
any criteria for needing one within 1 hour (focal neurological deficit, GCS <13 or <15
2 hours after the injury, no features of a basal skull fracture etc), other than that he is over the
age of 65 and has had a fall from a dangerous height (off a stepladder). These indicate that he
needs a scan within 8 hours, even if he feels well and has no neurological deficits. As with all
head injury patients, he should have regular neuro-observations and assessment of GCS as a
precautionary measure.
See Paper 2, Question 28, Page 26 for a full explanation of CT head scan guidelines for
head injury.
5) Correct answer: D
This patient has a left abducens (sixth cranial) nerve palsy. This is because an abducens nerve
palsy affects the lateral rectus muscle, which moves the eye outwards when looking laterally
or to the side. So, when the patient is asked to look in the direction of the side affected,
the affected eye would remain stationary and be unable to move to that side, whereas the
unaffected eye would be able to move on that side. Causes of an abducens nerve palsy are
variable and include malignancy, trauma, and aneurysms.
A right-sided cranial nerve palsy would present in the same way, but the right eye would
be affected and thus be unable to move to the right.
An oculomotor (third cranial) nerve palsy would present with a dilated pupil that is
unreactive to light, ptosis due to palsy of the levator palpebrae superioris (muscle that lifts the
eyelid up), and a ‘down and out’ pupil facing down and laterally.
A trochlear (fourth cranial) nerve palsy would present with the opposite of a third cranial
nerve palsy, the patient may complain of ‘vertical’ double vision (2 objects seen on top of the
other), and the pupil would be unable to move down and out when asked to, so it would
remain in place while the unaffected eye would be able to move down and out as it supplies the
superior oblique muscle that moves the eye in such a manner.
A diagram summarising this is shown below (note that the right eye is affected by all
pathologies).
6) Correct answer: E
This patient has unilateral sensorineural hearing loss as suggested by the pattern of examination
of Rinne’s and Weber’s tests. When you come across unilateral sensorineural hearing loss, think
vestibular schwannoma or acoustic neuroma until otherwise indicated. This is a tumour of the
Schwann cells lining the vestibulocochlear nerve, which is responsible for hearing and balance,
and thus presents with unilateral hearing loss. An urgent MRI of the cerebellopontine angle
(where the vestibulocochlear nerve arises from) and referral to neurosurgery is advised as these
tumours can be removed, though hearing is often compromised as a result.
Hallpike’s and Epley’s manoeuvre would be used for diagnosing and treating benign
paroxysmal positional vertigo; a CT head scan would be less effective than an MRI at delineating
the tumour; and repeated hearing tests at 2 months would likely show the same pattern.
7) Correct answer: C
Yes, more dermatome questions. They are so commonly asked that it is worth going over them
again and again! The medial side of the leg is supplied by L4, the big toe is L5, and the dorsum
of the foot and the sole of the foot is S1. See the table below for a list of lower limb dermatomes
you need to know:
Dermatome Supplies
L1 Pockets
L2 Between pockets and knee
L3 Knee
L4 Medial side of calf
L5 Big toe
S1 Dorsum of foot and sole
8) Correct answer: E
This gentleman’s ABCD2 score is 5 because being over the age of 60 scores him 1 point, he
is on antihypertensive medication so you can assume he has hypertension which would score
him 1 more point, he had unilateral weakness so gets 2 points, his symptoms lasted 15 minutes
(10-59 min) minutes which gets him 1 point, and he does not have diabetes. A score of 4 or
higher indicates high risk of stroke, so he should be seen by a stroke clinician or team as soon as
possible (within a few days) for evaluation and treatment initiation (usually of an antiplatelet
medication for stroke prevention). The ABCD2 score is used to evaluate the risk of a future
stroke within one month of a transient ischaemic attack and is shown below:
Feature Score
Age: >60 yrs 1
Blood pressure
>140/90 or currently undergoing treatment for hypertension 1
Clinical features
Unilateral weakness 2
Speech disturbance 1
Any other symptoms 0
Duration of symptoms
0–10 min 0
10–59 min 1
>59 min 2
Diabetes
Yes 1
No 0
9) Correct answer: A
This patient has a cerebral abscess as suggested by the classic triad of fever, headache, and
neurological deficit. It is also supported by the fact that the patient is an Intravenous drug
user as suggested by the track marks, which increases the risk, and the CT finding of a well-
circumscribed mass with a ring border, which is classical of a cerebral abscess. Glioblastoma
can also cause such CT findings, but in the context of an acutely unwell patient with fever and
intravenous drug use, cerebral abscess is more likely.
Cerebral abscess is a collection of pus and fluid in the brain. It presents with the
abovementioned classic triad of symptoms and can be caused by local spread of infection,
such as from a paranasal sinus infection (the most common reported cause), otitis media and
mastoiditis, or haematogenous spread from bacterial endocarditis. The most common causes
are Staphylococcus and Streptococcus organisms.
MRI is the imaging method of choice for diagnosing cerebral abscess, which may reveal a
ring-enhancing lesion in the brain parenchyma. Management is with high strength, long-term
antibiotic therapy, which is often required to be given intravenously.
Progressive multifocal leukoencephalopathy is a progressive and fatal disease caused by
the John Cunningham virus, which manifests symptoms in severely immunocompromised
patients. It causes demyelination of the CNS and causes a wide array of presentations from
cognitive impairment, ataxia, hemiparesis, and aphasia. It can be diagnosed by cerebrospinal
fluid PCR, there is no effective treatment, and infection is often fatal.
The treatment of an acute cluster headache is with high flow oxygen and a triptan (e.g.,
sumatriptan). Prevention of cluster headache is with verapamil. For a full breakdown, see
Paper 1, Question 16, Page 6.
This gentleman has unfortunately suffered a carotid dissection. This is likely given the fact that
he now has neck pain and left-sided weakness, seemingly following a mild twisting of his neck
from looking after his children, which is a common presentation. The only other plausible
diagnosis from the list is tertiary syphilis, but this is not likely since there is no history of HIV
or primary syphilis infection or any risk factors (e.g., risky sexual behaviour).
Carotid dissection is caused by a tear in the intima in the wall of the blood vessel, which
can compromise blood flow to certain areas of the brain due to thrombus formation and cause
a stroke. Risk factors are neck trauma such as a road traffic accident (which in some cases is
seemingly trivial; e.g., a chiropractic manipulation!) and connective tissue disorders, such as
Marfan syndrome and Ehlers-Danlos syndrome.
It is unfortunately a common cause of stroke in patients younger than 40.
Investigations are a carotid ultrasound and CT angiogram of vessels which is more
accurate, and management is often with antiplatelets to minimise the risk of stroke or prevent
another if one has already occurred.
Homocystinuria is an autosomal recessive inherited condition with thromboembolic
tendencies, while CADASIL stands for cerebral autosomal dominant arteriopathy with
subcortical infarcts and leukoencephalopathy. That’s all you need to know; it most likely won’t
come up in finals.
The correct answer is listeria monocytogenes. This is suggested by the Gram stain, which reveals
a Gram-positive rod (Neisseria meningitidis is a Gram-negative bacillus and E. coli is a Gram-
negative rod). This makes it less likely to be Neisseria meningitis or E. coli, and the raised
cerebrospinal fluid pressure and presence of neutrophils rule out viral factors. Tuberculosis is
less likely because the glucose is not very low, and it is neutrophils, not lymphocytes, that were
found. For interpretation of cerebrospinal fluid, see Paper 4, Question 35, Page 58.
The correct diagnosis is Bell’s palsy. This is supported by lower motor neuron findings on
examination (e.g., inability to close the eye on the affected side, loss of forehead wrinkles) as
well as the sudden onset without other features of a stroke, such as speech disturbance and
unilateral weakness.
Bell’s palsy is a facial nerve palsy, thought to be caused by an inflammatory response to an
agent (possibly viral infection) with paralysis of the facial nerve, which gives the characteristic
signs. It is a lower motor neuron lesion and will specifically give signs like the inability to close
the eye on the affected side, loss of forehead wrinkles, and hyperacusis (sounds heard much
louder than they actually are).
Treatment is with high-dose corticosteroids as long as there is no contraindication, and
most patients recover within 4–6 months, but for some patients the problem persists long
term.
Ramsay-Hunt syndrome is a variant of Bell’s palsy caused by the varicella zoster virus
that affects the geniculate ganglion specifically, and therefore you get features of Bell’s (the
facial nerve palsy) and features affecting the ear, such as a characteristic painful, erythematous,
blistering (vesicular) rash affecting the ear or the mouth. There may also be tinnitus or
hyperacusis.
The correct answer here is a hemiplegic gait. This is where the limb is held in extension, and
the affected limb swings around (i.e., circumducts) during the swing phase to prevent the feet
from dragging. The most common cause is a stroke or other upper motor neuron lesion. An
explanation of the different kinds of gait and what might be causing them are shown below:
(Continued )
enlarged or ‘bulging’ anterior fontanelles. The most likely cause in this case would be an
intraventricular haemorrhage — a bleed that expands into the ventricles which blocks outflow
of cerebrospinal fluid, causing dilatation of ventricles and hydrocephalus as a result. It is much
more common in babies born pre-term due to fragility of blood vessels in the brain.
Myelomeningocele would present with a visible defect over the back containing parts of
the spinal cord, while encephalocele would present with a large scalp protrusion, often filled
with fluid and usually on the back of the head of the neonate. Although cerebral palsy is in
some cases caused by an intraventricular haemorrhage, it is not the cause of the hydrocephalus.
This lady is presenting with subacute combined degeneration of the cord. This is the result of
long-term vitamin B12 deficiency, most likely linked to her vegan diet, as meat products are
a source of vitamin B12. This causes degeneration of some parts of the spinal cord, namely
the dorsal column and medial lemniscus system and corticospinal tract. You get impaired
proprioception and vibration, motor weakness, and a characteristic reflex pattern of weak or
brisk knee jerks and the opposite for ankle jerks. Treatment is with replacement of vitamin
B12, either through diet change or supplementation. It is usually reversible.
Inherited motor neurone disease is possible but highly unlikely; she is not taking
any medication which rules out drug-induced peripheral neuropathy; tabes dorsalis is a
manifestation of tertiary syphilis which occurs 20 years after a syphilis infection and is unlikely
in a 21 year-old female; and she does not have diabetes which makes diabetic neuropathy a
much less likely possibility, and it would present with ‘glove and stocking’ sensory loss most
prominently in the hands and feet.
Hemiplegic migraine is the correct diagnosis because the person is having a migraine (dull
headache) and has weakness, but detailed MR scanning is normal which effectively rules out a
stroke. A stroke is also less likely given the age of the patient, and stroke is more likely to cause
weakness than tingling. Either way, a patient with weakness and headache should be assumed
to be having a stroke or transient ischaemic attack until proven otherwise, and in this case,
this is effectively ruled out by the normal MRI head scan taken one day after presentation. An
episode of multiple sclerosis is possible but does not explain the headache, while a subarachnoid
haemorrhage could present as a stroke but the headache would be much more severe, and it
would usually be picked up by the acute CT or MRI scan. Conversion disorder is possible but
less likely given the headache.
Hemiplegic migraine is a rare phenomenon of migraine with associated one-sided
weakness that can persist for days after the headache has settled. In 50% of cases, it is inherited
This consultant is asking about paroxysmal hemicrania, a rarer type of headache characterised
by sharp stabbing pain, severe uniorbital pain, and autonomic symptoms such as conjunctival
injection or lacrimation, miosis, and ptosis. It is known for its rapid responsiveness to a
medication called indomethacin, and that’s all you need to know for finals (see table below).
Cluster headache would more likely involve a male patient with alarm clock headaches,
lacrimation, and other autonomic symptoms; SUNCT is characterised by headaches that
happen up to a hundred times a day in some patients, and is responsive to anti-seizure
medications such as lamotrigine, topiramate, and gabapentin; a thunderclap headache is one
that reaches maximum intensity within 5 minutes of starting; and the most serious diagnosis
to exclude is a subarachnoid haemorrhage.
Dix-Hallpike to diagnose, Epley to treat. This patient has benign paroxysmal positional vertigo
(BPPV) and is likely experiencing an acute attack. The Dix-Hallpike test is used for diagnosis,
and this is done by placing the patient in a sitting position on a bed, positioning the head
at 30°, and then taken to 30° below bed level. It is positive if vertigo is experienced after
an interval period, and you should see nystagmus on examination. Nystagmus typically lasts
20–40 seconds and wanes with repeated testing.
Epley’s manoeuvre is used to treat BPPV and is successful in 75% of cases. This involves
lying patients down in a Dix-Hallpike position, turning their head 90° and it holding for
30 seconds, and sitting them upright whilst holding their head sideways.
A guide to BPPV, Meniere’s disease, labyrinthitis, and vestibular neuritis is shown
below:
Vestibular
BPPV Meniere’s disease Labyrinthitis neuritis
Key Sudden vertigo Tinnitus, vertigo, Vertigo, nausea, Vertigo, nausea,
features that lasts seconds, sensorineural and vomiting and vomiting
less severe hearing loss, ear from viral from viral
on repeated fullness infection, hearing infection, NO
movements, loss hearing loss
Dix-Hallpike to
diagnose, Epley
to treat
(Continued )
(Continued )
Vestibular
BPPV Meniere’s disease Labyrinthitis neuritis
Hearing No Yes Yes Yes
loss?
Time Seconds Hours Days/weeks Days/weeks
interval
Cause Crystals Endolymph ?Viral infection ?Viral infection
of calcium abnormality
carbonate
dislodged into
semi-circular
canals
Treatment Epley manoeuvre Low salt diet, Acute attack: Acute attack:
vestibular vestibular vestibular
sedatives sedatives sedatives
(cinnarizine/ (prochlorperazine, (prochlorperazine,
prochlorperazine), cyclizine) cyclizine)
surgery
This patient has spastic diplegic cerebral palsy- this is because the weakness affects the two
limbs specifically. The other subtypes of cerebral palsy are shown in the table below:
Cerebral palsy is defined as a non-progressive disorder of movement, posture, and tone
due to a lesion or damage to the developing brain. Causes can be separated into antenatal,
perinatal, and postnatal.
Treatments are mainly about managing function and improving quality of life, and
includes physiotherapy, orthopaedic referrals, and baclofen to manage spasticity, while surgical
options include selective dorsal rhizotomy — a procedure where the nerves are selectively
removed to reduce spasticity and can improve quality of life if appropriate.
This patient has a relative afferent pupillary defect. This is when the eye dilates to light when
it should not while performing a swinging light test (moving a torch from one eye to the
other). Causes include optic neuritis, glaucoma, and optic nerve lesions such as tumours (most
commonly meningiomas and astrocytoma).
Optic chiasm lesions would cause bitemporal hemianopia (tunnel vision); an occipital
cortex lesion would cause homonymous hemianopia with macular sparing; and a lesion in the
paramedian pontine reticular formation would cause one and a half syndrome, characterised
by a combination of ipsilateral conjugate horizontal gaze palsy in one eye and ipsilateral
internuclear ophthalmoplegia in the other.
This young man has post-concussion syndrome, most likely in part due to head injuries
accumulated throughout his rugby career. Concussion is caused by trauma to the head and
presents with memory loss after a minor or major head trauma. Post-concussion syndrome
occurs after this, and symptoms include persistent memory deficit, irritability, migraine-type
headaches, and inability to concentrate. Treatment is conservative management as 90% of
cases have symptom resolution within 2 weeks, and symptoms may rarely persist for up to
1 year. All other answers are not really appropriate.
This lady has carpal tunnel syndrome caused by a narrowing of the carpal tunnel and pressure
from the flexor retinaculum (a retaining band that runs above it) on the median nerve which
passes through it, causing symptoms. These symptoms include numbness, paraesthesia, and
weakness. The symptoms may be worse at night and are typically improved by waking up and
shaking the affected hand (‘wake and shake’). Risk factors include hypothyroidism, rheumatoid
arthritis, acromegaly, and female sex. Examination often reveals wasting of the thenar
eminence muscles of the thumb (as these are supplied by the nerve), and this is an indication
for decompressive surgery. Two examination tests are associated: Tinel’s test (tapping over the
median nerve centrally just before the wrist) causes numbness and reproduction of symptoms,
and Phalen’s test (flexing the wrist for 30–60 seconds) causes numbness and reproduces the
symptoms. (Tinel’s = Tapping, Phalen’s = Flexing). Treatment is with managing symptoms
(e.g., pain medication), wrist splint which extends the arm at night to prevent symptoms, and
steroid injections (but symptoms recur in 80%). Final treatment is surgical decompression of
the area, often by releasing the flexor retinaculum.
Ulnar nerve palsy would present with claw hand as innervation to the two lumbricals
that extend the fingers is affected. If the lesion is at the elbow causing the ulnar nerve palsy, all
ulnar muscles would lose innervation and thus clawing is absent. As clawing is therefore worse
in distal lesions, it is called the ulnar paradox. You may also get wasting of the hypothenar
eminence and dorsal interossei (small muscles of the hand).
Radial nerve palsy is also called ‘Saturday night palsy’, and it often occurs when a person
compresses the radial nerve overnight (usually by leaning their affected arm over the back of a
chair and sleeping on it). This causes loss of wrist extension and classically causes ‘wrist drop’.
Tardive dyskinesia is defined as involuntary movements that affect the face, tongue, and
mouth, and is linked to long-term typical antipsychotics used historically in the treatment
The correct answer here is central pontine myelinolysis (contemporarily known as osmotic
demyelination syndrome), a potentially life-threatening complication of correcting
hyponatraemia (low sodium) too quickly. If too much fluids with sodium is given to correct
this, it can cause shrinkage of brain endothelial cells, with immune response cells crossing the
blood brain barrier, activating immune cells, and causing demyelination of the pons. Symptoms
include motor abnormalities that progress to flaccid quadriplegia, respiratory paralysis, altered
mental status, and coma. Risk factors include a change in serum sodium greater than 12mEq/L
in 24 hours, alcohol abuse, hypokalaemia, and liver disease.
Progressive multifocal leukoencephalopathy is a rare complication of lymphoma,
leukaemia, but most commonly immunosuppression (usually due to AIDS, organ
transplantation, or patients taking disease-modifying drugs for multiple sclerosis). It is an
infection of oligodendrocytes by the human polyomavirus 2 (John Cunningham virus) which
causes demyelination of the white matter of the cerebral hemispheres. Clinical features are
dementia, hemiparesis, and aphasia which progress rapidly in a patient with risk factors as
above. The only treatment is reversing immunosuppression or treating the cause (e.g., AIDS).
The correct answer is Ramsay Hunt syndrome, a condition that is similar to Bell’s palsy and
is characterised by shingles in the geniculate ganglion, a facial nerve palsy identical to Bell’s,
and a characteristic vesicular rash around the external auditory meatus and/or soft palate,
which differentiates it from regular Bell’s. The reason for this is involvement of the geniculate
ganglion, which supplies part of the ear. There may also be deafness and vertigo or unsteadiness.
Treatment is usually with antiviral medication (acyclovir) and steroids, with complete recovery
being less likely than Bell’s alone.
This patient has cauda equina syndrome, which is a neurological emergency and can result
in permanent neurological deficit (e.g., limb weakness, incontinence) if not treated urgently.
As the patient has presented acutely and has good functional status, urgent neurosurgical
decompression is indicated in the absence of contraindications. See Paper 2, Question 24,
Page 25 for a thorough explanation of cauda equina syndrome.
This is lateral medullary syndrome — loss of pain and temperature on the ipsilateral side of the
face and loss of pain and temperature on the contralateral side of the body. The most common
cause is a stroke, and it can also present with a sudden onset of vertigo, vomiting, and ataxia
(posterior circulatory stroke symptoms). Medial medullary syndrome is remembered as having
weakness unlike the lateral medullary syndrome, as well as paralysis and wasting of tongue
muscles on the ipsilateral side with contralateral hemiplegia, and loss of vibration and joint
position sense. It is caused by occlusion of the lower basilar or vertebral artery.
The correct diagnosis is central cord syndrome, because the lady has presented with a
hyperextension injury (changing a lightbulb by elderly patients is a common exam scenario)
and has examination findings of weakness in the arms but relative sparing of the lower limbs.
See Paper 1, Question 40, Page 15 for an explanation of central cord syndrome.
Edrophonium (tensilon) test is the correct answer, because the patient has myaesthenia gravis
and the way to diagnose this is by the tensilon test. This is where you give the patient a short-
acting acetylcholinesterase inhibitor, which can increase acetylcholine levels temporarily. In a
patient with myaesthenia gravis, there is often a transient improvement in weakness or ptosis.
A patient without myaesthenia gravis would not have much difference. A CT scan of the
thorax could be used to look for a thymoma (thymus tumours are seen in 10% of those with
myaesthenia gravis), but would not reveal the exact diagnosis. The rest of the tests (e.g., MRI
head scan, lumbar puncture) would not be useful for this patient.
This question asks about the treatments for refractory Parkinson’s outside of the traditional
medications (e.g., levodopa, dopamine agonists). The correct answer is apomorphine. This
is not actually related to morphine; instead it is a rapidly acting dopamine agonist used in
treatment-resistant Parkinson’s for ‘on-off’ periods, during which the patient does not have the
effect of the Parkinson’s medications and is frozen or slow (‘off’) versus having the medication
and being hyperkinetic (‘on’).
The other treatments for Parkinson’s that is resistant to medication include COMT
inhibitors (stops the breakdown of dopamine by inhibiting the enzymes that break it down),
MAO inhibitors (similar action, but limited by side effects), and surgery. Two surgical options
include intrajejunal duodopa, which is an intrajejunal pump that releases dopamine, and deep
brain stimulation, where an implantable device is inserted near the basal ganglia to reduce
tremors. For finals, no further knowledge of these treatments is required; just know that they
are available and can be used if medications have failed.
The impaired awareness, automatisms like lip smacking, and odd behaviours all suggest
complex partial seizures. The metallic taste and epigastric sensation, often described as ‘rising’,
indicate that this seizure arises in the temporal lobe and is either partial or focal. For a guide to
other seizures, see Paper 3, Question 31, Page 42.
The correct amount of time to wait after thrombolysis before starting antiplatelet therapy in
a patient with ischaemic stroke is 24 hours. After thrombolysis, a CT head scan within 24
hours is recommended, and if this is normal, patients are given aspirin 300 mg per day for 14
days, and then clopidogrel 75 mg indefinitely. If clopidogrel is contraindicated, aspirin 75 mg
or modified release dipyridamole 200 mg can be used instead. For a detailed explanation of
stroke, see Paper 2, Question 11, Page 20.
Based on the cerebrospinal fluid sample, this patient has bacterial meningitis. The features
indicative of bacterial meningitis are high opening pressure, the presence of neutrophils (or
polymorphonucleocytes), low glucose level, and a positive Gram stain, in this case a Gram-
negative coccus with the most likely cause being Neisseria meningitidis. In viral meningitis, the
opening pressure is normal or slightly raised, protein is often normal, white cell count is often
normal and positive for lymphocytes only, and glucose is often normal. Tuberculosis meningitis
is classified by very high opening pressure and very low glucose level, cryptococcal meningitis
will produce a positive ‘India ink’ stain, and listeria meningitis will produce a Gram-positive
rod on analysis and should be suspected in elderly patients presenting with meningitis.
A table summarising the different kinds of cerebrospinal fluid is shown below:
Fungal/ Subarachnoid
Feature Normal Bacterial Viral tuberculosis haemorrhage
Opening 10–20 >30 (high) Normal High High
pressure or mildly
(cmH2O) increased
Appearance Clear Turbid, Clear Fibrin web Blood stained,
cloudy yellow
(xanthochromia)
12 hours later
Protein (g/L) 0.18–0.45 >1 (high) <1 0.1–0.5 High
Glucose 2.8–4.2 <2.2 (low) Normal 1.6–2.5 Normal
(mmol/L) (sometimes
very low)
Gram stain No Positive Normal Normal, may Normal
organisms be positive on
India Ink if
cryptococcal or
acid-fast bacilli
if tuberculosis
Glucose- >0.6 <0.4 (low) >0.6 <0.4 Normal
cerebrospinal
fluid: serum
ratio
White cell <3 >500 <1000 100–500 High
count (raised) (lymphocytes)
90%
PMN
*PMN= Polymorphonuclear leukocytes
This patient has toxoplasmosis. This is caused by the parasitic infection toxoplasma gondii
and is characterised by fever, confusion, and CNS signs. It is the most common cause of brain
lesions in patients with HIV, which is heavily implied in this patient given the past history
of tuberculosis, oesophageal candidiasis, and cytomegalovirus infections. Imaging classically
shows intracerebral calcifications, which are pathognomonic of the diagnosis and hence
distinguishing of toxoplasmosis.
CNS lymphoma would present with nonspecific symptoms. A patient with a history of
lymphoma would have solid, periventricular space-occupying lesions.
A cerebral abscess is a possibility given the history, but the lesion would be cystic, round,
and usually without calcifications.
This patient has myelomeningocele, the most severe type of spina bifida. This is because
myelomeningocele is an open defect in the spinal canal, with the contents exposed without a
covering sac. It commonly presents antenatally and is diagnosed on fetal anomaly scans or at birth
with the deformity and neurological complications evident. The neurological complications of
myelomeningocele include bladder and bowel dysfunction, as well as movement disorders
like weakness. All spina bifida types are linked to folic acid deficiency, which is the only real
risk factor. Treatment is with surgical correction often in the first few hours of delivery, and
treatment outcomes are poor, with many suffering long-term neurological disability as a result
of the defect.
Spina bifida occulta is a less severe form of spina bifida and presents asymptomatically on
examination in an otherwise well child. A textbook exam sign is a well child with a ‘tuft of hair’
over the lumbar spine area. Later in childhood, children may get problems such as leg pain and
recurrent urinary tract infections due to tethering of the cord (diastematomyelia).
Meningocele presents with a defect but less prominent neurological problems than
myelomeningocele, and unlike myelomeningocele, the defect is often closed instead of open.
Outcomes are good after surgical correction.
Encephalocele just describes extrusion of the brain and meninges through a midline skull
defect, which can be corrected surgically. Patients often have underlying associated cerebral
malformations.
Anencephaly refers to failure of development of most of the cranium and brain. Affected
infants are stillborn or die shortly after birth. It is often detected antenatally, and termination
of pregnancy is usually offered.
Brainstem lesions or pathology usually cause weakness of the ipsilateral side of the face and
weakness of the opposite side of the body.
Lesions of the cerebrum would usually only affect one side, lesions of the spinal cord
almost always affect both sides, ventricle lesions would just present with symptoms of raised
intracranial pressure due to hydrocephalus (e.g., headache, nausea, vomiting), and cavernous
sinus lesions would most likely present with cavernous sinus syndrome if symptomatic (see
Paper 2, Question 26, Page 25).
This patient has hydrocephalus (dilatation of the ventricles that carry cerebrospinal fluid),
which is suggested by the image showing massive lateral, third, and fourth ventricles. That
is probably all you need to think of for hydrocephalus for exams: massive ventricles that will
be visible to the naked eye. It is most commonly caused by obstruction of the ventricular
system which drains cerebrospinal fluid, often due to a brain tumour, cyst, or structural
defect. Treatment of acute hydrocephalus is with a ventriculoperitoneal shunt that allows the
ventricles to drain excess cerebrospinal fluid into the peritoneum in the abdomen. Malignant
middle cerebral artery syndrome would show a large, wedge-shaped area of infarct affecting
the middle cerebral artery territory (most of the scan); a frontal space-occupying lesion is not
visible on this scan; there is no air in the brain to suggest pneumocephalus; and a colloid cyst
is not present.
Remember to describe the scan fully in an OSCE station (e.g., patient details, type of
scan), and not just the diagnosis!
The patient is most likely in a long-term neurological rehabilitation ward because she suffered
a massive stroke 6 months ago. The CT scan shows a large infarct in the middle cerebral artery
territory and, as a result, the patient is likely to be left with significant neurological deficit. It
is unlikely to be oedema as there is no space-occupying lesion, and it is too large and uniform
to be vascular dementia (which would show one or lots of small hypodensities). If something
is darker on a CT scan than the surrounding brain matter, it is an infarct or less likely fluid.
The skull is completely formed, so she most likely has not had a decompressive craniectomy
(operation to reduce massive brain swelling in stroke patients).
1) Correct answer: C
This patient has Todd’s paresis, which refers to weakness of a limb after a seizure episode. It is
not very common, but most frequently occurs in focal seizures and leads to a hemiparesis or
hemiplegia that persists after a seizure (but can also include confusion, amnesia, and almost
any deficit depending on the particular anatomic epileptic focus). Patients usually recover
slowly, but it is often falsely interpreted as a stroke. Along with hemiplegic migraine, it can
resemble a stroke and is one of the stroke mimics, which can be remembered by the four S’s:
Seizures, Sepsis, Syncope, and Sugar.
2) Correct answer: A
3) Correct answer: E
The most likely diagnosis here is a cardiac arrythmia. The factors that suggest this are the
symptoms of dizziness, palpitations, and syncope, which are suggestive of existing cardiovascular
disease. Epilepsy is less likely given the lack of aura beforehand and the fast recovery without
incontinence (these are all more likely in seizures). There is no postural change to suggest a
vasovagal episode, and carotid sinus syncope is less likely here (a syncope that results from
pressure on the carotid sinus, such as from wearing tight clothes like collared shirts).
4) Correct answer: B
Myaesthenia gravis is not a cause of Horner’s syndrome, while the others — multiple
sclerosis, lung cancer, carotid dissection, and cavernous sinus thrombosis — are. See Paper 1,
Question 9, Page 3 for a detailed explanation of Horner’s syndrome.
5) Correct answer: C
6) Correct answer: C
This patient has neuromyelitis optica, which is also known as Devic’s disease. This is because the
patient has symptoms of multiple sclerosis, but cerebrospinal fluid Is negative for oligoclonal
bands — a hallmark of multiple sclerosis — and positive for anti-aquaporin 4 antibodies,
which are diagnostic. The presence of bilateral optic neuritis also makes neuromyelitis optica
a more likely diagnosis. Treatment with glucocorticoids, azathioprine or cyclophosphamide,
and/or plasmapheresis seems to be equally effective for neuromyelitis optica than for multiple
sclerosis.
Transverse myelitis describes an acute inflammatory disorder affecting the spinal cord
with subsequent loss of function. Causes are usually part of a para-infectious immune response
and may follow viral infection, such as HSV. It also occurs as part of multiple sclerosis and
neuromyelitis optica. Treatment is with high-dose steroids or other immunosuppressive
therapies.
7) Correct answer: A
This patient has dysarthria, whereby cognition (thinking processes) is normal but the muscles
controlling speech are ineffective, causing slurred speech despite cognition being intact. Causes
include stroke, space-occupying lesions, and any disruption to the speech pathway. A guide to
the different kinds of ‘dys-’ terms used in neurology is summarised below:
Dysphasia: Disordered speech due to higher sensory dysfunction — can be categorised
into Broca’s (expressive) and Wernicke’s (receptive).
Dysmetria: Lack of coordination of movement characterised by under- or overshooting.
The classical example is asking the patient to touch your finger with their finger — and they
will classically ‘overshoot’ or go past your finger.
Dysgeusia: Altered taste
Dysphonia: Altered voice (also known as hoarseness)
Dysphagia: Altered swallowing (gastrointestinal symptom!)
8) Correct answer: B
This patient has polymyositis, an inflammation of the proximal muscles (e.g., the shoulders,
hips, glutes) due to infiltration of CD8+ T cells into the muscle, leading to necrosis of muscle
fibres which causes weakness. Moreover, the patient does not have a skin rash or evidence
of skin involvement, thus ruling out dermatomyositis. Patients may also present with fever,
muscle pain, and tenderness.
The cause of myositis is unknown. Diagnosis is clinical, but creatine kinase may be raised
and muscle biopsy and electromyography may show inflammatory changes. Management is
with corticosteroids and immunosuppression.
Dermatomyositis has features of polymyositis with features of skin manifestations like
photosensitivity, a ‘heliotrope’ rash around the eyes and a linear red rash over the knuckles and
proximal phalanges (Gottron papules). In adults, it is often paraneoplastic (associated with
malignancy).
9) Correct answer: B or C
This lady has evidence of peripheral neuropathy, based on the ‘glove and stocking’ sensory
loss that is more prominent distally. The different causes of peripheral neuropathy should be
remembered (which is the purpose for this strange question!) and include diabetes, subacute
combined degeneration of the cord, alcohol abuse, and inherited disorders like Charcot-Marie-
Tooth disease. Drugs can also cause peripheral neuropathy and is most commonly linked to
anticancer drugs such as platinum compounds (e.g., cisplatin) and alkaloids (e.g., vincristine,
vinblastine), so both B and C are correct! Other important drugs that can cause peripheral
neuropathy are amiodarone (used for cardiac arrythmias) and nitrofurantoin (used for urinary
tract infections).
The correct answer here is procyclidine. This is because the patient has akathisia, which is
motor restlessness that is often involuntary. It is usually caused by antipsychotic medications,
and as this gentleman is currently undergoing active treatment for paranoid schizophrenia, he
is most likely taking one. Akathisia describes a feeling of motor ‘restlessness’ and can be treated
This patient has a strange constellation of symptoms and signs, especially given the recent stress
of her exams, but normal examination makes a functional disorder a possible diagnosis. The
consultant is testing Hoover’s sign, which is elicited by asking a patient to extend the weak
limb and then flexing the unaffected limb — it is positive if the patient automatically extends
the ‘weaker’ limb. It does not confirm a functional disorder, particularly in this lady whose
chief complaint is altered balance, but her history suggests it.
For an explanation of all the signs, see Paper 1, Question 26, Page 9.
The most likely diagnosis here is a vestibular schwannoma, based on the sensorineural hearing
loss without other symptoms. It needs to be excluded urgently, and the best way to do this is
an MRI head scan (specifically of the cerebellopontine angle). For a detailed explanation of
vestibular schwannomas, see Paper 4, Question 6, Page 48. The other investigations listed, like
the CT head scan, are less reliable and a general rule of thumb is that for acute problems like
stroke, subarachnoid haemorrhage, or bleeds, a CT scan is the best investigation to choose,
whereas for more subacute pathologies like tumours, an MRI is best.
This patient has pituitary apoplexy. This is because they have presented with thunderclap
headache, which is a common presentation of a subarachnoid haemorrhage, but the history
of having acromegaly and a previous pituitary tumour makes pituitary apoplexy more likely.
Pituitary apoplexy is an uncommon complication of pituitary tumours due to the
occurrence of infarction followed by haemorrhage into the tumour. It produces sudden onset
headache (often simulating the thunderclap headache seen in subarachnoid haemorrhage) with
rapidly progressive visual failure and extraocular nerve palsies with acute pituitary insufficiency.
It can often be managed conservatively with replacement of hormones and close monitoring
of vision, but surgical decompression may be required if there is significant deterioration in
vision.
Empty sella syndrome is often asymptomatic and describes a defect in the diaphragma
sella with extension of the subarachnoid space. All or most of the sella turcica (where the
pituitary gland lives) is devoid of apparent pituitary tissue, but pituitary function is usually
normal.
Sheehan’s syndrome is hypopituitarism due to ischaemia of the pituitary gland that occurs
after a postpartum haemorrhage. It most commonly presents with failure to establish lactation
at birth, amenorrhoea, or other features of hypopituitarism.
Yes, another question on subarachnoid haemorrhage and lumbar puncture timing — because
this is very important and one of the most likely neurological questions to come up in exams,
KNOW this pathway. The lumbar puncture would have an appearance of xanthochromia due
to bilirubin detected in the cerebrospinal fluid, which gives a classical ‘yellow’ appearance.
Bilirubin is formed as a breakdown product of red blood cells that enter the cerebrospinal
fluid due to blood from the subarachnoid haemorrhage. This is why you have to wait 12 hrs
after the onset of symptoms to perform a lumbar puncture — the red blood cells need time
to break down first, otherwise the bilirubin that you test for may be falsely recognised as not
being present. For more information on lumbar punctures, see Paper 4, Question 35, Page 58.
This patient unfortunately has brainstem death, and there are a few ways in which this can
be diagnosed. Crucially, all brainstem reflexes should be absent. This includes: pupils fixed
and unresponsive to bright light (and are often fixed and dilated), corneal reflexes absent, no
vestibulo-ocular reflexes on caloric testing, no gag reflex, and spontaneous respiration absent.
All of the other features identified are not tests used to indicate possible brainstem death.
This patient has Parinaud syndrome. This is a rare syndrome and is very unlikely to be required
for finals, so feel free to skip here. It is caused by compression of the dorsal midbrain, which
most commonly includes pineal region tumours and cysts. Other causes include hydrocephalus
and tectal plate lesions. Parinaud syndrome has the classic triad of clinical signs: small
irregular (pseudo-Argyll Robertson) pupils, supranuclear gaze palsy (inability to look up) and
convergence-retraction nystagmus. Management is by treating the cause of the dorsal midbrain
lesion; anything else is really not worth knowing!
Tertiary syphilis would just cause pseudo-Argyll Robertson pupils; a Holmes-Adie pupil
describes an asymmetrical, often dilated pupil that reacts sluggishly to light in an otherwise
healthy female and is thought to have a viral aetiology; and a Marcus Gunn pupil is one that
dilates rather than constricts in response to the swinging light test, indicating an optic nerve
lesion.
This patient unfortunately has Creutzfeldt-Jakob disease. This disease is very rare (occurring
in approximately 1 in 1 million people) and is caused by a prion protein — a rogue protein
that envelops and ‘infects’ other proteins in the brain, causing them to misfold. The exact
cause is often unknown as most cases are sporadic, but it is sometimes passed to patients
from contaminated meat (referred to as ‘mad cow disease’) or contaminated neurosurgical
instruments. Creutzfeldt-Jakob disease is characterised by rapid memory dysfunction,
personality change, general decline and death, usually within 18 months of diagnosis. On
examination, startle myoclonus (electric shock-type movements) may be found. Unfortunately,
there is no treatment for this disease, and most patients decline quickly and die within 2 years.
Benzodiazepines are GABA agonists and act on the GABA receptor to reduce depolarisation
and hence an action potential. They can be used as antiepileptic drugs, antispasmodics,
hypnotics, and sedatives. An example of a glutamate antagonist would be Riluzole, which is
used in motor neurone disease.
This patient has presented with inferior quadrantanopia — this means that the affected area
has to be the parietal lobe — because the upper optic radiation fibres that pass through the
parietal lobe actually supply the lower visual field, while the lower optic radiation fibres that
pass through the temporal lobe supply the upper visual field. If the lesion was in the temporal
lobe, it would cause superior quadrantanopia (strange, I know). The fact that the visual loss is
on the right side means that the opposite lobe of the brain must be affected, hence the answer
being the left parietal lobe.
For explanations of visual fields and defects, see Paper 1, Question 36, Page 12.
This child unfortunately has Moya Moya disease. It is highly unlikely to come up in finals, so I
would advise just knowing that it is a differential in any child with multiple ischaemic strokes.
It is characterised by idiopathic narrowing of the intracerebral arteries, leads to strokes in
children, and is more common in Japanese or Asian patients. That’s all you need to know, but
for the enthusiasts, there is a pretty cool neurosurgical operation you can do to help manage
the condition (google ‘EDAS Moyamoya’).
CADASIL stands for cerebral autosomal dominant arteriopathy with small subcortical
infarcts and leukoencephalopathy, and it is an autosomal dominant inherited condition
causing transient ischaemic strokes, strokes, and dementia in young adult patients (usually
around the age of 40).
Hurler syndrome is a rare, autosomal recessive lysosomal storage disease that usually
presents within a few weeks of birth and causes cognitive impairment, heart disease,
characteristic facies, and reduced life expectancy.
This patient has diabetic neuropathy. The reasons for this are the metabolic risk factors
like raised BMI, presence of sensory loss in a ‘glove and stocking’ pattern (with symptoms
more prevalent the more distal the limbs are), and the deformed foot — called a ‘Charcot
joint’ — which happens due to altered proprioception and sensation, causing injuries to
the foot over time due to misplacement. The presence of diabetic ulcers also increases the
likelihood of diabetes being the cause of the patient’s symptoms.
Patients’ blood sugar should be controlled within a medical range (HbA1c of 48-53
depending on whether they are taking any hypoglycaemic medications) and managed from
there. Other peripheral complications include foot ulcers, osteomyelitis (infection of the
bones), and amputation of digits.
Buerger’s disease (now known as thromboangiitis obliterans) is a non-atherosclerotic
vasculitis that is linked to an inflammatory response in the arteries and usually affects male
patients who are heavy smokers. Symptoms are associated with vascular compromise like
intermittent claudication, rest pain, or tissue loss and frequently resolves on smoking cessation.
This young boy unfortunately has subacute sclerosing panencephalitis, a rare complication
of measles that occurs many years after the initial infection. The exact nature of the disease is
unclear, but patients usually present years after the illness in childhood and adolescence with
intellectual decline, apathy, and clumsiness followed by myoclonus, ocular manifestations, and
dementia. Regrettably, there is no treatment and most patients succumb to the disease within
a few years.
Ah, the ‘localising the lesion’ question. Here is the guide. The correct answer is the pons — it is
in the brainstem because you get ipsilateral symptoms on the side of the face to the lesion with
contralateral weakness. For the pons specifically, the nerves affected are 5–8 (cranial nerves 3–4
arise from the midbrain, 5–8 from the pons, and 9–12 from the medulla), and as this patient
has an abducens nerve palsy (sixth cranial nerve), the answer must be the pons.
The correct answer here is tuberous sclerosis, which is explained in Paper 3, Question 18,
Page 38.
How you manage acute migraine is as follows: according to the latest NICE guidelines, the
management of acute migraine involves a combination of an oral triptan and an NSAID,
or an oral triptan and paracetamol. For people aged 12–17 years old, consider using a nasal
triptan over an oral triptan. If these are not effective, consider a non-oral preparation of
metoclopramide and a non-oral NSAID or triptan. For migraine, see Paper 2, Question 7,
page 19.
This patient has progressive multifocal leukoencephalopathy (PML). This is a very rare
demyelinating disease that is caused by reactivation of the John Cunningham virus and
only affects immunocompromised patients, most likely those with HIV/AIDS or are
immunocompromised due to leukaemia, post-transplants, or immunosuppressive medications
(in particular natalizumab, a monoclonal antibody treatment for multiple sclerosis). It usually
causes progressive general decline with altered mental status, motor deficits, limb and gait
ataxia, and double vision being common. The only way to diagnose PML is by brain biopsy.
Prognosis is poor and if untreated, PML is fatal within 6 months. Highly active antiretroviral
therapy and steroids may prolong survival.
Unfortunately, Samira has Sheehan’s syndrome, an ischaemic necrosis of the anterior pituitary
gland that often occurs during childbirth. As the pituitary gland doubles in size during
pregnancy and becomes highly vascular, any haemorrhage during childbirth can reduce blood
pressure and blood supply to the gland, causing necrosis. It most commonly presents after a
difficult childbirth, with the most common symptom being lactation failure due to prolactin
deficiency followed by secondary amenorrhoea. Oestrogen replacement may be required, and
depending on severity, replacement of all pituitary hormones may be required as well.
Source: Master Medicine: General and Systematic Pathology (2009)
The following questions (30–38) are shorter questions that overlap to some degree with
questions in the previous practice papers.
The correct management of status epilepticus is intravenous lorazepam 4mg, which can be
repeated once after 10 minutes if required. Buccal midazolam or rectal diazepam can be used
if there is no intravenous access or the patient is in a remote location.
For migraine prevention, the first line is propranolol. If asthma, it is topiramate, but also
remember that Ṯopiramate is Ṯeratogenic. Sumatriptan with high flow oxygen is for acute
cluster headache, while verapamil is for prevention of cluster headache.
Source: https://cks.nice.org.uk/topics/headache-cluster/management/management/
Head injury with loss of consciousness in a young patient followed by quick recovery and
then a drop in GCS level? Think extradural haematoma until proven otherwise. Elderly or
alcohol misuse plus trivial head injury? Think acute subdural haematoma. Cerebrospinal fluid
rhinorrhoea, panda eyes, bruising over the mastoid process, and haemotympanum (bleeding
from the ear)? Think basal skull fracture.
History of malignancy with new onset subacute neurological symptoms or signs? Think
cerebral metastases.
If there is suspected spinal cord compression, first administer dexamethasone and lie the patient
flat, and if the patient is fit enough and stabilised, refer them for urgent surgical decompression
or oncology input.
This scan shows an acute on chronic subdural haematoma. This is because acute blood (white)
is present in a banana or concave shape, but there is also the presence of chronic blood (black) as
part of the same haematoma. That means that this is an acute-on-chronic subdural haematoma,
most likely the result of the patient’s repeated falls. Options include conservative management
or surgical drainage if there is midline shift on scans or if the patient is symptomatic (weakness
or reduced GCS). See Paper 1, Question 38, Page 13 for more information on the different
kinds of haematomas.
There is a large space-occupying lesion on this scan, evidenced by it being round, regular,
and just a lesion! Differentials for any mass on an MRI head scan include tumours (e.g.,
meningioma, glioblastoma), cerebral abscesses (the lesion may be rounder and have a regular
outline), and metastases (most commonly from lung cancer). Strange causes like infections and
vascular pathologies (e.g., arteriovenous malformations, aneurysms, and inflammatory diseases
like multiple sclerosis plaques) are also possible, but should only be mentioned in clinical
exams after the most common causes! Further interpretation (e.g., area, extent of oedema) is
most likely not required for medical school examinations.
ABCD2 score: Age, Blood pressure, Clinical features, Duration, Diabetes — to predict stroke
risk after transient ischaemic attack.
Acute stress reaction: Stress++ event, numbness, detachment, and strange symptoms, resolve
with support.
Akathisia: Motor restlessness, desire to move limbs constantly.
Alzheimer’s disease: Degenerative disease due to Tau protein accumulation, hippocampus
affected, short-term memory loss, treat with acetylcholinesterase inhibitors.
Anterior cord syndrome: Flexion injury, loss of pain and temperature with weakness worse in
legs than arms.
Argyll Robertson pupil: Bilateral small, irregular pupil, fixed to light but constricts on
convergence — think neurosyphilis!
Arteriovenous malformation: Congenital tangle of blood vessels, presents with bleeding or
seizures.
Bamford (Oxford) stroke criteria: Unilateral weakness or sensory deficit, homonymous
hemianopia, and higher cerebral dysfunction; LACS is 1, PACS is 2, and TACS is all 3!
Basal skull fracture signs: Cerebrospinal fluid rhinorrhoea, haemotympanum, raccoon or
panda eyes, blood behind the ears (Battle’s sign).
(Benign) Essential tremor: Benign tremor, family history, improves with alcohol, treat with
propranolol or primidone.
Benign (Rolandic) epilepsy with centrotemporal spikes: Focal seizures during sleep or in the
early morning, abnormal sensation on the tongue or face, most resolve by age 16.
Brain metastases: Any patient with cancer and new headache or raised intracranial pressure
symptoms, most commonly from the lung > breast > melanoma > colon.
Brainstem death: No sleep-wake cycles or active breathing, fixed pupils, absent oculovestibular,
corneal, cough, or gag reflex (rule out reversible causes first!).
Broca’s (expressive) aphasia: Understands but cannot speak, appears frustrated, left frontal lobe
(commonly due to stroke).
Brown-Séquard syndrome: ‘One leg weak and one leg numb’, spinal cord hemisection.
Bulbar palsy: Lower motor neuron lesion of the 7th to 12th cranial nerves, flaccid tongue and
fasciculations, absent gag reflex or jaw jerk, caused by Guillain-Barré and poliomyelitis.
Carotid artery dissection: Cause of stroke in young patients, (trivial) neck injury causes tear à
clot embolises and leads to stroke.
Carpal tunnel syndrome: Weakness, paraesthesia, Tinel’s and Phalen’s sign to diagnose, treat
with conservative management → splints → steroids → surgical decompression.
Cauda equina syndrome: Back pain or sciatic pain with bladder or bowel disturbance OR
bilateral leg pain OR saddle or genital sensory disturbance — urgent MRI to rule it out,
surgical decompression needed!
Cavernoma: Benign collection of blood vessels, present with seizures — remove if symptomatic
Cavernous sinus thrombosis: Painful ophthalmoplegia, chemosis, and proptosis, caused by
sinus infection or cancer spread.
Central cord syndrome: Hyperextension injury, weakness of the upper limbs but sparing of the
lower limbs, most recover.
Central pontine myelinolysis (osmotic demyelination syndrome): Acute confusion and drop in
GCS from rapid correction of hyponatraemia (brain swelling).
Cerebral abscess triad: Fever, headaches, limb weakness (intravenous drug use and
immunocompromise) — scan shows ring-enhancing lesion, treat with antibiotics.
Cerebral perfusion pressure: Mean arterial pressure, intracranial pressure.
Charcot-Marie-Tooth disease: Autosomal dominant inherited peripheral neuropathy, pes
cavus and champagne bottle legs present.
Chiari malformation: Enlarged cerebellum protruding through foramen magnum, posterior
headaches or neck pain on coughing, treat with surgical decompression.
Claude’s syndrome: Third cranial nerve palsy and contralateral ataxia (midbrain stroke).
Cluster headache: Males, restlessness with autonomic symptoms, O2 and triptan acute with
verapamil to prevent.
Cluster headache treatment: O2 and triptan if acute, verapamil to prevent.
Coma: Lack of consciousness, no response to stimuli, no sleep-wake cycles, brainstem activity
present.
Corticobasal degeneration: + Parkinson’s + motor symptoms
Creutzfeldt-Jakob disease: Prion disease, rapid dementia, myoclonus, and death.
Cushing’s triad for raised intracranial pressure: Hypertension, bradycardia, irregular respiratory
pattern.
Degenerative cervical myelopathy: Weakness with loss of dexterity in the hands, older patients,
decompressive surgery to stop progression.
Delirium: Acute alteration of consciousness, agitated, sleep-wake cycle reversal, reversible
unlike dementia.
Dermatomyositis: Myositis plus heliotrope (violet) rash on eyelids with Gottron (violet)
papules, think malignancy!
Diffuse axonal injury: Massive brain injury (e.g., decreased blood flow from cardiac arrest),
diffuse white matter changes on scan, poor prognosis.
Encephalitis: Meningitis plus seizures and altered consciousness, HSV is the most common
cause, treat with intravenous acyclovir (antiviral).
Encephalocele: Neural tube defect, brain goes through skull defect in neonates → surgery.
Extradural haematoma: Young patient with head injury, lucid interval, lens-shaped on CT scan
(life-threatening).
Frontal lobe syndrome: Lack of executive function (planning, processing, words/speech,
inhibition) — also known as dysexecutive syndrome.
Gerstmann’s syndrome: Clinical tetrad of finger agnosia, acalculia, agraphia, and confusion
between the right and left sides.
Giant cell arteritis: Elderly, headache with jaw claudication, scalp tenderness, and polymagia
rheumatica require urgent steroids to prevent blindness!
Guillain-Barré syndrome: Ascending areflexic paralysis, previous campylobacter or diarrhoeal
infection.
Glioblastoma: Grade IV brain tumour, horrible prognosis despite surgery, radio and
chemotherapy.
Hemiplegic migraine: Weakness that persists after migraine, need to exclude stroke first.
Herniation (subfalcine): Movement of brain contents across the falx (due to mass effect from
tumour or bleeding), often asymptomatic.
Herniation (tentorial/uncal): Dilated pupil and contralateral hemiparesis, from tumour or
bleed pressing on the midbrain.
Herniation (tonsillar): Large mass causes cerebellar tonsils to go through the foramen magnum
→ breathing centres compromised.
Holmes-Adie pupil: Dilated pupil that is sluggish to light but sometimes accommodates,
young females, benign finding.
Horner’s syndrome: Ptosis, miosis, enophthalmos, facial anhidrosis (apical lung tumour is a
common cause).
Meralgia paraesthesia: Lateral cutaneous nerve of the thigh, poorly controlled diabetics —
burning pain down lateral thigh — improves with diabetes control.
Metastatic (malignant) spinal cord compression: Any cancer patient with new back pain
or neurological disturbance, lie flat and administer dexamethasone 16mg, neurosurgical or
oncology referral.
Migraine: Headache, photophobia, females, 4-72hrs — management with propranolol or
topiramate.
Miller-Fischer syndrome: Ataxia, areflexia, and ophthalmoplegia with descending weakness.
MRC grades of muscle power: 0 = no movement, 1 = flicker, 2 = gravity gone, 3 = not resistance,
4 = resistance but not full, 5 = full power.
Moya Moya disease: Child with repeated strokes, more common in Japanese children.
Motor neurone disease: Weakness, dysphagia, upper and lower motor neuron signs, incurable
(riluzole improves survival by 3 months).
Myasthenia gravis: Muscle weakness or diplopia worse after activity, thymoma in 10% of
patients, acetylcholinesterase inhibitors to treat.
Myelomeningocele: Open defect where the spinal cord is linked to folate deficiency, operated
at birth, often associated with movement and bowel or bladder function problems.
Multiple sclerosis: CNS symptoms (optic neuritis, sensory/motor) separated in time and
space — young females, relapsing-remitting the most common type.
Narcolepsy: Hypocretin deficiency — daytime sleepiness, cataplexy, hypnagogic hallucinations
(when going to sleep), sleep paralysis.
Neurofibromatosis type 1: Autosomal dominant, neurofibromas, Lisch nodules, angiofibromata
with brain tumours.
Neurofibromatosis type 2: Autosomal dominant, bilateral vestibular schwannomas with
meningiomas.
Neuromyelitis optica (NMO, Devic’s disease): Bilateral optic neuritis with spinal/CNS
symptoms- similar to MS but anti-aquaporin 4 antibodies positive.
Normal pressure hydrocephalus: Wet, wacky, wobbly — urinary incontinence, dementia, and
gait ataxia.
Panayiotopoulos syndrome: Benign epilepsy type, 1–5 years old, vomiting or autonomic
symptoms during sleep, occipital spikes on EEG, remits in adolescence.
Parinaud syndrome: Dorsal midbrain syndrome — upward gaze palsy, pseudo Argyll-robertson
pupil, and convergence retraction nystagmus.
Tuberous sclerosis: Triad of intellectual disability, intractable epilepsy, and facial angiofibromas
(adenoma sebaceum) — also presents with ash leaf macules and Shagreen patch on back.
Vestibular schwannoma (acoustic neuroma): Schwann cell tumour, unilateral sensory hearing
loss (cranial nerve VIII) with additional fifth and seventh cranial nerve involvement; think
neurofibromatosis type 2 if bilateral!
Weber’s syndrome: Third cranial nerve palsy and contralateral hemiplegia, midbrain infarct or
lesion.
Wernicke’s (receptive) aphasia: Temporal lobe, patient talks fluently but words make no sense!
Wernicke’s encephalopathy: Acute vitamin B1 deficiency, triad of confusion, ataxia, and
ophthalmoplegia.
Wilson’s disease: Autosomal recessive disorder of copper metabolism, psychiatric disturbance
in children, Kayser-Fleischer ring around the eye.
Index
Benign Paroxysmal Positional Vertigo, 53 Headache, 3, 6, 19, 24, 29, 33, 40–42, 47, 50,
Brain Tumour, 24, 27, 33, 38, 48, 49, 67, 53, 71
73, 76 Head Injury, Management, 1, 4, 26, 29, 39, 48
Brown-Séquard Syndrome, 11 Herniation, Cerebral, 25
Horner’s Syndrome, 64
Cauda Equina Syndrome, 25, 55 Huntington’s Disease, 18
Cavernous Sinus Thrombosis, 25 Hydrocephalus, 10, 52, 60
Central Cord Syndrome, 56
Central Pontine Myelinolysis, 55 Idiopathic Intracranial Hypertension, 2, 34
Cerebral Abscess, 2, 30 Infantile Spasms, 71
cerebral Palsy, 53
Chiari Malformation, 41 Korsakoff’s Syndrome, 37, 43
Claude Syndrome, 35
Cluster Headache, 6, 29, 50 Lambert-Eaton Myaesthenic Syndrome, 20, 63
Concussion, 54 Lewy Body Dementia, 35
Consciousness, level of, 39, 68 Localising the Lesion/Anatomy, 21, 23, 28, 59,
Cranial nerve palsy, 12, 48, 51 69, 70
Creutzfeldt-Jakob Disease, 22, 68 Lumbar Puncture, 5, 51, 57, 67, 74
Cushing’s Triad, 1
Meningitis, 35, 44, 57
Degenerative Cervical Myelopathy, 24 Meralgia Paraesthesia, 37
Degenerative Spinal Cord Disease, 27, 47 Metastases, Brain, 5
Delirium, 31, 36 Metastatic Spinal Cord Compression (MSCC),
Dementia, 21, 35, 64 11, 39, 74
Dermatomes, 9, 27, 49 Migraine, 3, 19, 52, 70, 72
Diabetic Neuropathy, 70 Millard-Gubler Syndrome, 35
Diffuse Axonal Injury, 13 MRC Power Assessment, 22, 73
Dissection, Vertebral Artery, 50 Motor Neurone Disease, 4, 18
Moya Moya Disease, 69
Encephalitis, 35 Multiple Sclerosis, 17, 27, 28, 33, 42
Essential Tremor, 37, 38 Multiple Systems Atrophy, 73
Extradural Haematoma, Acute, 23, 29, 73 Myaesthenia Gravis, 20, 56
Scan Interpretation, 13, 14, 29, 30, 45, 46, 60, Visual Disturbance, 39
61, 75, 76
Seizures, Types of, 18, 28, 36, 38, 42, 43, 57, Weber Syndrome, 35
63, 70 Wernicke’s Encephalopathy, 8, 43
Seizures, Treatment Of, 5, 43
Sheehan’s Syndrome, 72 Upper and Lower Motor Neuron, 10, 38